You are on page 1of 49

POLITICAL LAW REVIEW

Part I
by
MIGUEL B. LICERALDE SR.
University of the Cordilleras

Chapter I.

A. INTRODUCTION
1. Political law defined [Macariola v. Asuncion, 114
SCRA 77]

That branch of public law which deals with the organization
and operations of the governmental organs of the State and
defines the relations of the State with the inhabitants of its
territory.

Macariola v. Asuncion
A.M. No. 133-J May 31, 1982
114 SCRA 77

The provision in the Code of Commerce which prohibits
judges, justices, etc., (public officers) from engaging in
business within the territorial jurisdiction of their courts is
political in nature and therefore, said provision was deemed
abrogated when there was a change of sovereignty from
Spain to the United States at the turn of the century. Political
laws are deemed abrogated if there is a change of
sovereignty and unless re-enacted under the new sovereign,
the same is without force and effect.

On 6 Aug 1968, Macariola filed a complaint against Judge
Asuncion with acts unbecoming a judge. The judge apparently
bought a property (formerly owned by Macariola) which was
involved in a civil case decided by him; and on 31 Aug 1966, the
Asuncion couples conveyed their share and interest in the said
property to The Traders Manufacturing and Fishing Industries Inc.
The act of Asuncion engaging in commerce is said to be a
violation of pars 1 & 5, Art 14 of the Code of Commerce which
prohibits judges in active service (among others) to do so within
the limits of the place where they discharge their duties.
HELD: Art 14 (Anti Graft and Corrupt Practices Act, effective Aug
1888) of the Code of Commerce, prohibiting judges from
engaging in commerce was political in nature and so was
automatically abrogated with the end of Spanish rule in the
country (Change of Sovereignty to the US by virtue of cession,
1898).

2. Scope of Political Law

Constitutional Law
Administrative Law
Law of Public Officers
Law on Municipal Corporations
Election Law

3. Constitution-defined and classified.

That written instrument enacted by direct action of the
people by which the fundamental powers of the government
are established, limited, and defined, by which those powers
are distributed among the several departments for their safe
and useful exercise for the benefit of the benefit of the body
politic. - Sir Liceralde, (Malcolm, Philippine Constitutional Law,
p.6)

Kinds of Constitution
Unwritten
Consists of rules, which have not been integrated into a
single concrete form but are scattered in various sources, such as
statutes of a fundamental character, judicial decisions,
commentaries of publicists, customs and traditions and certain
common law principle.

Written
One whose precepts are embodied in one document or set
of documents.

Conventional
Enacted constitution, formally struck off at a definite time
and place following a conscious and deliberate effort taken by a
constituent body or ruler.

Cumulative
The result of a political evolution not inaugurated at any
specific time but changing by accretion rather than by any
systematic method.

Rigid
Is one that can be amended only by a formal and
usually difficult process.

Flexible
Is one that can be changed by an ordinary legislation.

4. Parts of a written constitution

Constitution of liberty
Consists of series of prescriptions setting forth the
fundamental civil and political rights of the citizens and imposing
limitations on the powers o government as a means of securing
the enjoyment of those rights. (Articles III, II, IV. V and XII)

Constitution of government
Series of provisions outlining the organization of the
government, enumerating its powers, laying down certain rules
relative to its administration and defining the electorate. (Articles
VI and XI)

Constitution of Sovereignty
Consists of provisions pointing out the mode or procedures
in accordance with which the formal changes in the fundamental
law maybe brought about and change. (Article XVII)

5. Interpretation & construction of the Constitution;

a. Self-executing & non self-executing (Manila
Prince Hotel vs. GSIS, 267 SCRA 408
[1997].

A provision which lays down a general principle, such
as those found in Art. II of the 1987 Constitution is usually
not self-executing. But a provision, which is complete in
itself and becomes operative without the aid of
supplementary or enabling legislation, or that which
supplies sufficient rule by means of which the right it
grants may be enjoyed or protected, is self-executing. Thus
a constitutional provision is self-executing if the nature and
extent of the right conferred and the liability imposed are
fixed by the constitution itself, so that they can be
determined by an examination and construction of its
terms, and there is no language indicating that the subject
is referred to the legislature for action.


Facts:

The Government Service Insurance System (GSIS), pursuant to
the privatization program of the Philippine Government under
Proclamation 50dated 8 December 1986, decided to sell
through public bidding 30% to 51%of the issued and
outstanding shares of the Manila Hotel (MHC). In a close
bidding held on 18 September 1995 only two bidders
participated: Manila Prince Hotel Corporation, a Filipino
corporation, which offered to buy 51% of the MHC or
15,300,000 shares atP41.58 per share, and Renong Berhad,
a Malaysian firm, with ITT-Sheraton as its hotel operator, which
bid for the same number of shares at P44.00 per share, or
P2.42 more than the bid of petitioner. Pending the declaration of
Renong Berhard as the winning bidder/strategic partner and the


execution of the necessary contracts, the Manila Prince Hotel matched the bid
price of P44.00per share tendered by Renong Berhad in a letter to
GSIS dated 28 September 1995. Manila Prince Hotel sent a
managers check to the GSIS in a subsequent letter,
but which GSIS refused to accept. On 17 October 1995, perhaps
apprehensive that GSIS has disregarded the tender of the
matching bid and that the sale of 51% of the MHC may
be hastened by GSIS and consummated with Renong Berhad,
Manila Prince Hotel came to the Court on prohibition and
mandamus. The next day the Court issued a temporary restraining
order enjoining the respondents from perfecting and
consummating the sale to the Malaysian firm.

Issue(s):

1. Whether the provisions of the Constitution, particularly
Article XIISection 10, are self-executing.

2. Whether the 51% share is part of the national patrimony.

Held:

The 1987 Philippine Constitution, Article XII Section 10 provides: The Congress
shall, upon recommendation of the economic and planning agency,
when the national interest dictates, reserve to citizens of the
Philippines or to corporations or associations at least sixty per
centum of whose capitalist owned by such citizens, or such higher
percentage as Congress may prescribe, certain areas of investments.
The Congress shall enact measures that will encourage the formation and
operation of enterprises whose capital is wholly owned by
Filipinos. In the grant of rights, privileges, and concessions
covering the national economy and patrimony, the State shall give
preference to qualified Filipinos. The State shall regulate and
exercise authority over foreign investments within its national
jurisdiction and in accordance with its national goals and
priorities.

b. Art. II provisions generally not self-executing
provisions (Kilosbayan v. Morato, 246 SCRA
540)

Facts:
a. Result of GR 113375 (KIlosbayan vs. Guingona)- invalidated
contract of lease between Phil.Charity Sweepstake Office (PCSO)
and Phil. Gaming Management Corp. (PGMC)
i. To be in violation of the charter of PCSO

b.Jan 25, 1995- parties signed an Equipment Lease Agreement
(ELA) for lease of online lotteryequipment and accessories

i. Rental is 4.3% of gross amount of ticket sales by PCSO
at which in no case be lessthan an annual rental
computed at P35,000 per terminal in commercial
operation.
ii. Rent is computed bi-weekly
iii. Term is 8 years
iv. Upon expiration of term, PCSO can
purchase the equipment at P25M

c.Petitions declared ELA as invalid

i. Amended ELA is null and void being the same with the
old lease contract

ii. Assuming ELA is materially different from the old lease
contract, it is still inconsistent with the PCSOs charter

iii. Amended ELA is null and void for being violative of the
law on public bidding, it has not been approved by
the President and it is not most advantageous to the govt.

d.PCSO and PGMC filed separate comments:

i. ELA is a different lease contract with none of the
vestiges in the prior contract

ii.ELA is not subject to public bidding because it fell in the
exception provided in EO No. 301

iii. Power to determine if ELA is advantageous vests in the
BOD of PCSO

iv. Lack of funds of PCSO cannot purchase the its own
online lottery equipment

v. Petitioners seek to further their moral crusade

vi. Petitioners do not have a legal standing because they
were not parties to the contract

Issues:
a. Does the petitioners have legal standing?
b. Is the ELA valid?

Held/Ratio:

a. NO. Petitioners do not have a legal standing.

STARE DECISIS cannot apply. The previous ruling
sustaining the standing of the petitioners is a departure from
the settled rulings on real parties in interest because no
constitutional issues were actually involved. Also, LAW OF THE
CASE (opinion delivered on a former appeal) cannot also apply.
Since the present case is not the same one litigated by the
parties before in Kilosbayan vs.Guingona, Jr., the ruling cannot
be in any sense be regarded as the law of this case. The
parties are the same but the cases are not.

RULE ON CONCLUSIVENESS OF JUDGMENT cannot
still apply. An issue actually and directly passed upon and
determine in a former suit cannot again be drawn in question in
any future action between the same parties involving a different
cause of action. But the rule does not apply to issues of law at
least when substantially unrelated claims are involved. When
the second proceeding involves an instrument or transaction
identical with, but in a form separable from the one dealt with in
the first proceeding, the Court is free in the second proceeding
to make an independent examination of the legal matters at
issue. Since ELA is a different contract, the previous decision
does not preclude determination of the petitioners standing.

Accordingly, STANDING is a concept in constitutional
law and here no constitutional question is actually involved. The
more appropriate issue is whether the petitioners are
REALPARTIES in INTEREST.

b.YES. ELA is valid. It is different with the prior lease
agreement:1. PCSO now bears all losses because the operation
of the system is completely in its hands ii. Fixing the rental rate
to a minimum is a matter of business judgment and the Court is
not inclined to review. iii.Rental rate is within the 15% net
receipts fixed by law as a maximum. (4.3% of gross receipt is
discussed in the dissenting opinion of Feliciano, J.).

In the contract, it stated that the parties can change their
agreement. Petitioner states that this would allow PGMC to
control and operate the on-line lottery system. The Court held
that the claim is speculative. In any case, in the construction of
statutes, the presumption is that in making contracts, the
government has acted in good faith. The doctrine that the
possibility of abuse is not a reason for denying power.

It was held in Kilosbayan Vs. Guingona that PCSO does
not have the power to enter into any contract which would
involve it in any form of collaboration, association, or joint
venture for the holding of sweepstakes activities. This only
mentions that PCSO is prohibited from investing in any activities
that would compete in their own activities.

It is claimed that ELA is a joint venture agreement which
does not compete with their own activities. The Court held that
is also based on speculation. Evidence is needed to show that
the transfer of technology would involve the PCSO and its
personnel in prohibited association with the PGMC.

E.O. 301 (on law of public bidding) applies only to
contracts for the purchase of supplies, materials and equipment


and not on the contracts of lease. Public bidding for leases is only
for privately-owned buildings or spaces for government use or
of govt owned buildings or spaces for private use.

In a nutshell, petitioners have no standing. ELA is a valid
lease contract. Petition for prohibition, review and/or injunction is
dismissed.

6. Historical background:
a. Malolos Constitution
b. American Organic Acts
c. 1935 Constitution
d. 1973 Constitution
e. 1985 snap presidential election

The Philippines has had a total of six constitutions since the
Proclamation of Independence on June 12, 1898. In 1899, the
Malolos Constitution, the first Philippine Constitutionthe first
republican constitution in Asiawas drafted and adopted by the
First Philippine Republic, which lasted from 1899 to 1901.

During the American Occupation, the Philippines was
governed by the laws of the United States of America. Organic
Acts were passed by the United States Congress for the
administration of the Government of the Philippine Islands. The
first was the Philippine Organic Act of 1902, which provided for a
Philippine Assembly composed of Filipino citizens. The second
was the Philippine Autonomy Act of 1916, which included the first
pledge of Philippine independence. These laws served as
constitutions of the Philippines from 1902 to 1935.

In 1934, the United States Congress passed the Philippine
Independence Act, which set the parameters for the creation of a
constitution for the Philippines. The Act mandated the Philippine
Legislature to call for an election of delegates to a Constitutional
Convention to draft a Constitution for the Philippines. The 1934
Constitutional Convention finished its work on February 8, 1935.
The Constitution was submitted to the President of the United
States for certification on March 25, 1935. It was in accordance
with the Philippine Independence Act of 1934. The 1935
Constitution was ratified by the Filipino people through a national
plebiscite, on May 14, 1935 and came into full force and effect on
November 15, 1935 with the inauguration of the Commonwealth of
the Philippines. Among its provisions was that it would remain the
constitution of the Republic of the Philippines once independence
was granted on July 4, 1946.

In 1940, the 1935 Constitution was amended by the National
Assembly of the Philippines. The legislature was changed from a
unicameral assembly to a bicameral congress. The amendment
also changed the term limit of the President of the Philippines from
six years with no reelection to four years with a possibility of being
reelected for a second term.
During World War II the Japanese-sponsored government nullified
the 1935 Constitution and appointed Preparatory Committee on
Philippine Independence to replace it. The 1943 Constitution was
used by the Second Republic with Jose P. Laurel as President.

Upon the liberation of the Philippines in 1945, the 1935
Constitution came back into effect. The Constitution remained
unaltered until 1947 when the Philippine Congress called for its
amendment through Commonwealth Act No. 733. On March 11,
1947 the Parity amendment gave United States citizens equal
rights with Filipino citizens to develop natural resources in the
country and operate public utilities. The Constitution, thereafter,
remained the same until the declaration of martial law on
September 23, 1972.

Before President Marcos declared Martial Law, a
Constitutional Convention was already in the process of
deliberating on amending or revising the 1935 Constitution. They
finished their work and submitted it to President Marcos on
December 1, 1972. President Marcos submitted it for ratification in
early January of 1973. Foreseeing that a direct ratification of the
constitution was bound to fail, Marcos issued Presidential Decree
No. 86, s. 1972, creating citizens assemblies to ratify the newly
drafted constitution by means of a Viva Voce vote in place of
secret ballots. Marcos announced that it had been ratified and in
full force and effect on January 17, 1973. Although the 1973
Constitution had been ratified in this manner, opposition
against it continued. Chief Justice Roberto V. Concepcion in his
dissenting opinion in the case ofJavellana v. Executive
Secretary, exposed the fraud that happened during the citizens
assembly ratification of the 1973 Constitution on January, 10
15, 1973. However, the final decision of this case was that the
ratification of the 1973 Constitution was valid and was in force.

When democracy was restored in 1986, President
Corazon C. Aquino issued Proclamation No. 3, suspending
certain provisions of the 1973 Constitution and promulgating in
its stead a transitory constitution. A month later, President
Aquino issued Proclamation No. 9, s. 1986, which created a
Constitutional Commission tasked with writing a new charter to
replace the 1973 Constitution. The commission finished its
work at 12:28 a.m. of October 16, 1986. National Plebiscite was
held on February 2, 1987, ratifying the new constitution. On
February 11, 1987, by virtue of Proclamation No. 58, President
Aquino announced the official canvassing of results and the
ratification of the draft constitution. The 1987 Constitution finally
came into full force and effect that same day with the President,
other civilian officials, and members of the Armed Forces
swearing allegiance to the new charter.

f. READ: concurring opinion of J. Puno in
Republic v. Sandigan, 407 SCRA 10

It is an established rule that the State cannot be
estopped by the mistakes of its agents.

States right to due process of law; Double jeopardy; dismissal
of a criminal case based on Demurrer to evidence---whether it
could be the subject of a Petition for Certiorari under Rule 65 by
the PEOPLE OF THE PHILIPPINES.

g. Nature of government after Feb. 25, 1986 up to
the proclamation (No. 3) of the Freedom
Constitution on March 24, 1986 (REPUBLIC
v. Sandigan, 407 SCRA 10)

FACTS: P e t i t i o n e r R e p u b l i c , t h r o u g h t h e
Presidential Commission on Good Government
( P C G G ) , r e p r e s e n t e d b y t h e O f f i c e o f t h e
Sol i c i t or Gener al ( OSG) , f i l ed a pet i t i on f or forfei
ture before the Sandiganbayan. Petitioner
s o u g h t t h e d e c l a r a t i o n o f t h e
a g g r e g a t e amount of US$356 mi ll i on (now
est imat ed t o
b e m o r e t h a n U S $ 6 5 8 m i l l i o n i n c l u s i v e o f
interest) deposited in escrow in the PNB, as ill-gotten
wealth. The funds were previously
heldby t he f ol l owi ng f i v e ac c ount gr oups , us i ng
v ar i ous f or ei gn f oundat i ons i n c er t ai n Swi s s ba
nk s . I n addi t i on, t he pet i t i on s ought t heforfeiture
of US$25 million and US$5 million in
t r eas ur y not es whi c h exc eed ed t h e Mar c os
coupl es sal ari es, ot her l awf ul i ncome as
wel l a s i n c o m e f r o m l e g i t i m a t e l y a c
q u i r e d property.The t r eas ur y not es ar e f r o z en at
t he Central Bank of the Philippines by virtue of the f reeze
order i ssued by t he PCGG. Bef ore t he case was set for
pre-trial, a General Agreement
a n d t h e S u p p l e m e n t a l A g r e e m e n t d a t
e d Dec ember 28, 1993 wer e ex ec ut ed by t heM
a r c o s c h i l d r e n a n d t h e n P C G G C h a i r m a n M
agtanggol Gunigundo for a global settlement of t he asset s of
t he Marcos f ami ly t o i dent if y, col l at e, cause t he
i nvent ory of and dist ri but e all assets presumed to be
owned by the Marcos family under the conditions contained
therein.

I SSUE: WON t he Swi ss f unds can be f orf ei t ed i n
f av or of t he Repu bl i c , on t he bas i s of t h e
Marcoses lawful income.

HELD: NO.

R A 1 3 7 9 r a i s e s t h e p r i m a f a
c i e p r e s u m p t i o n t h a t a p r o p e r t y i s u n l a w f


u l l y ac qui r ed, henc e s ubj ec t t o f or f ei t ur e, i f i t s
amount or value is manifestly disproportionate to the official salary
and other lawful income of t he publ i c of fi cer who owns i t .
The f ol l owi ng
f a c t s m u s t b e e s t a b l i s h e d i n o r d e r t h a
t forfeiture or seizure of the Swiss deposits maybe effected:

(1) ownership by the public officer
o f m o n e y o r p r o p e r t y a c q u i r e d d u r i n g h i s i n
c u m b e n c y , w h e t h e r i t b e i n h i s n a m e o r ot h
er wi s e, and

( 2) t he ext ent t o whi c h t he amount of that money or
property exceeds,
i.e . , i s g r o s s l y d i s p r o p o r t i o n a t e t o ,
t h e legitimate income of the public officer.

The s pous es Fer di nand and I mel da Mar c os
were public officials during the time material to the present case
was never in dispute.

T h e s p o u s e s a c c u m u l a t e d s a l a r y o f $304,3
72.43 should be held as the only known l awf ul i ncome of t he
Marcoses si nce t hey di d not file any Statement of Assets and
Liabilities (SAL), as required by law, from which their net worth
could be determined. Besides, under the
1935 Cons t i t ut i on, Fer di nand E. Mar c os as P r e
s i d e n t c o u l d n o t r e c e i v e
" a n y o t h e r emolument from the Government or any of
its subdi vi si ons and i nstrument al it i es".

Li kewi se, und er t he 1973 Cons t i t ut i on, Fer di na
nd E. Marcos as Presi dent coul d "not recei ve duri ng
h i s t e n u r e a n y o t h e r e m o l u m e n t f r o m t h e
Government or any other source."
T h e i r o n l y k n o w n l a w f u l i n c o m e o f $304,372.
43 can t heref ore l egal l y and f airl yserve as basis for
determining the existence of a pr i ma f ac i e c as e of
f or f ei t ur e of t h e Swi s s f unds. The Republ i c di d not
f ail t o est abl i sh a prima facie case for the forfeiture of the
Swiss deposits.
T h e S w i s s d e p o s i t s w h i c h
w e r e t ransf erred t o and are deposi t ed i n escrow at
t he Phi l i ppi ne Nat i onal Bank i n t he est i mat ed aggregate
amount of US$658,175,373.60 as of 31 January 2002, plus
interest, were forfeited in favor of the Republic.

h. Freedom Constitution

Freedom Constitution overtook the 1973 constitution as
the constitutional convention led by the late Cecilia Munoz Palma
in drafting the new constitution.

By the order of the president, Proclamation No. 9 created
a constitutional Commission composed of 50 members to be
appointed by her. Took effect on Feb 2, 1987, the date of its
ratification. (covers pending cases during the Freedom
Constitution).

B. THE 1987 CONSTITUTION-
1. Adoption and ratification on 02 Feb. 1987; its effectivity
(De Leon v. Esguerra, 153 SCRA 602)

Facts: Alfredo de Leon won as barangay captain and other
petitioners won as councilmen of barangay Dolores, taytay, rizal.
On February 9, 1987, de leon received memo antedated
december 1, 1986signed by OIC Gov. Benhamin Esguerra,
february 8, 1987, designating Florentino Magno, as new captain
by authority of minister of local government and similar memo
signed February 8, 1987,designated new councilmen.

Issue: Whether or not designation of successors is valid.

Held: No, memoranda has no legal effect.

1. Effectivity of memoranda should be based on the date when it
was signed. So, February 8, 1987and not December 1, 1986.

2. February 8, 1987, is within the prescribed period. But
provisional constitution was no longer in
efffect then because 1987 constitution has been ratified and its
transitory provision, Article XVIII, sec. 27 states that all previous
constitution were suspended.

3. Constitution was ratified on February 2, 1987. Thus, it
was the constitution in effect. Petitioners now acquired security
of tenure until fixed term of office for barangay officials has been
fixed. Barangay election act is not inconsistent with constitution.


2. Supremacy of the Constitution (Manila Prince Hotel
vs. GSIS, 267 SCRA 408 [1997];

A constitution is a system of fundamental laws for the
governance and administration of a nation. It is
supreme, imperious, absolute and unalterable except
by the authority from which it emanates. It has been
defined as the fundamental and paramount law of the
nation. It prescribes the permanent framework of a
system of government, assigns to the different
departments their respective powers and duties, and
establishes certain fixed principles on which
government is founded. The fundamental conception in
other words is that it is a supreme law to which all
other laws must conform and in accordance with which
all private rights must be determined and all public
authority administered.

Under the doctrine of constitutional supremacy, if
a law or contract violates any norm of the
constitution that law or contract whether
promulgated by the legislative or by the executive
branch or entered into by private persons for
private purposes is null and void and without any
force and effect. Thus, since the Constitution is the
fundamental, paramount and supreme law of the
nation, it is deemed written in every statute and
contract.

3. exception-Lawyers League for a Better Philippines
v. Aquino, GR 73748, 5/22/86; In re: Bermudez

FACTS: On February 25, 1986, President Corazon Aquino
issued Proclamation No. 1 announcing that she and Vice
President Laurel were taking power.2.On March 25,
1986, proclamation No.3 was issued providing the basis of the
Aquino government assumption of power by stating that the
"new government was installed through a direct exercise of the
power of the Filipino people assisted by units of the New Armed
Forces of the Philippines."

ISSUE: Whether or not the government of Corazon Aquino is
legitimate.

HELD:Yes. The legitimacy of the Aquino government is not a justiciable
matter but belongs to the realm of politics where only the people are
the judge. The Court further held that:

1.The people have accepted the Aquino government which is in
effective control of the entire country;

2.It is not merely a de fact on government but in fact and law a de jure
government; and

3.The community of nations has recognized the legitimacy of
the new government.

A. PREAMBLE
We, the sovereign Filipino people, imploring the aid of almighty
god, in order to build a just and humane society and establish a
government that shall embody our ideals and aspirations,
promote the common good, conserve and develop our
patrimony, and secure to ourselves and our posterity the
blessings of independence and democracy under the rule of law
and a regime of truth, justice, freedom, love, equality and peace,
do ordain and promulgate this constitution.



a. 1987 Preamble compared with the 1973,1935

Changes in the Preamble of the Philippine Constitution:

Rolando Suarez: Introduction to Law.

1. Just like the 1935 and the 1973 preamble, the 1987
preamble is not a source of rights or obligations. It is
merely an aid in ascertaining the meaning of ambiguous
provisions in the body of the constitution.
2. Imploring the aid of Divine Providence now appear
imploring the aid of Almighty God to make the reference
to God more personal and direct. And consonant with
Filipino religiosity.
3. The words general welfare now read common good
the word general being indicative of exceptions.
4. The words patrimony of the nation now read our
patrimony to make it more emphatic.
5. The words blessings of democracy now reads
blessings of independence and democracy to
underscore the importance of true independence even in
the presence of democratic beliefs and practices.
6. The last portion of the preamble contains new words
such as rule of law truth freedom and love. There is
a need to assert the need for love in the face of varying
political and social beliefs. Freedom includes freedom
from fear and from want. TRUTH emphasizes the need
to act on the basis of real facts and figures

Changes if the Preamble as discussed by Fr. Joaquin Bernas
in the 1987 Constitution of the Philippines: A Commentary
2003 ed.
1. Guidance to aid- as the more all-embracing term.
2. Enhance to conserve and develop
3. Addition to more dynamic word aspiration to the
passive sounding ideals.
4. general welfare to common good- intended to project
the idea o social order that enables every citizen to
attain his or her fullest development economically,
politically, culturally and spiritually. The rejection of
the phrase general welfare was based on the
apprehension that the phrase could be interpreted as
meaning greatest good for the greatest number, to
avoid discrimination, persecution of the minority and
mob rule.
5. rule of law was inserted instead of the words rule or
regime
6. Love- it serves as a monument to th elove that
prevented bloodshed in the February Revolution of
1896.
7. Truth is a protest against the deception that
characterized the Marcos regime.
8. Truth Justice freedom love captures a stream in
Catholic thought which sees peace as the fruit of the
convergence of these words.


b. Function of the preamble. Is it a source of rights? What
are its purposes and effects?

The preamble is not a source of right but a source of light
because it sets down origin, purpose and scope of the
Constitution, which is useful as an aid in ascertaining the meaning
of ambiguous provisions of the body of the Constitution.

The preamble bears witness to the fact that the Constitution
is the manifestation of the sovereign will of the Filipino people.

READ: Aglipay v. Ruiz, 64 Phil 20I

It is almost trite to say now that in this country we enjoy both
religious and civil freedom. All the officers of the Government,
from the highest to the lowest, in taking their oath to support and
defend the constitution, bind themselves to recognize and respect
the constitutional guarantee of religious freedom, with its inherent
limitations and recognized implications. It should be stated that
what is guaranteed by our Constitution is religious liberty, not
mere religious toleration.

Religious freedom, however, as a constitutional mandate
is not inhibition of profound reverence for religion and is not
denial of its influence in human affairs. Religion as a profession
of faith to an active power that binds and elevates man to his
Creator is recognized. And, in so far as it instills into the minds
the purest principles of morality, its influence is deeply felt and
highly appreciated. When the Filipino people, in the preamble of
their Constitution, implored "the aid of Divine Providence, in
order to establish a government that shall embody their ideals,
conserve and develop the patrimony of the nation, promote the
general welfare, and secure to themselves and their posterity
the blessings of independence under a regime of justice, liberty
and democracy," they thereby manifested reliance upon Him
who guides the destinies of men and nations. The elevating
influence of religion in human society is recognized here as
elsewhere. In fact, certain general concessions are
indiscriminately accorded to religious sects and denominations.

Facts:
The petitioner (Gregorio Aglipay) is the Supreme Head of
the Philippine independent Church that seeks the issuance from
the court of writ of prohibition preventing the respondent
Director of Post (Ruiz) from issuing and selling a
commemorative stamp of the 33
rd
International Eucharistic
Congress as enacted and appropriated under RA 4052.
The petitioner contented that RA 4052 is unconstitutional
as it violated Article VI Section 29 (2) stating that no public
money shall be appropriated, applied or paid to any sect, church
or any system of religion including its priests preacher etc
The law in question appropriates P60, 000 for printing the
new commemoration stamps. (The stamp design is suggestive
of Roman Catholicism i.e chalice)

Issue:
Does RA 4052 unconstitutional as it violates the provision
of the constitution specifically Article VI Section 29 (2)?

Held:
No. RA 4052 does not violate the provision of the
constitution as alleged by the petitioner.
In the case at bar, the commemorative stamp are not
issued and sold to appropriate or pay the Roman Catholic
Church but its purpose is to advertise the Philippines and attract
tourist in the Philippines. The proceeds are not for any religious
institution but for the interest of the public.

ARTICLE II.- FUNDAMENTAL PRINCIPLES AND STATE
POLICIES-

Generally not self-executing provisions (KILOSBAYAN V.
MORATO, 246 SCRA 540)

Supra.

Section 1. The Philippines is a democratic and republican state.
Sovereignty resides in the people and all government authority
emanates from them.

III. PHILIPPINES AS A STATE

a. State defined; distinguished from nation Collector vs.
Rueda, 42 SCRA 23 [1971]

State is a legal or juristic concept, while nation is an ethnic
or racial concept.

Facts: The respondent is the administrator of the estate of the
decease Maria Cerdera located in Tangier, Morocco. After
deliberation and assessment, the petitioner demands the
respondent a sum of money as inheritance and deficiency
estate tax.

The respondent insisted that the estate of Maria Cerdera is
exempt from such taxes as provided under Section 122 of the
National Internal Revenue Code. As an answer, the petitioner
contested the grounds of the respondents stating that Tangier is
not considered foreign country as prescribed and that it is not
reciprocal under the same Code thereby not exempt from taxes.



Issue: Does Tangier can be considered as a foreign country
under Section 122 of the National Internal Revenue Code thereby
exempting the estate of the deceased Maria Cerdera from taxes?

Held: Yes. Tangier is foreign country and thereby the estate of
the decedent should be exempted from taxes. It is also reciprocal
to the provisions under the NIR Code.
Foreign country refers to the government of that foreign
power, which, although not an international person in the sense of
international law, does not impose transfer or death taxes upon
intangible personal properties of our citizen not residing therein,
or whose law allows a similar exemptions from such taxes,
therefore, the law on Tangier is reciprocal to that of the provision
on the National Internal Revenue Code.

It does not necessarily to have international personality for
Tangier to be considered as foreign country.

b. Elements of State (people, territory, government, sovereignty)

As held in COLLECTOR VS. CAMPOS RUEDA, 42 SCRA 23, the
elements of a state are:
1. people;
2. territory;
3. sovereignty; and
4. government.


1). PEOPLE
a). As inhabitants-Sec. 1,Art. XIII; Sec. 15, Art. II; Sec. 2,
Art. III

ARTICLE XIII
SOCIAL JUSTICE AND HUMAN RIGHTS
Section 1. The Congress shall give highest priority to the
enactment of measures that protect and enhance the right of all
the people to human dignity, reduce social, economic, and political
inequalities, and remove cultural inequities by equitably diffusing
wealth and political power for the common good.
To this end, the State shall regulate the acquisition, ownership,
use, and disposition of property and its increments.
ARTICLE II
DECLARATION OF PRINCIPLES AND STATE
POLICIES

Section 15. The State shall protect and promote the right to health
of the people and instill health consciousness among them.
ARTICLE III
BILL OF RIGHTS
Section 1. No person shall be deprived of life, liberty, or property
without due process of law, nor shall any person be denied the
equal protection of the laws.
Section 2. The right of the people to be secure in their persons,
houses, papers, and effects against unreasonable searches and
seizures of whatever nature and for any purpose shall be
inviolable, and no search warrant or warrant of arrest shall issue
except upon probable cause to be determined personally by the
judge after examination under oath or affirmation of the
complainant and the witnesses he may produce, and particularly
describing the place to be searched and the persons or things to
be seized.

b). As citizens- Preamble; Sec. 1 & 4, Art. II; Sec. 7, Art. III

PREAMBLE
We, the sovereign Filipino people, imploring the aid of
Almighty God, in order to build a just and humane society,
and establish a Government that shall embody our ideals
and aspirations, promote the common good, conserve and
develop our patrimony, and secure to ourselves and our
posterity, the blessings of independence and democracy
under the rule of law and a regime of truth, justice,
freedom, love, equality, and peace, do ordain and
promulgate this Constitution.

ARTICLE II
DECLARATION OF PRINCIPLES AND STATE POLICIES
PRINCIPLES
Section 1. The Philippines is a democratic and republican State.
Sovereignty resides in the people and all government authority
emanates from them.
Section 4. The prime duty of the Government is to serve and
protect the people. The Government may call upon the people
to defend the State and, in the fulfillment thereof, all citizens
may be required, under conditions provided by law, to render
personal, military or civil service.

ARTICLE III
BILL OF RIGHTS

Section 7. The right of the people to information on matters of
public concern shall be recognized. Access to official records,
and to documents and papers pertaining to official acts,
transactions, or decisions, as well as to government research
data used as basis for policy development, shall be afforded the
citizen, subject to such limitations as may be provided by law.

c). As electors- Sec. 4, Art. VII; Sec. 2, Art. XVI; Sec.
25, Art. XVIII
ARTICLE VII
EXECUTIVE DEPARTMENT
Section 4. The President and the Vice-President shall be
elected by direct vote of the people for a term of six years which
shall begin at noon on the thirtieth day of June next following the
day of the election and shall end at noon of the same date, six
years thereafter. The President shall not be eligible for any re-
election. No person who has succeeded as President and has
served as such for more than four years shall be qualified for
election to the same office at any time.
No Vice-President shall serve for more than two successive
terms. Voluntary renunciation of the office for any length of time
shall not be considered as an interruption in the continuity of the
service for the full term for which he was elected.
Unless otherwise provided by law, the regular election for
President and Vice-President shall be held on the second
Monday of May.
The returns of every election for President and Vice-President,
duly certified by the board of canvassers of each province or
city, shall be transmitted to the Congress, directed to the
President of the Senate. Upon receipt of the certificates of
canvass, the President of the Senate shall, not later than thirty
days after the day of the election, open all the certificates in the
presence of the Senate and the House of Representatives in
joint public session, and the Congress, upon determination of
the authenticity and due execution thereof in the manner
provided by law, canvass the votes
The person having the highest number of votes shall be
proclaimed elected, but in case two or more shall have an equal
and highest number of votes, one of them shall forthwith be
chosen by the vote of a majority of all the Members of both
Houses of the Congress, voting separately.
The Congress shall promulgate its rules for the canvassing of
the certificates.
The Supreme Court, sitting en banc, shall be the sole judge of
all contests relating to the election, returns, and qualifications of
the President or Vice-President, and may promulgate its rules
for the purpose.
ARTICLE XVI
GENERAL PROVISIONS
Section 2. The Congress may, by law, adopt a new name for the
country, a national anthem, or a national seal, which shall all be
truly reflective and symbolic of the ideals, history, and traditions
of the people. Such law shall take effect only upon its ratification
by the people in a national referendum.



ARTICLE XVIII
TRANSITORY PROVISIONS


Section 25. After the expiration in 1991 of the Agreement between
the Republic of the Philippines and the United States of
America concerning military bases, foreign military
bases, troops, or facilities shall not be allowed in the
Philippines except under a treaty duly concurred in by the
Senate and, when the Congress so requires, ratified by a
majority of the votes cast by the people in a national
referendum held for that purpose, and recognized as a
treaty by the other contracting State.

2) TERRITORY; ARTICLE I- National Territory
ARTICLE I
NATIONAL TERRITORY
The national territory comprises the Philippine archipelago, with all
the islands and waters embraced therein, and all other territories
over which the Philippines has sovereignty or jurisdiction,
consisting of its terrestrial, fluvial and aerial domains, including its
territorial sea, the seabed, the subsoil, the insular shelves, and
other submarine areas. The waters around, between, and
connecting the islands of the archipelago, regardless of their
breadth and dimensions, form part of the internal waters of the
Philippines.

a). What is the scope and how is the national territory defined
under the 1935, 1973, 1987 Constitution?

b). Reasons for constitutional definition of national territory
under the 1935, 1973 & 1987 constitution.

c). Changes in the definition of the national territory under 1987
Constitution. Effects?

Under the 1935 Constitution:

The Philippines comprises all territory ceded to the United States
by the Treaty of Paris concluded between the United States and
Spain on the tenth day of December, eighteenth hundred and
ninety-seven, the limit of which is set forth in Article II of said
treaty, together with all the islands in the treaty concluded at
Washington, between the United states and Spain on the seventh
day of November, nineteen hundred, and in the treaty concluded
between the United States and Great Britain on the second day of
January, nineteen hundred and thirty, and all territory over which
the present Government of the Philippine Islands exercises
jurisdiction.

The article gave four points of reference of the determination
of Philippine territory.

1. Treaty of Paris on December 10. 1898.
2. The Treaty of Washington on November 7, 1900
3. The treaty between the United States and Great Britain
on January 2, 1930
4. all territory over which the present government of the
Philippine Islands exercises jurisdiction.

Under the 1973 Constitution

The national territory comprises the Philippine archipelago,
with all the islands and waters embraced therein, and all other
territories belonging to the Philippines by historic right or legal
title, including the territorial sea, the air space, the subsoil, the
seabed, the insular shelves and the other submarine areas
over which the Philippines has sovereignty and jurisdiction.
The waters around, between and connecting the islands of
the archipelago, irrespective of their breadth and dimensions,
form part of the internal waters of the Philippines.

The national territory of the Philippines under the 1973
constitution is divided into three groups:

1. The Philippine archipelago
2. Other territories belonging to the Philippines
3. Philippine waters, air space and submarine areas

Thus, the Philippines under this constitution has a
horizontal reach consisting of lands and waters, upward reach
consisting of air space over the land and waters and a
downward reach consisting of submarine areas.

Reason for constitutional definition of the national
territory.

A territorial definition of the territory was necessary for the
preservation of our national wealth, for national security,
and as a manifestation of our solidarity as a people


1987 Constitution:
It merely removed language possibly offensive to an
ASEAN neighbor and achieved a more logical sequencing of the
elements that make up the territory but preserved everything
else found in the 1973 constitution.
The 1987 constitution retained the territorial concepts of
Philippine archipelago, internal waters, terrestrial, fluvial and
aerial domains. Removed concepts of acquisition of territories
by historic or legal rights, which provides conflicts to claims in
Sabah.



d). Archipelagic doctrine-Elements & Methods in fixing the
base line

What is the archipelagic doctrine or archipelago theory?

A principle brought up by the adoption of the straight
baseline method and internal waters forming the territory of the
archipelago.

It is the 2
nd
sentence of Section 1, Art. I of the Constitution
which states that the waters around, between and connecting
the islands of the archipelago, regardless of their breadth and
dimensions, form part of the internal waters of the Philippines.

Internal water (Archipelagic waters in the 1987 constitution by
virtue of the UNCLOS):

Consist of all parts of the sea landwards from the baseline
as well as inland rivers and lakes.

Consists of All waters around) Which are subject to the
territorial sovereignty of the coastal state. But are not subject to
the right of innocent passage by other states.

Territorial sea:
Consists of marginal belt of maritime waters adjacent to
the baselines extending 12 nautical miles outward. Outside
territorial seas are high seas.
Territorial seas are subject to rights of innocent passage.

According to the UNCLOS, the result of the 12 mile rule
implementation is an overlapping of adjacent states, thus it
provided rules that, implementation of equidistant rule or the
dividing line is a median line equidistant from the opposite
baselines. Not applied if in cases of historical or legal rights.
Other measurements will be required.

Insular Shelf:
Consists of a) seabed and subsoil of the submarine areas
adjacent to the coastal state but outside the territorial sea, to a
depth of 200 meters or beyond that limit, to where the depth
allow exploitation and b) the seabed and subsoil of areas
adjacent to islands.

The coastal state may then have the right to explore and
exploit minerals and resources therein, install installations and
safety zones. But the right does not affect the right of navigation
of other state.

Straight Baseline Method:
Instead of following the sinuosities of the coast, straight
lines are drawn connecting selected points on the coast without


appreciable departure from the general shape of the coast.(
Adopted in the Anglo-Norwegian Fisheries Case)

It was upheld under the RA 3046 amended by RA 5446,
defining the baseline territory of the territorial sea of the
Philippines.

Normal Baseline method:
Line drawn following the low water line along the coast as
marked on the large-scale charts officially recognized by the
Coastal state.

e). Read: RA 9522 defining the archiplegic baselines; PD
1596 - June 11, 1978- Annexing Kalayaan Islands as
municipality of Palawan; PD 1599 - June 11, 1978-
Declaring the 200- mile Economic Zone; UNCLOS

Conflict arising from UNCLOS vis--vis the Philippine
National territory: as discussed under UNCLOS and its
Implications on the Territorial Sovereignty of the Philippines
by Prof. Merlin Magallona:

Main Argument (is it favorable: NO!!!)

The UNCLOS raises fundamental questions, which bear
directly on the security and integrity of the Philippine State. The
impact of the UNCLOS on the Philippine situation is nothing short
of reorganizing the countrys entire territorial regime, with the
result that Philippine sovereignty becomes unrecognizable as it
established under its Constitutional system.

The Philippine Territory in accordance to its provision under the
Constitution.

The political boundaries of the Philippine territory was
defined under the provisions of the Treaty of Paris, thus, plotting
the Philippine territory under such treaty, there is a huge
rectangular shape surrounding the Philippines, having 600 miles
width and 12, 000 miles in length. Eastern boundaries of 300 miles
from the tip of Luzon and 150 miles west from the tip. RA 3046,
states that the political boundaries of the Philippine archipelago
might extend beyond the treaty limits defined under Article II of the
Treaty- subject to the rights of innocent passage.

The United States rejects such interpretation contending that
territorial boundaries should be defined under principles of
international law and not on treatises. Thus, they assert the 12
mile rule of territorial sovereignty of every state having entities of
international status.

The UNCLOS and its implication to the Philippine territory:

The Philippines will then be recognized as an Archipelagic
state and will adopt the straight baseline method.
1. The territorial sea of the Philippines will then be
measured up to 12 nautical miles without exceeding.
Within these territory, the Philippines can exercise
sovereignty subject to the right of innocent passage.
2. From the 12-mile territorial boundary, it can draw another
12 nautical miles wide to be its contiguous zone. This
area has only jurisdictional rights to the extent necessary
to enforce its custom, revenue, immigration or sanitary
law. The Philippines do not have full sovereignty of the
said area.
3. And the Philippines can then draw a line from its
baselines 200 nautical miles as an exclusive economic
zone where the state can explore and exploit minerals.
Beyond these zones are high seas. However, no state
can claim any exercise of sovereignty in this area.

Effects
According to the limits defined in the Treaty of Paris and of
the RA 3046, the total territory of the Philippines would have been
520,700 sq. mi., but the implementation of the UNCLOS, the
Philippine territory reduces into only 230, 000 sq. mi. or a
reduction of almost 50%.



Other effects to the Philippine territory by the UNCLOS

Archipelagic waters
Under the constitution of the Philippines, its internal or
national waters are defined as waters around, between and
connecting the island of the archipelago, regardless of its
breadth and dimensions, form part of the internal waters of the
Philippines and within the internal waters, the Philippines have
full territorial sovereignty as it was in its land territory.

Under the UNCLOS, the internal waters is changed into
Archipelagic waters and its sovereignty is watered down with
some conditions.

1. UNCLOS regulates the rights of innocent passage in
internal waters, such that all foreign ships including
warships will enjoy the rights of innocent passage and
is allowed to navigate through the Philippine waters.
2. the Philippine, within its Archipelagic waters will
recognize traditional fishing rights and activities of its
neighboring states.
3. the Philippines shall recognize and respect the existing
submarine cables laid by other states and passing
through its waters, including repairs and replacements.

Thus, what has been regarded under the constitution that
the Philippines enjoy full sovereignty over its internal waters are
radically reduced by the UNCLOS. It was subjected into more
restrictions that its territorial sea,.

1. Archipelagic Sea Lanes: the International Highways

The UNCLOS, provides Archipelagic sea lanes, in effect,
the Philippines are forced to provide sea lane passage and air
routes, suitable for the navigation of foreign ships and aircrafts.
These lanes are drawn across

There are two elements a) right of passage of all ships and b)
right of over flight of all aircraft. These passages, the Philippines
dont have the capacity to exercise its sovereign powers.


MAGALLONA vs. ERMITA, GR 187167, 8/6/11

FACTS OF THE CASE:
The antecedent facts of this case emerged upon the passing of
Republic Act 3046 in 1961. The laws purpose is to demarcate the
maritime baselines of the Philippines as it was deemed to be an archipelago. RA
3046 stood unchallenged until 2009, when Congress amended it and passed
RA 9522. This amending law shortened one baseline and determined new
base points of the archipelago. Moreso, it has identified the Kalayaan Island
Group and the Scarborough Shoal, as "regimes of islands", generating their own
maritime zones. The petitioners filed a case assailing the constitutionality of RA
9522. To their opinion, the law has effectively reduced the maritime territory of the
country. With this, Article I of the 1987 Constitution will be violated. The petitioners
also worried that that because of the suggested changes in the maritime
baselines will allow for foreign aircrafts and vessels to traverse the
Philippine territory freely. In effect, it steps on the states
sovereignty and national security. Meanwhile, the Congress insisted that in no
way will the amendments affect any pertinent power of the state. It also deferred
to agree that the law impliedly relinquishes the Philippines claims over Sabah.
Lastly, they have questioned the normative force of the notion that all the waters
within the rectangular boundaries in the Treaty of Paris. Now, because this treaty
still has undetermined controversies, the Congress believes that in the
perspective of international law, it did not see any binding obligation to honor it.
Thus, this case of prayer for writs of certiorari and prohibition is filed before the
court, assailing the constitutionality of RA 9522.

THE COURTS RULING:
The Court dismissed the case. It upheld the constitutionality of the law and made
it clear that it has merely demarcated the countrys maritime zones
and continental shelves in accordance to UNCLOS III.
Secondly, the Court found that the framework of the regime of islands
suggested by the law is not incongruent with the Philippines enjoyment of
territorial sovereignty over the areas of Kalayaan Group of Islands and the
Scarborough. Third, the court reiterated that the claims over Sabah remained
even with the adoption of the amendments. Further, the Court importantly
stressed that the baseline laws are mere mechanisms for the UNCLOS III to
precisely describe the delimitations. It serves as a notice to the international family


of states and it is in no way affecting or producing any effect like enlargement or
diminution of territories. With regard to the petitioners assertion that RA
9522 has converted the internal waters into archipelagic waters, the
Court did not appear to be persuaded. Instead, the Court suggested that the political
branches of Government can pass domestic laws that will aid in the competent
security measures and policies that will regulate innocent passage. Since the Court
emphasized innocent passage as a right based on customary law, it also believes
that no state can validly invoke sovereignty to deny a right acknowledged by modern
states. In the case of archipelagic states such as ours, UNCLOS III required the
imposition of innocent passage as a concession in lieu of their right to claim the entire
waters landward baseline. It also made it possible for archipelagic states to be
recognized as a cohesive entity under the UNCLOS III.


3). Government-defined

The agency or instrumentality through which the will of
the state is formulated, expressed, and realized. (US v. Dorr, 2
Phil. 332)

a). Govt. of the Philippines-defined
see- Sec. 2, Revised Adm. Code

A term refers to the corporate governmental entity through which
the functions of the governments are exercised throughout the
Philippine Islands, including, save as the contrary appears from
the context, the various arms through which political authority is
made effective in said Islands, whether pertaining to the central
government or to the provincial or municipal branches or other
form of local government.

b). Functions of government: constituent (governmental) and
Ministrant (proprietary) Bacani v. Nacoco, 100 Phil 468[1956]

Constituent:
Compulsory functions which constitute the very bonds of
society. As adopted in the Bacani vs. NACOCO Case by Pres.
Wilson:
1. The keeping of order and providing the protection of
persons and property from violence and robbery.
2. The fixing of legal relations between man and wife and
between parents and children.
3. The regulation of the holding, transmission and
interchange of property, and the determination of its
liabilities for debt or crimes.
4. The determination of contract rights between individuals.
5. The definition and punishment of crimes.
6. The administration of Justice in Civil Cases.
7. The determination of political duties, privileges and
relations of citizens.
8. Dealing of the sate with foreign powers. The preservation
of the state from external danger or encroachments and
the advancement of its international interest.

Ministrant Functions:

Optional functions of the government intended for achieving
a better life for the community.
1. For public welfare which private capital would not
naturally undertake.
2. Those things where the government is better equipped to
administer public welfare than those from a private
entities.

Note:
Such distinctions became obsolete because of the growing
complexities and increasing social challenges by which the
government is bound to satisfy. The ACCFA and CUGCO Case.

Bacani vs. National Coconut Corporation
100 Phil 468

Facts:
The National Coconut Corporation (respondent) requested
copies of stenographic notes from the stenographers of Branch VI
of Court of First Instance of Manila Bacani and Mateo (petitioners).
Upon delivery of the requested stenographic notes, the latter
(petitioners) charges the former (respondent) one-peso per page
for the 714-page notes.
Upon the inspection of the Auditor General of the
Corporation, he disallowed the payment and sought the
recovery of the amount paid contending that NACOCO is
exempt from such payment under the Rule 130 of the Rules of
Court, since the corporation is a government institution under
the Revised Administrative Code, performing a constituent
function.

Issue:
Does NACOCO a government entity under the Revised
Administrative Code and thus exempted from the payment of
the stenographic fees?

Held:
No. NACOCO is not part of government as defined by
the Revised Administrative Code. As provided by the same
Code, the Government of the Philippine Islands is a term which
refers to the corporate governmental entity through which the
functions of the government are exercised throughout the
Philippine Islands, including, save as the contrary appears from
the context, the various arms through which political authority is
made effective is said Islands, Whether pertaining to the central
Government of to the provincial or municipal branches or other
form of local government.
NACOCO was a corporation with personality distinct from
the government, therefore cannot avail exemptions from
payment. Thus it does not fir to the definition given even though
it performs some functions of the government.


Note:
Constituent functions of the government
(compulsory functions which constitutes the
very bonds of society). AS provided by US President
Wilson

The keeping of order and providing the protection of persons
and property from violence and robbery.

The fixing of the legal relations between man and wife and
between parents and children.

The regulation of the holding, transmission and interchange of
property, and the determination of its liabilities for debt or for
crime

The determination of contract rights between individuals.

The definition and punishment of crime.

The administration of justice in civil cases.

The determination of political duties, privileges and relations of
citizens.

Dealings of the state with foreign powers; the preservation of
the state from external danger or encroachment and the
advancement of its international interest.

Minisrant functions are optional functions of government
intended for achieving a better life for the community.

public welfare, public works, regulation of trade and industry


c). Distinction has become obsolete
ACCFA v, CUGCO, 30 SCRA 649 (agricultural credit);

Are the two-fold function of government as enumerated by the
Supreme Court in BACANI VS. NACOCO, 100 Phil. 468
(Ministrant [merely directory] and Constituent [Mandatory]
Functions) still applicable today?

No more as held in ACCFA VS. CUGCO, 30 SCRA 649. This
is due to complexities of the changing society, the two-fold
function of the government as classified by President Wilson is
no longer relevant as a result of the changing society wherein
what are considered merely ministrant functions of the State
before are now considered constituent , or vice versa.



NHC vs. JUCO, 134 SCRA 172, (1985) (housing);

Facts: Juco was an employee of the NHA. He filed a complaint for
illegal dismissal w/ MOLE but his case was dismissed by the labor
arbiter on the ground that the NHA is a govt-owned corp. and
jurisdiction over its employees is vested in the CSC. On appeal,
the NLRC reversed the decision and remanded the case to the
labor arbiter for further proceedings. NHA in turn appealed to the
SC

ISSUE: Are employees of the National Housing Corporation, a
GOCC without original charter, covered by the Labor Code or by
laws and regulations governing the civil service?

HELD: Sec. 11, Art XII-B of the Constitution specifically provides:
"The Civil Service embraces every branch, agency, subdivision
and instrumentality of the Government, including every
government owned and controlled corporation.

The inclusion of GOCC within the embrace of the civil service
shows a deliberate effort at the framers to plug an earlier loophole
which allowed GOCC to avoid the full consequences of the civil
service system. All offices and firms of the government are
covered.
This consti provision has been implemented by statute PD 807 is
unequivocal that personnel of GOCC belong to the civil service
and subject to civil service requirements.
"Every" means each one of a group, without exception. This case
refers to a GOCC. It does not cover cases involving private firms
taken over by the government in foreclosure or similar
proceedings.

xxx
For purposes of coverage in the Civil Service, employees of govt-
owned or controlled corps. whether created by special law or
formed as subsidiaries are covered by the Civil Service Law, not
the Labor Code, and the fact that pvt. corps. owned or controlled
by the govt may be created by special charter does not mean that
such corps. not created by special law are not covered by the Civil
Service.
xxx
The infirmity of the resp's position lies in its permitting the
circumvention or emasculation of Sec. 1, Art. XII-B [now Art IX, B,
Sec. 2 (1)] of the Consti. It would be possible for a regular ministry
of govt to create a host of subsidiary corps. under the Corp. Code
funded by a willing legislature. A govt-owned corp. could create
several subsidiary corps. These subsidiary corps. would enjoy the
best of two worlds. Their officials and employees would be
privileged individuals, free from the strict accountability required by
the Civil Service Dec. and the regulations of the COA. Their
incomes would not be subject to the competitive restraint in the
open market nor to the terms and conditions of civil service
employment. Conceivably, all govt-owned or controlled corps.
could be created, no longer by special charters, but through
incorp. under the general law. The Constitutional amendment
including such corps. in the embrace of the civil service would
cease to have application. Certainly, such a situation cannot be
allowed For more case digests and law school notes visit
lizajamarga.com.


(Fontanilla v. Maliaman, 2/27/91-GR 55963 (water irrigation).
FACTS: National Irrigation Administration (NIA), a government
agency, was held liable for damages resulting to the death of the
son of herein petitioner spouses caused by the fault and/or
negligence of the driver of the said agency. NIA maintains that it is
not liable for the act of its driver because the former does not
perform primarily proprietorship functions but governmental
functions.
ISSUE: Whether or not NIA may be held liable for damages
caused by its driver.
HELD: Yes. NIA is a government agency with a corporate
personality separate and distinct from the government, because its
community services are only incidental functions to the principal
aim which is irrigation of lands, thus, making it an agency with
proprietary functions governed by Corporation Law and is liable
for actions of their employees.

PEA V. Yujuico, 351 SCRA 280[2001] (roads)

FACTS: The present petition for review on certiorari is an
offshoot of this Courts final and executory decision in Public
Estates Authority (PEA) v. Jesus S. Yujuico and Augusto Y.
Carpio (2001 PEA Case)which settled the issue on overlapping
parcels of land between petitioner on one hand, and Jesus S.
Yujuico (Yujuico) and Augusto Y. Carpio (Carpio) on the other,
by upholding the Compromise Agreement executed by the
parties.In the 2001 PEA Case, the Court affirmed the dismissal
of PEAs petition for relief from judgment questioning the
Compromise Agreement approved by Branch 258 of the
Regional Trial Court of Paraaque City, ruling that the petition
was filed beyond the 60-day period allowed by Sec. 3, Rule 38
of the Rules of Court; and that it would not be right to allow a
mere change of PEAs management to defeat the operation of
the rules on reglementary period. The crux of the present
controversy is the implementation of the Compromise
Agreement which provides that, among other things:

c. The SECOND PARTY is also given the OPTION TO
PURCHASE an additional 7.6 hectares of land and CBP
1-A. The land subject of the OPTION shall be located and
identified in the area to be agreed upon by the parties
under a separate arrangement.

i. The OPTION must be exercised within a
period of three (3) years from the date this
Compromise Agreement has been approved by
the Court and the Compromise Judgment has
been issued and become final.
ii. The value of the land subject of the OPTION
shall be based on the fair market value as
determined by PEA on the date of the exercise
of the OPTION.
iii. The OPTION shall be exercisable in
increments of 5,000 square meters.
iv. In the event that the SECOND PARTY would
develop the property at CBP-1A subject of their
option, through a joint venture agreement or other
business arrangements, the FIRST PARTY shall
have the right of first refusal to develop the same.
v. Within the option period, if the FIRST PARTY
will have an offer to purchase or develop the
property, the SECOND PARTY shall be notified by
PEA and shall be required to match the offer. If the
SECOND PARTY cannot match the offer, the PEA
shall be free to sell or award the development to
the offeror. (emphasis and underscoring supplied)
On January 26, 1999, respondents informed petitioner of their
intention to exercise the option to purchase.
By Omnibus Motion of June 6, 2002, Yujuico and Carpio,
assisted by Benedicto V. Yujuico (Benedicto) acting as their
attorney-in-fact, moved that the trial court issue an Order for,
among other things, the appointment of three licensed real
estate appraisers who shall submit a report on the fair market
value of the subject property on the date of the exercise of the
option to purchase stipulated in the Compromise Agreement;
and the suspension of the three-year option period until the trial
courts approval of the appraisers report.
By letter of March 26, 2004,however, petitioner set the terms
and conditions for respondents exercise of the option to
purchase.
Respondents did not heed petitioners imposition of a 122-day
period to exercise the option to purchase. Instead, they filed
with the trial court a Supplemental Omnibus Motion praying for
an Order directing, among other things:


(B) PEA and BENEDICTO V. YUJUICO, Attorney-in-Fact of
[herein respondents], (aa) to implement the OPTION TO
PURCHASE the 7.6 Has. to be taken from PEA-CBP-IA with
specific boundaries delineated by the parties as shown in ANNEX
B hereof; (bb) to consider the actual condition of said 7.6 Has.
and the prevailing real estate market on or about January 26,
1999, the date of the exercise of the OPTION, which was
reiterated in subsequent letters to PEA in determining the fair, just
and bona fide market price of said 7.6 Has. by the PEA; and (cc)
which exercise of the OPTION is well within the 3-year period from
the date the Court-approved Compromise Agreement became
final as provided in Par. (c)(i) of the said Compromise Agreement.
In its Comment to the Motion,

petitioner contended that the
determination of the fair market value of the property subject of the
option to purchase had been lodged in it by the Compromise
Agreement; and that the period for the exercise of the option had
expired, respondents not having exercised the same within three
years from the date the compromise judgment became final.
By Order of January 11, 2005,the trial court denied respondents
Supplemental Omnibus Motion, holding that, among other things,
it is petitioner which has the exclusive right to determine the fair
market value of the land that respondents want to purchase
pursuant to the Compromise Agreement.
Their motion for reconsideration having been denied by Order
dated June 7, 2005,

respondents appealed to the Court of Appeals
via certiorari.
By Decision of August 31, 2007,

the appellate court granted
respondents petition. It held that, among other things, the
Compromise Agreement stipulated that the price to be determined
by petitioner cannot be any price conceived by whim, but must be
the fair market value, which has acquired a definite meaning in
the world of business; and that it must accordingly determine the
fair market value of the property instead of remanding the case to
the trial court in order to put an end to the litigation.
The appellate court went on to set the fair market value at
P13,000 per square meter at the time the option to purchase was
sought to be exercised, finding that the property was then still raw
land and not a ready-to-build site.
Petitioners motion for reconsideration having been denied by
Resolution dated February 20, 2008, it filed the present Petition for
Review on Certiorari.
Petitioner reiterates its position before the trial court, adding that
the appellate court has no authority to impose upon the parties a
judgment different from the terms of their Compromise Agreement;
and that the appellate court erroneously adopted the valuation of
the appraiser, Royal Asia Corporation, which was hired and paid
by respondents.
Respondents, on the other hand, argue that, among other things,
the question of proper valuation raised by petitioner is one of fact,
and thus prohibited in a petition for review; that the appellate court
correctly applied the Compromise Agreement according to its
intent; and that assuming that the Compromise Agreement gives
petitioner the sole authority to determine the price of the property,
such stipulation is void for being purely potestative and violative of
the principle of mutuality of contracts.
RULING: The petition must fail.
The present case turns on the pivot of the option to purchase
provided in the Compromise Agreement which, having been
judicially affirmed, constitutes res judicata upon the parties.
A compromise agreement intended to resolve a matter already
under litigation is a judicial compromise. Having judicial mandate
and entered as its determination of the controversy, such judicial
compromise has the force and effect of a judgment. It transcends
its identity as a mere contract between the parties, as it becomes
a judgment that is subject to execution in accordance with the
Rules of Court. Thus, a compromise agreement that has been
made and duly approved by the court attains the effect and
authority of res judicata, although no execution may be issued
unless the agreement receives the approval of the court where the
litigation is pending and compliance with the terms of the
agreement is decreed.
To simply say that, by the earlier-quoted term of the
Compromise Agreement respecting petitioners evaluation of the
land subject of the option to purchase on the basis of its fair
market value on the date of the exercise of the option, petitioner
has the exclusive prerogative to determine the purchase price of
the subject land is a very myopic interpretation.
The proper interpretation of the stipulation is that petitioner is
given the right to determine the price of the subject land,
provided it can substantiate that the same is its
fairmarketvalueasofthedateof theexerciseoftheoption. The
term fair market value in the stipulation cannot be ignored
without running afoul of the intent of the parties. It not being
disputed that respondents exercised the option to purchase on
January 26, 1999, the valuation should thus be based on the fair
market value of the property on the said date.
Indeed, as the appellate court held, in order to write finis to the
case, the fair market value of the property must be determined
on the basis of the existing records, instead of still remanding
the case to the trial court.
Fair market value has acquired a settled meaning in law and
jurisprudence. It is the price at which a property may be sold by
a seller who is not compelled to sell and bought by a buyer who
is not compelled to buy, taking into consideration all uses to
which the property is adapted and might in reason be applied.
The criterion established by the statute contemplates a
hypothetical sale.
Given this yardstick, the Court found no cogent reason to
disturb the factual finding of the appellate court that the proper
valuation of the property is P13,000 per square meter as of
January 26, 1999. As it correctly explained, the value was
arrived at through the market data approach, which is based on
sales and listings of comparable property registered within the
vicinity; and that the property was classified as raw land
because there were yet no houses and facilities like electricity,
water and others at the time of the exercise of the option.
The rule is well-established that if there is no showing of error in
the appreciation of facts by the appellate court as in the present
case, the Court treats it as conclusive.
A word on petitioner. Its bad faith is abundantly clear. It did not
respond to respondents notification of their intention to exercise
the stipulated option to purchase until after more than four
years, or on March 26, 2004, when it surprised respondents with
an exorbitant price for the property and gave them only 122
days within which to purchase the same. Undeniably, it is
enfeebling the Compromise Agreement under the guise of
enforcing it, which the Court will not sanction.

SHIPSIDE, INC. vs. CA, GR 143377 2/20/01- (special
economic zone)

Facts: The petitioner filed a certiorari with the CA containing the
requisite certification on non-forum shopping but failed to attach
proof that the person signing the certification was authorized to
do so. The CA dismissed the petition. The petitioner submits a
motion for reconsideration which attached a secretarys
certificate attesting to the signatorys authority to sign
certificates against forum shopping on behalf of the petitioner.
When the court of CA denied the motion, the petitioner sought
relief with the SC.


Issue: Whether the CA erred in dismissing the petition of
Shipside Inc.

Ruling: Yes, the CA erred in the dismissal of the petition. The
SC revised the decision of CA recognizing the belated filing of
the certifications against forum shopping as permitted in
exceptional circumstances. It further held that with more reason
should a petition be given due course when this incorporates a
certification on non-forum shopping without evidence that the


person signing the certifications was an authorized signatory and
the petitioner subsequently submits a secretarys certificate
attesting to the signatorys authority in its motion for consideration.
The court allows belated submission of certifications showing
proof of the signatorys authority in signing the certification of
forum shopping.

d). De Jure and De Facto governments
Co Kim Chan vs. Valdez, 75 PHIL 113

As held in CO KIM CHAM VS. VALDEZ TAN KEH, 75 Phil. 113,
the three (3) kinds of de facto governments are:

The first, or government de facto in a proper legal sense, is
that government that gets possession and control of, or usurps, by
force or by the voice of the majority, the rightful legal governments
and maintains itself against the will of the latter, such as the
government of England under the Commonwealth, first by
Parliament and later by Cromwell as Protector.

The second is that which is established and maintained by
military forces who invade and occupy a territory of the enemy in
the course of war, and which is denominated a government of
paramount force, as the cases of Castine, in Maine, which was
reduced to British possession in the war of 1812, and Tampico,
Mexico, occupied during the war with Mexico, by the troops of the
United States.

And the third is that established as an independent
government by the inhabitants of a country who rise in insurrection
against the parent state of such as the government of the
Southern Confederacy in revolt not concerned in the present case
with the first kind, but only with the second and third kinds of de
facto governments.

"But there is another description of government, called also
by publicists a government de facto, but which might, perhaps, be
more aptly denominated a government of paramount force. Its
distinguishing characteristics are

(1), that its existence is maintained by active military power with
the territories, and against the rightful authority of an established
and lawful government; and
(2), that while it exists it necessarily be obeyed in civil matters by
private citizens who, by acts of obedience rendered in submission
to such force, do not become responsible, or wrongdoers, for
those acts, though not warranted by the laws of the rightful
government.

e). What is the nature of the government under the Freedom
Constitution-Letter of Justice REYNATO PUNO, 210 SCRA
589,598[1992];

Facts: The petitioner Justice of the Court of Appeals, Reynato
Puno wrote a letter to the SC seeking for correction of his seniority
ranking in The Court of Appeals.

After a reorganization of the Judiciary after the 1986 EDSA
Revolution, by virtue of the BP 129 as amended by the EO 33, the
petitioner was appointed by the Screening Committee assigned to
reorganize the Judiciary in to rank eleven (11) as Associate
Justice of the CA. After it was signed by Pres. Aquino during the
Freedom Constitution, the petitioners seniority rank changed and
fell into rank 26.

The petitioner now argues that the President violated the
provisions of the EO 33 and that she complied with his own EO 33
to reorganize the Judiciary, by appointing a different set of ranks.

The petitioner also argued that the Freedom Constitution
proclaimed by the president still recognize the ungratified 1973
constitution and that all the rights under the latter should still be
present under the Freedom Constitution, and thus the petitioner
ranking should not fall but changed into rank 5.


Issue: Does the President under the then enforced Freedom
Constitution has the power to modify any seniority ranking in the
CA, in this case provision of EO 33 amending BP 129?
Held:Yes. Under the Freedom Constitution, the government of
the President is revolutionary in nature such that it can exercise
the powers of the Executive and Legislative. Thus, it can modify
even her own EO 33 such that revolutionary powers can
disregard any precedence or seniority rank.
Thus, the President was free to appoint to the new Court
of Appeals in the order of precedence she wanted as part of his
revolutionary powers.

Lawyers League for a Better Philippines v. Aquino, 5/22/86

supra.

f). What is the legal distinction between EDSA PEOPLE
POWER I and EDSA PEOPLE POWER II? Estrada v.
Arroyo GR. No. 146738. March 2, 2001)

FACTS: Petitioner sought to enjoin the respondent
Ombudsman from conducting any further proceedings in any
criminal complaint that may be filed in his office, until after the
term of petitioner as President is over and only if legally
warranted. Erap also filed a Quo Warranto case, praying for
judgment confirming petitioner to be the lawful and incumbent
President of the Republic of the Philippines temporarily unable
to discharge the duties of his office, and declaring respondent to
have taken her oath as and to be holding the Office of the
President, only in an acting capacity pursuant to the provisions
of the Constitution.
HELD:
FIRST: The cases at bar pose legal and not political questions.
The principal issues for resolution require the proper
interpretation of certain provisions in the 1987 Constitution,
notably section 1 of Article II, and section 8 of Article VII, and
the allocation of governmental powers under section II of Article
VII. The issues likewise call for a ruling on the scope of
presidential immunity from suit. They also involve the correct
calibration of the right of petitioner against prejudicial publicity.
As early as the 1803 case of Marbury v. Madison, the doctrine
has been laid down that it is emphatically the province and duty
of the judicial department to say what the law is . . .
The Court also distinguished between EDSA People Power I
and EDSA People Power II. EDSA I involves the exercise of the
people power of revolution which overthrew the whole
government. EDSA II is an exercise of people power of
freedom of speech and freedom of assembly to petition the
government for redress of grievances which only affected the
office of the President. EDSA I is extra constitutional and the
legitimacy of the new government that resulted from it cannot be
the subject of judicial review, but EDSA II is intra constitutional
and the resignation of the sitting President that it caused and
the succession of the Vice President as President are subject to
judicial review. EDSA I presented political question; EDSA II
involves legal questions.
SECOND: Using the totality test, the SC held that petitioner
resigned as President.
a. The proposal for a snap election for president in May
where he would not be a candidate is an indicium that petitioner
had intended to give up the presidency even at that time.
b. The Angara diary shows that the President wanted only
five-day period promised by Reyes, as well as to open the
second envelop to clear his name.
If the envelope is opened, on Monday, he says, he will leave by
Monday.
The President says. Pagod na pagod na ako. Ayoko na
masyado nang masakit. Pagod na ako sa red tape,
bureaucracy, intriga. (I am very tired. I dont want any more of
this its too painful. Im tired of the red tape, the bureaucracy,
the intrigue.)
I just want to clear my name, then I will go.


The SC held that this is high grade evidence that the petitioner
has resigned. The intent to resign is clear when he said x x x
Ayoko na masyado nang masakit. Ayoko na are words of
resignation.
c. During the negotiations, the resignation of the petitioner
was treated as a given fact. The only unsettled points at that time
were the measures to be undertaken by the parties during and
after transition period.
d. His resignation was also confirmed by his leaving
Malacaang. In the press release containing his final statement,
(1) he acknowledged the oath-taking of the respondent as
President of the Republic albeit with the reservation about its
legality; (2) he emphasized he was leaving the Palace, the seat of
the presidency, for the sake of peace and in order to begin the
healing process of our nation. He did not say he was leaving the
Palace due to any kind of inability and he was going to re-assume
the presidency as soon as the disability disappears; (3) he
expressed his gratitude to the people for the opportunity to serve
them. Without doubt, he was referring to the past opportunity
given him to serve the people as President; (4) he assured that he
will not shirk from any future challenge that may come ahead in
the same service of our country. Petitioners reference is to a
future challenge after occupying the office of the president which
he has given up; and (5) he called on his supporters to join him in
the promotion of a constructive national spirit of reconciliation and
solidarity. Certainly, the national spirit of reconciliation and
solidarity could not be attained if he did not give up the
presidency. The press release was petitioners valedictory, his
final act of farewell. His presidency is now in the past tense.
THIRD: The petitioner is permanently unable to act as President.
Section 11 of Article VII provides that Congress has the ultimate
authority under the Constitution to determine whether the
President is incapable of performing his functions. Both houses
of Congress have recognized respondent Arroyo as the President.
The House of Representative passed on January 24, 2001 House
Resolution No. l75 which states: RESOLUTION EXPRESSING
THE SUPPORT OF THE HOUSE OF REPRESENTATIVES TO
THE ASSUMPTION INTO OFFICE BY VICE PRESIDENT
GLORIA MACAPAGAL-ARROYO AS PRESIDENT OFTHE
REPUBLIC OF THE PHILIPPINES, EXTENDING ITS
CONGRATULATIONS AND EXPRESSING ITS SUPPORT FOR
HER ADMINISTRATION AS A PARTNER IN THE ATTAINMENT
OF THE NATIONS GOALS UNDER THE CONSTITUTION. The
Senate also passed Senate Resolution No. 82 which states:
RESOLUTION CONFIRMING PRESIDENT GLORIA
MACAPAGAL-ARROYOS NOMINATION OF SEN. TEOFISTO T.
GUINGONA, JR. AS VICE PRESIDENT OF THE REPUBLIC OF
THE PHILIPPINES
Implicitly clear in that recognition is the premise that the inability of
petitioner Estrada is no longer temporary. Congress has clearly
rejected petitioners claim of inability. Even if petitioner can prove
that he did not resign, still, he cannot successfully claim that he is
a President on leave on the ground that he is merely unable to
govern temporarily. That claim has been laid to rest by Congress
and the decision that respondent Arroyo is the de jure President
made by a co-equal branch of government cannot be reviewed by
the Supreme Court.
FOURTH: The petitioner does not enjoy immunity from suit.
The Supreme Court rejected petitioners argument that he cannot
be prosecuted for the reason that he must first be convicted in the
impeachment proceedings. The impeachment trial of petitioner
Estrada was aborted by the walkout of the prosecutors and by the
events that led to his loss of the presidency. On February 7, 2001,
the Senate passed Senate Resolution No. 83 Recognizing that
the Impeachment Court is Functus Officio. Since the
Impeachment Court is now functus officio, it is untenable for
petitioner to demand that he should first be impeached and then
convicted before he can be prosecuted. The plea, if granted,
would put a perpetual bar against his prosecution. The debates in
the Constitutional Commission make it clear that when
impeachment proceedings have become moot due to the
resignation of the President, the proper criminal and civil cases
may already be filed against him.
The SC also ruled in In re: Saturnino Bermudez that incumbent
Presidents are immune from suit or from being brought to court
during the period of their incumbency and tenure but not
beyond. Considering the peculiar circumstance that the
impeachment process against the petitioner has been aborted
and thereafter he lost the presidency, petitioner cannot demand
as a condition sine qua non to his criminal prosecution before
the Ombudsman that he be convicted in the impeachment
proceedings.
Also, petitioner cannot cite any decision of the SC licensing the
President to commit criminal acts and wrapping him with post-
tenure immunity from liability. The rule is that unlawful acts of
public officials are not acts of the State and the officer who acts
illegally is not acting as such but stands in the same footing as
any other trespasser.
FIFTH: Petitioner was not denied the right to impartial trial.
Pervasive publicity is not per se prejudicial to the right of an
accused to fair trial. The mere fact that the trial of appellant was
given a day-to-day, gavel-to-gavel coverage does not by itself
prove that the publicity so permeated the mind of the trial judge
and impaired his impartiality. In the case at bar, the records do
not show that the trial judge developed actual bias against
appellant as a consequence of the extensive media coverage of
the pre-trial and trial of his case. The totality of circumstances
of the case does not prove that the trial judge acquired a fixed
opinion as a result of prejudicial publicity which is incapable if
change even by evidence presented during the trial. Appellant
has the burden to prove this actual bias and he has not
discharged the burden.

g). Forms of Government- Direct democracy, Assembly,
Parliamentary, Presidential

PRESIDENTIAL: The Philippines is a republic with a
presidential form of government wherein power is equally
divided among its three branches: executive, legislative, and
judicial.
One basic corollary in a presidential system of government is
the principle of separation of powers wherein legislation belongs
to Congress, execution to the Executive, and settlement of legal
controversies to the Judiciary.

DIRECT DEMOCRACY: (also known as pure democracy) is a
form of democracy in which people decide (e.g. vote on, form
consensus on, etc.) policy initiatives directly, as opposed to a
representative democracy in which people vote for
representatives who then decide policy initiatives.

ASSEMBLY: lower house of the bicameral Parliament, typically
60 members, the one that decides and make laws

PARLIAMENTARY: a system of democratic governance of a
state in which the executive branch derives its democratic
legitimacy from, and is held accountable to, the legislature
(parliament); the executive and legislative branches are thus
interconnected. In a parliamentary system, the head of state is
normally a different person from the head of government. This is
in contrast to a presidential system in a democracy, where the
head of state often is also the head of government, and most
importantly: the executive branch does not derive its democratic
legitimacy from the legislature.

h). What form of government did we have under the 1973
Constitution as revised under 1981 amendments? Free
Telephone Workers Union v. Minister, 108 SCRA 757, 763-
5[1981]





The government of the Philippines under the 1973
Constitution is essentially presidential with parliamentary
features.

Facts: On September 14, 1981, there was a notice of strike with
the Ministry of Labor for unfair labor practices stating the following
grounds: "1) Unilateral and arbitrary implementation of a Code of
Conduct; 2) Illegal terminations and suspensions of our officers
and members as a result of the implementation of said Code of
Conduct; and 3) Unconfirmation of call sick leaves and its
automatic treatment as Absence Without Official Leave of
Absence (AWOL) with corresponding suspensions, in violation of
our Collective Bargaining Agreement." After which came, on
September 15, 1981, the notification to the Ministry that there was
compliance with the two-thirds strike vote and other formal
requirements of the law and Implementing Rules. Several
conciliation meetings called by the Ministry followed, with the
petitioner manifesting its willingness to have a revised Code of
Conduct that would be fair to all concerned but with a plea that in
the meanwhile the Code of Conduct being imposed be
suspended-a position that failed to meet the approval of private
respondent. Subsequently, on September 25, 1981, the
respondent certified the labor dispute to the National Labor
Relations Commission for compulsory arbitration and enjoined any
strike at the private respondent's establishment. The labor dispute
was set for hearing by respondent National Labor Relations
Commission on September 28, 1981. There was in the main an
admission of the above relevant facts by public respondents.
Private respondent, following the lead of petitioner labor union,
explained its side on the controversy regarding the Code of
Conduct, the provisions of which as alleged in the petition were
quite harsh, resulting in what it deemed indefinite preventive
suspension-apparently the principal cause of the labor dispute.
The very next day after the filing of the petition, this Court issued
the following resolution: "Considering the allegations contained,
the issues raised and the arguments adduced in the petition for
Certiorari with prayer for a restraining order, the Court Resolved to
(a) require the respondents to file an [answer], not a motion to
dismiss, on or before Wednesday, October 7, 1981; and (b) [Set]
this case for hearing on Thursday, October 8, 1981 at 11:00
o'clock in the morning." After the parties were duly heard, Solicitor
General Estelito P. Mendoza appearing for the public
respondents, the case was considered ripe for decision.

Issue: Whether or not there is undue delegation to the MOLE.

Held: No. The Delegation to the MOLE of the power to assume
jurisdiction in the labor dispute was likely to affect the national
interest or to certify the same to the NLRC for arbitration does not
constitute an undue delegation of legislative powers.

The allegation that there is undue delegation of legislative powers
cannot stand the test of scrutiny. The power which he would deny
the Minister of Labor by virtue of such principle is for petitioner
labor union within the competence of the President, who in its
opinion can best determine national interests, but only when a
strike is in progress. Such admission is qualified by the
assumption that the President "can make law," an assertion which
need not be passed upon in this petition. What possesses
significance for the purpose of this litigation is that it is the
President who "shall have control of the ministries." It may
happen, therefore, that a single person may occupy a dual
position of Minister and Assemblyman. To the extent, however,
that what is involved is the execution or enforcement of legislation,
the Minister is an official of the executive branch of the
government. The adoption of certain aspects of a parliamentary
system in the amended Constitution does not alter its essentially
presidential character. Article VII on the presidency starts with this
provision: "The President shall be the head of state and chief
executive of the Republic of the Philippines." Its last section is an
even more emphatic affirmation that it is a presidential system that
obtains in our government.

i). Administration distinguished from government

Government is distinct from administration. Government is the
agency which formulates, expresses and realizes the will of the
people. On the other hand, administration is composed of group of
persons in whose hands the reins of government are for the time
being. It is the administration that runs the affairs of the
government for a given period of time, after which another
administration may be called upon by the people to serve them.
That is why we say that administration changes but the
government does not.

4). SOVEREIGNTY

a). Legal and political

Legal: the power to issue final commnds

Political: the sum total of all the influences which lie
behind the law

b). Characteristics: permanent, exclusive, comprehensive,
absolute, indivisible, inalienable, and imprescriptible


THE PROVINCE OF NORTH COTABATO vs. THE
GOVERNMENT OF THE REPUBLIC OF THE
PHILIPPINES PEACE PANEL ON ANCESTRAL
DOMAIN (GRP), et al., G.R. No. 183591, October 14,
2008

On August 5, 2008, the Government of the Republic of the
Philippines (GRP) and the MILF, through the Chairpersons of
their respective peace negotiating panels, were scheduled to
sign a Memorandum of Agreement on the Ancestral Domain
(MOA-AD) Aspect of the GRP-MILF Tripoli Agreement on
Peace of 2001 in Kuala Lumpur, Malaysia.

The MILF is a rebel group which was established in March
1984 when, under the leadership of the late Salamat Hashim, it
splintered from the Moro National Liberation Front (MNLF) then
headed by Nur Misuari, on the ground, among others, of what
Salamat perceived to be the manipulation of the MNLF away
from an Islamic basis towards Marxist-Maoist orientations.

The signing of the MOA-AD between the GRP and the
MILF was not to materialize, however, for upon motion of
petitioners, specifically those who filed their cases before the
scheduled signing of the MOA-AD, this Court issued a
Temporary Restraining Order enjoining the GRP from signing
the same.

The MOA-AD was preceded by a long process of
negotiation and the concluding of several prior agreements
between the two parties beginning in 1996, when the GRP-MILF
peace negotiations began. On July 18, 1997, the GRP and
MILF Peace Panels signed the Agreement on General
Cessation of Hostilities. The following year, they signed the
General Framework of Agreement of Intent on August 27, 1998.

On July 23, 2008, the Province of North Cotabato

and
Vice-Governor Emmanuel Piol filed a petition, docketed as
G.R. No. 183591, for Mandamus and Prohibition with Prayer for
the Issuance of Writ of Preliminary Injunction and Temporary
Restraining Order
.
Invoking the right to information on matters
of public concern, petitioners seek to compel respondents to
disclose and furnish them the complete and official copies of the
MOA-AD including its attachments, and to prohibit the slated
signing of the MOA-AD, pending the disclosure of the contents
of the MOA-AD and the holding of a public consultation thereon.
Supplementarily, petitioners pray that the MOA-AD be declared
unconstitutional .

ISSUES:

1. Whether there is a violation of the peoples
right to information on matters of public
concern (1987 Constitution, Article III,
Sec. 7) under a state policy of full
disclosure of all its transactions involving
public interest (1987 Constitution, Article
II, Sec. 28) including public consultation
under Republic Act No. 7160 (LOCAL
GOVERNMENT CODE OF 1991)



If it is in the affirmative, whether prohibition
under Rule 65 of the 1997 Rules of Civil
Procedure is an appropriate remedy;

2. Whether by signing the MOA, the
Government of the Republic of the Philippines
would be BINDING itself

a) to create and recognize the
Bangsamoro Juridical Entity (BJE) as
a separate state, or a juridical,
territorial or political subdivision not
recognized by law;

b) to revise or amend the Constitution
and existing laws to conform to the
MOA;

c) to concede to or recognize the claim
of the Moro Islamic Liberation Front
for ancestral domain in violation of
Republic Act No. 8371 (THE
INDIGENOUS PEOPLES RIGHTS
ACT OF 1997), particularly Section
3(g) & Chapter VII (DELINEATION,
RECOGNITION OF ANCESTRAL
DOMAINS)

If in the affirmative, whether the Executive
Branch has the authority to so bind the
Government of the Republic of the
Philippines;

3. Whether the inclusion/exclusion of the
Province of North Cotabato, Cities of
Zamboanga, Iligan and Isabela, and the
Municipality of Linamon, Lanao del Norte
in/from the areas covered by the projected
Bangsamoro Homeland is a justiciable
question; and

HELD:

As per MO-AD, the territory of the Bangsamoro homeland is
described as the land mass as well as the maritime, terrestrial,
fluvial and alluvial domains, including the aerial domain and the
atmospheric space above it, embracing the Mindanao-Sulu-
Palawan geographic region.



More specifically, the core of the BJE is defined as the
present geographic area of the ARMM thus constituting the
following areas: Lanao del Sur, Maguindanao, Sulu, Tawi-Tawi,
Basilan, and Marawi City. Significantly, this core also includes
certain municipalities of Lanao del Norte that voted for inclusion in
the ARMM in the 2001 plebiscite .

Outside of this core, the BJE is to cover other provinces,
cities, municipalities and barangays, which are grouped into two
categories, Category A and Category B. Each of these areas is to
be subjected to a plebiscite to be held on different dates, years
apart from each other. Thus, Category A areas are to be
subjected to a plebiscite not later than twelve (12) months
following the signing of the MOA-AD . Category B areas, also
called Special Intervention Areas, on the other hand, are to be
subjected to a plebiscite twenty-five (25) years from the signing of
a separate agreement the Comprehensive Compact .

The Parties to the MOA-AD stipulate that the BJE shall have
jurisdiction over all natural resources within its internal waters,
defined as extending fifteen (15) kilometers from the coastline of
the BJE area ; that the BJE shall also have territorial waters,
which shall stretch beyond the BJE internal waters up to the
baselines of the Republic of the Philippines (RP) south east and
south west of mainland Mindanao; and that within these territorial
waters, the BJE and the Central Government (used
interchangeably with RP) shall exercise joint jurisdiction, authority
and management over all natural resources . Notably, the
jurisdiction over the internal waters is not similarly described as
joint.

The MOA-AD further provides for the sharing of minerals
on the territorial waters between the Central Government and
the BJE, in favor of the latter, through production sharing and
economic cooperation agreement . The activities which the
Parties are allowed to conduct on the territorial waters are
enumerated, among which are the exploration and utilization of
natural resources, regulation of shipping and fishing activities,
and the enforcement of police and safety measures . There is
no similar provision on the sharing of minerals and allowed
activities with respect to the internal waters of the BJE.

The MOA-AD states that the BJE is free to enter into any
economic cooperation and trade relations with foreign countries
and shall have the option to establish trade missions in those
countries. Such relationships and understandings, however, are
not to include aggression against the GRP. The BJE may also
enter into environmental cooperation agreements .

The external defense of the BJE is to remain the duty and
obligation of the Central Government. The Central Government
is also bound to take necessary steps to ensure the BJEs
participation in international meetings and events like those of
the ASEAN and the specialized agencies of the UN. The BJE is
to be entitled to participate in Philippine official missions and
delegations for the negotiation of border agreements or
protocols for environmental protection and equitable sharing of
incomes and revenues involving the bodies of water adjacent to
or between the islands forming part of the ancestral domain .

With regard to the right of exploring for, producing, and
obtaining all potential sources of energy, petroleum, fossil fuel,
mineral oil and natural gas, the jurisdiction and control thereon
is to be vested in the BJE as the party having control within its
territorial jurisdiction. This right carries the proviso that, in
times of national emergency, when public interest so requires,
the Central Government may, for a fixed period and under
reasonable terms as may be agreed upon by both Parties,
assume or direct the operation of such resources .

The sharing between the Central Government and the BJE
of total production pertaining to natural resources is to be 75:25
in favor of the BJE .

The MOA-AD provides that legitimate grievances of the
Bangsamoro people arising from any unjust dispossession of
their territorial and proprietary rights, customary land tenures, or
their marginalization shall be acknowledged. Whenever
restoration is no longer possible, reparation is to be in such form
as mutually determined by the Parties .

The BJE may modify or cancel the forest concessions,
timber licenses, contracts or agreements, mining concessions,
Mineral Production and Sharing Agreements (MPSA), Industrial
Forest Management Agreements (IFMA), and other land tenure
instruments granted by the Philippine Government, including
those issued by the present ARMM .
The MOA-AD describes the relationship of the Central
Government and the BJE as associative, characterized by
shared authority and responsibility. And it states that the
structure of governance is to be based on executive, legislative,
judicial, and administrative institutions with defined powers and
functions in the Comprehensive Compact.

The MOA-AD provides that its provisions requiring
amendments to the existing legal framework shall take effect
upon signing of the Comprehensive Compact and upon
effecting the aforesaid amendments, with due regard to the
non-derogation of prior agreements and within the stipulated
timeframe to be contained in the Comprehensive Compact. As
will be discussed later, much of the present controversy
hangs on the legality of this provision.
The BJE is granted the power to build, develop and
maintain its own institutions inclusive of civil service, electoral,
financial and banking, education, legislation, legal, economic,
police and internal security force, judicial system and
correctional institutions, the details of which shall be discussed
in the negotiation of the comprehensive compact.



As stated early on, the MOA-AD was set to be signed on
August 5, 2008 by Rodolfo Garcia and Mohagher Iqbal,
Chairpersons of the Peace Negotiating Panels of the GRP and the
MILF, respectively. Notably, the penultimate paragraph of the
MOA-AD identifies the signatories as the representatives of the
Parties, meaning the GRP and MILF themselves, and not merely
of the negotiating panels . In addition, the signature page of the
MOA-AD states that it is WITNESSED BY Datuk Othman Bin
Abd Razak, Special Adviser to the Prime Minister of Malaysia,
ENDORSED BY Ambassador Sayed Elmasry, Adviser to
Organization of the Islamic Conference (OIC) Secretary General
and Special Envoy for Peace Process in Southern Philippines, and
SIGNED IN THE PRESENCE OF Dr. Albert G. Romulo,
Secretary of Foreign Affairs of RP and Dato Seri Utama Dr. Rais
Bin Yatim, Minister of Foreign Affairs, Malaysia, all of whom were
scheduled to sign the Agreement last August 5, 2008.

ON THE FIRST SUBSTANTIVE ISSUE

Petitioners invoke their constitutional right to
information on matters of public concern, as provided in
Section 7, Article III on the Bill of Rights:

Sec. 7. The right of the people to
information on matters of public concern shall
be recognized. Access to official records, and
to documents, and papers pertaining to official
acts, transactions, or decisions, as well as to
government research data used as basis for
policy development, shall be afforded the
citizen, subject to such limitations as may be
provided by law

As early as 1948, in Subido v. Ozaeta , the Court has
recognized the statutory right to examine and inspect public
records, a right which was eventually accorded constitutional
status.

The right of access to public documents, as enshrined in
both the 1973 Constitution and the 1987 Constitution, has been
recognized as a self-executory constitutional right .

In the 1976 case of Baldoza v. Hon. Judge Dimaano , the
Court ruled that access to public records is predicated on the right
of the people to acquire information on matters of public concern
since, undoubtedly, in a democracy, the pubic has a legitimate
interest in matters of social and political significance.

x x x The incorporation of this right in the
Constitution is a recognition of the fundamental
role of free exchange of information in a
democracy. There can be no realistic
perception by the public of the nations
problems, nor a meaningful democratic
decision-making if they are denied access to
information of general interest. Information is
needed to enable the members of society to
cope with the exigencies of the times. As has
been aptly observed: Maintaining the flow of
such information depends on protection for both
its acquisition and its dissemination since, if
either process is interrupted, the flow inevitably
ceases. x x x




In the same way that free discussion enables members of
society to cope with the exigencies of their time, access to
information of general interest aids the people in democratic
decision-making by giving them a better perspective of the vital
issues confronting the nation

so that they may be able to criticize
and participate in the affairs of the government in a responsible,
reasonable and effective manner. It is by ensuring an unfettered
and uninhibited exchange of ideas among a well-informed public
that a government remains responsive to the changes desired by
the people .



The MOA-AD is a matter of public concern

That the subject of the information sought in the
present cases is a matter of public concern

faces no serious
challenge. In fact, respondents admit that the MOA-AD is
indeed of public concern . In previous cases, the Court found
that the regularity of real estate transactions entered in the
Register of Deeds , the need for adequate notice to the public
of the various laws the civil service eligibility of a public
employee the proper management of GSIS funds allegedly
used to grant loans to public officials the recovery of the
Marcoses alleged ill-gotten wealth and the identity of party-list
nominees among others, are matters of public concern.
Undoubtedly, the MOA-AD subject of the present cases is of
public concern, involving as it does the sovereignty and
territorial integrity of the State, which directly affects the lives
of the public at large.

Matters of public concern covered by the right to
information include steps and negotiations leading to the
consummation of the contract. In not distinguishing as to the
executory nature or commercial character of agreements, the
Court has categorically ruled:

x x x [T]he right to information
contemplates inclusion of negotiations
leading to the consummation of the
transaction. Certainly, a consummated
contract is not a requirement for the exercise of
the right to information. Otherwise, the people
can never exercise the right if no contract is
consummated, and if one is consummated, it
may be too late for the public to expose its
defects.

Requiring a consummated contract will keep
the public in the dark until the contract, which
may be grossly disadvantageous to the
government or even illegal, becomes fait
accompli. This negates the State policy of full
transparency on matters of public concern, a
situation which the framers of the Constitution
could not have intended. Such a requirement
will prevent the citizenry from participating in
the public discussion of any proposed
contract, effectively truncating a basic right
enshrined in the Bill of Rights. We can allow
neither an emasculation of a constitutional
right, nor a retreat by the State of its avowed
policy of full disclosure of all its transactions
involving public interest .

Intended as a splendid symmetry to the right to
information under the Bill of Rights is the policy of public
disclosure under Section 28, Article II of the Constitution
reading:

Sec. 28. Subject to reasonable conditions
prescribed by law, the State adopts and
implements a policy of full public disclosure of
all its transactions involving public interest .

The policy of full public disclosure enunciated in above-
quoted Section 28 complements the right of access to
information on matters of public concern found in the Bill of
Rights. The right to information guarantees the right of the
people to demand information, while Section 28 recognizes the
duty of officialdom to give information even if nobody demands .

The policy of public disclosure establishes a concrete
ethical principle for the conduct of public affairs in a genuinely
open democracy, with the peoples right to know as the
centerpiece. It is a mandate of the State to be accountable by
following such policy . These provisions are vital to the exercise
of the freedom of expression and essential to hold public
officials at all times accountable to the people .





SUMMARY

The petitions are ripe for adjudication. The failure of
respondents to consult the local government units or communities
affected constitutes a departure by respondents from their
mandate under E.O. No. 3. Moreover, respondents exceeded
their authority by the mere act of guaranteeing amendments to the
Constitution. Any alleged violation of the Constitution by any
branch of government is a proper matter for judicial review.

As the petitions involve constitutional issues which are of
paramount public interest or of transcendental importance, the
Court grants the petitioners, petitioners-in-intervention and
intervening respondents the requisite locus standi in keeping with
the liberal stance adopted in David v. Macapagal-Arroyo.

Contrary to the assertion of respondents that the non-signing
of the MOA-AD and the eventual dissolution of the GRP Peace
Panel mooted the present petitions, the Court finds that the
present petitions provide an exception to the moot and academic
principle in view of (a) the grave violation of the Constitution
involved; (b) the exceptional character of the situation and
paramount public interest; (c) the need to formulate controlling
principles to guide the bench, the bar, and the public; and (d) the
fact that the case is capable of repetition yet evading review.

The MOA-AD is a significant part of a series of agreements
necessary to carry out the GRP-MILF Tripoli Agreement on Peace
signed by the government and the MILF back in June 2001.
Hence, the present MOA-AD can be renegotiated or another one
drawn up that could contain similar or significantly dissimilar
provisions compared to the original.

The peoples right to information on matters of public
concern under Sec. 7, Article III of the Constitution is in splendid
symmetry with the state policy of full public disclosure of all its
transactions involving public interest under Sec. 28, Article II of the
Constitution. The right to information guarantees the right of the
people to demand information, while Section 28 recognizes the
duty of officialdom to give information even if nobody demands.
The complete and effective exercise of the right to information
necessitates that its complementary provision on public disclosure
derive the same self-executory nature, subject only to reasonable
safeguards or limitations as may be provided by law.

The contents of the MOA-AD is a matter of paramount public
concern involving public interest in the highest order. In declaring
that the right to information contemplates steps and negotiations
leading to the consummation of the contract, jurisprudence finds
no distinction as to the executory nature or commercial character
of the agreement.

An essential element of these twin freedoms is to keep a
continuing dialogue or process of communication between the
government and the people. Corollary to these twin rights is the
design for feedback mechanisms. The right to public consultation
was envisioned to be a species of these public rights.

At least three pertinent laws animate these constitutional
imperatives and justify the exercise of the peoples right to be
consulted on relevant matters relating to the peace agenda.

One, E.O. No. 3 itself is replete with mechanics for
continuing consultations on both national and local levels and for a
principal forum for consensus-building. In fact, it is the duty of the
Presidential Adviser on the Peace Process to conduct regular
dialogues to seek relevant information, comments, advice, and
recommendations from peace partners and concerned sectors of
society.

Two, Republic Act No. 7160 or the Local Government Code
of 1991 requires all national offices to conduct consultations
before any project or program critical to the environment and
human ecology including those that may call for the eviction of a
particular group of people residing in such locality, is implemented
therein. The MOA-AD is one peculiar program that unequivocally
and unilaterally vests ownership of a vast territory to the
Bangsamoro people, which could pervasively and drastically result
to the diaspora or displacement of a great number of inhabitants
from their total environment.

Three, Republic Act No. 8371 or the Indigenous Peoples
Rights Act of 1997 provides for clear-cut procedure for the
recognition and delineation of ancestral domain, which entails,
among other things, the observance of the free and prior
informed consent of the Indigenous Cultural
Communities/Indigenous Peoples. Notably, the statute does not
grant the Executive Department or any government agency the
power to delineate and recognize an ancestral domain claim by
mere agreement or compromise.

The invocation of the doctrine of executive privilege as a
defense to the general right to information or the specific right to
consultation is untenable. The various explicit legal provisions
fly in the face of executive secrecy. In any event, respondents
effectively waived such defense after it unconditionally disclosed
the official copies of the final draft of the MOA-AD, for judicial
compliance and public scrutiny.

IN SUM, the Presidential Adviser on the Peace Process
committed grave abuse of discretion when he failed to carry out
the pertinent consultation process, as mandated by E.O. No. 3,
Republic Act No. 7160, and Republic Act No. 8371. The furtive
process by which the MOA-AD was designed and crafted runs
contrary to and in excess of the legal authority, and amounts to
a whimsical, capricious, oppressive, arbitrary and despotic
exercise thereof. It illustrates a gross evasion of positive duty
and a virtual refusal to perform the duty enjoined.

The MOA-AD cannot be reconciled with the present
Constitution and laws. Not only its specific provisions but the
very concept underlying them, namely, the associative
relationship envisioned between the GRP and the BJE, are
unconstitutional, for the concept presupposes that the
associated entity is a state and implies that the same is on its
way to independence.

While there is a clause in the MOA-AD stating that the
provisions thereof inconsistent with the present legal framework
will not be effective until that framework is amended, the same
does not cure its defect. The inclusion of provisions in the
MOA-AD establishing an associative relationship between the
BJE and the Central Government is, itself, a violation of the
Memorandum of Instructions From The President dated March
1, 2001, addressed to the government peace panel. Moreover,
as the clause is worded, it virtually guarantees that the
necessary amendments to the Constitution and the laws will
eventually be put in place. Neither the GRP Peace Panel nor
the President herself is authorized to make such a guarantee.
Upholding such an act would amount to authorizing a usurpation
of the constituent powers vested only in Congress, a
Constitutional Convention, or the people themselves through the
process of initiative, for the only way that the Executive can
ensure the outcome of the amendment process is through an
undue influence or interference with that process.

While the MOA-AD would not amount to an
international agreement or unilateral declaration binding on the
Philippines under international law, respondents act of
guaranteeing amendments is, by itself, already a constitutional
violation that renders the MOA-AD fatally defective.

c). Effect of belligerent occupation on sovereignty-political
laws suspended subject to revival but does not apply to
members of the military (Ruffy v. Chief of Staff, 75 Phil
875);

Facts:
After the Japanese forces during the World War II landed
in Mindoro, instead of surrendering. Major Ramon Ruffy
retreated in the mountains and organized a guerilla unit called
the Bolo Combat Team together with other members of the
Philippine Constabulary and other civilians.

After some Special orders from the General Headquarters
of Gen. Douglas Mc Arthur, Colonel Jurado was appointed
Commanding Officer of the Bolo Area Combat team, and in


June 8,1944, Colo. Jurado relieved Maj. Ruffy and appointed
Capt. Esteban Beloncio as the Commanding Officer of the Bolo
battalion.

Lt. Jurado was allegedly killed by the petitioners, and after of
the commission of the offense, the petitioners withdrew/seceded
from the 6
th
Military district. Under the 93D of the Articles of War,
any person subject to military law who commits murder in time of
war shall suffer either death or life imprisonment and it does not
provide that the SC has the power to review the judgment of
military courts in death and life imprisonment sentences.

The petitioners filed a petition for prohibition, prohibiting the
Military Court Marshall to continue the trial in the grounds 1) that
the petitioners are not under the subject of military law and that 2)
the 93D Articles of War is a violation of the constitution under ART
VIII sec. 2 par 4 since it does not provide any review for the SC to
be made irrespective of whether the punishment id life or death.

Issue/s:
There are two main issue in the case at bar:
1) Whether or not the petitioners are not subject to
Military Law such that they should not be subjected
to military tribunal and
2) Whether or not the law in contention is
unconstitutional.
Held:
Regarding the first issue, the petitioners are indeed under
the subject in Military tribunals and that the case should be tried
under military law. As defined by the 2d Article of War, persons
who are the subject of military law are persons lawfully called,
drafted or ordered into, or to duty or for training in, the said
service, from the dates they are required by the terms of the call,
draft or order to obey the same.

In the case at bar, the fact that the Bolo Area Combat Team
where the petitioners are under service, though a guerilla and not
a regular army, accepted and received appointments, orders,
supplies, salaries and funds from the Southwest Pacific
Command, thereby they operated as a contingent Unit of the 6
th

Military district which is under the operation of the General
Headquarters of the USA Army.

Thus, the petitioners are part of the persons subject to
military Law and thereby should be tried under the Articles of
War.

In the second issue, 93d of the Articles of War is not
repugnant of the Art VII Sec. 2 par 4 of the Constitution, thus the
contention that the SC should have the jurisdiction over their case
is an error in part of the petitioners. It was held that military courts
are agencies of executive character and not part of the Judiciary.
One of the authorities of military courts is for the ordering of court
Marshall has been held to be attached to the constitutional
functions of the President as the commander in Chief,
independent of legislation. Thus, Court Marshall is
instrumentalities under the President in exercise of his position
granted by the Congress as the Commander in Chief of the Army.
There are no provisions of the Constitution violated.

Petition was denied in the absence of merit. The petitioner is
criminally liable and the law in question is not unconstitutional.

Non political laws continues unless changed; judicial
decisions on political matters during the occupation deemed
annulled (Peralta v. Director of Prisons, 75 PHIL 285)

Facts:
The petitioner-defendant, William F Peralta, was charge and
found guilty by the Court of Special and Exclusive Criminal
Jurisdiction for the crime of robbery, as defined and penalized by
the Act. 65 section 2 of the National Assembly, and was sentence
to life imprisonment. The Court of Special and Exclusive Criminal
jurisdiction was created pursuant to Ordinance 7 section 1
promulgated by the President of the Republic of the Philippines
during the Japanese occupation, as part of the Presidents vested
power under the Constitution.

The petitioner filed a writ of Habeas corpus based on the
grounds that 1) that Ordinance 7 that created the Court of
Special and Exclusive Criminal Jurisdiction was a political
instrumentality of the Japanese Imperial army which are
favorable to the occupant and repugnant to the Commonwealth
Constitution in the Philippines and 2) that Act 65 was a law of
political complexion since it punishes political offenses such as
treason, piracy, crimes against public security and order,
rebellion, sedition and other political crimes, and thus, it should
be abrogated and deemed null and void upon the re-occupation
and re establishment of the US Army in the Philippines.

Issue:
Whether or not Act 65 promulgated should be deemed null
and void upon re occupation of the former US Army in the
Philippines since it is a law of political complexion?

Held:
Yes. Act 65 is indeed a law of political complexion
since it punishes political offenses in the Philippines during the
Japanese occupation, and upon the re occupation of the US
Army in the Philippines, laws of political in nature and judicial
decisions rendered pursuant to these laws should be deemed
abrogated.

Under the principle of public international law, which is
the postliminium, all non-political laws is deemed good and valid
unless changed by the new belligerent occupant, also, it
prescribes that on laws of political in nature including judicial
decisions on political matters, is good and valid until the
continuance of occupation of the belligerent occupant, and shall
be cease to be effective and binding until a new occupant
acquired the same territory previously under the jurisdiction of
the former occupant where these political laws and judicial
decisions are enforced.
In the case at bar, there is no question that Act 65,
where the petitioner is criminally liable is good and valid as long
as the Japanese Army is in power in the Philippines, however,
upon the re acquisition of the US army in the Philippines, Act
65, a law of political complexion is abrogated.
Thus, the petitioners petition of writ of habeas corpus
is granted and petitioner was released.


d). Effect of change of sovereignty-political laws
abrogated unless reenacted.

Macariola v. Asuncion, 114 SCRA 77

Supra.

e). Sovereignty as Auto-Limitation

In the succinct language of Jellinek, it "is the property of a
state-force due to which it has the exclusive capacity of legal
self-determination and self-restriction." A state then, if it
chooses to, may refrain from the exercise of what
otherwise is illimitable competence." The opinion was at
pains to point out though that even then, there is at the most
diminution of jurisdictional rights, not its disappearance. (Cited
in REAGAN VS. COMMISSIONER, PEOPLE VS. GOZO, 53
SCRA 476 and COMMISSIONER VS. ROBERTSON, 143
SCRA 397

Reagan v. Commissioner, 30 SCRA 968

"By the Agreement, it should be noted, the Philippine
Government merely consents that the United States
exercise jurisdiction in certain cases. The consent was
given purely as a matter of comity, courtesy, or expediency.
The Philippine Government has not abdicated its
sovereignty over the bases as part of the Philippine
territory or divested itself completely of jurisdiction over
offenses committed therein. Under the terms of the treaty,
the United States Government has prior or preferential but
not exclusive jurisdiction of such offenses. The Philippine
Government retains not only jurisdictional rights not
granted, but also all such ceded rights as the United States
Military authorities for reasons of their own decline to make


use of. The first proposition is implied from the fact of
Philippine sovereignty over the bases; the second from the
express provisions of the treaty." "Nothing is better settled
than that the Philippines being independent and sovereign,
its authority may be exercised over its entire domain. There is
no portion thereof that is beyond its power. Within its limits,
its decrees are supreme, its commands paramount. Its laws
govern therein, and everyone to whom it applies must submit
to its terms. That is the extent of its jurisdiction, both
territorial and personal. Necessarily, likewise, it has to be
exclusive. If it were not thus, there is a diminution of
sovereignty." Then came this paragraph dealing with the
principle of auto-limitation: "It is to be admitted any state
may, by its consent, express or implied, submit to a
restriction of its sovereign rights. There may thus be a
curtailment of what otherwise is a power plenary in character.
That is the concept of sovereignty as auto-limitation, which,
in the succinct language of J ellinek, "is the property of a
state-force due to which it has the exclusive capacity of legal
self-determination and self-restriction." A state then, if it
chooses to, may refrain from the exercise of what otherwise
is illimitable competence." The opinion was at pains to point
out though that even then, there is at the most diminution of
jurisdictional rights, not its disappearance.

Facts: The petitioner, William Reagan is a civilian employee of the
American Aviation providing technical assistance to the US Air
Force in the Philippines appealed from the decision of the Court of
Tax Appeals, disputing the payment of the income tax assessed
on him by respondent CIR on an amount realized by him on his
sale of his auto mobile to a member of the US Marine Corps.

His contention was grounded that by virtue of the Military
Forces Agreement between the United States and the Philippines,
he should not be liable to any tax imposed by the Philippines,
since the transaction was made in Clark Pampanga, a territory
leased by the US from the Philippines and is considered a foreign
nation, thereby it is not under the jurisdiction of the Philippines to
tax.

Issue: Was the contention of the petitioner correct?

Held: No. The contention of the petitioner that Clark Air Base,
being a leased territory by the US from the Philippines and is
considered a foreign country and that the Philippines has no
jurisdiction to tax any transactions made in the said territory is a
mistake in part of the petitioner.

The exercise of sovereign power of a certain State over its
territory is supreme and paramount, such that the laws of that
sovereign State govern all transactions made within its territory. A
concept of sovereignty as an auto limitation, as Jellinek defined it
as is a property of a state-force due to which it has an
exclusive capacity of legal self-determination and self-
restriction tells us that a state, may by its consent, either in
express or implied, submit to a restriction of its sovereign.
Thus, the state may choose to restrict but not totally diminish its
jurisdiction over its territory or refrain from its exercise of sovereign
power.

In the case at bar, through the Military Base Agreement
between the US and the Philippines, leased Clark Airbase for the
establishment of the former of its Army installation, however, the
Philippines does not made any express or implied consent to
withdraw its jurisdiction over the said base, thus, Clark Air Base is
still a part of the territory of the Philippines and cannot be a
foreign country as contended, and that the Philippines can
impose taxes pursuant to the National Internal Revenue code.
The petitioner invoking that pursuant to the Military Bases
Agreement, he is exempt from tax payment of net income taxes a
US national serving in the US military, however, Reagan misread
the provision that further states that is not to be taxed on his
income unless it was derived from the Philippines source or
sources other than the US sources. The transaction made by the
petitioner was executed within the Philippine territory; thus, the
petitioner is liable to pay his net income taxes to the Philippine
government.


People v. Gozo, 53 SCRA 476

Facts Gozo bought a house and lot which was located inside
the US Naval Reservation which is within the territorial
jurisdiction of Olongapo City. Upon the advice of an assistant in
the Mayors Office and some neighbors, she demolished the
house without acquiring the necessary permits and then later on
erected another house. She was then charged by the City
Engineers Office for violating Mun. Ord No. 14 Series of 1964
which requires her to secure permits for any demolition and/or
construction within the City. She was convicted in violation
thereof by the lower court. She appealed and countered that the
City of Olongapo has no administrative jurisdiction over the said
lot because it is within a Naval Base of a foreign country.
ISSUE: Is the Municipal Ordinance enforceable within the US
Naval Base?
HELD: Yes. The Philippine Government has not abdicated its
sovereignty over the bases as part of the Philippine territory or
divested itself completely of jurisdiction over offenses committed
therein. Under the terms of the treaty, the United States
Government has prior or preferential but not exclusive
jurisdiction of such offenses. The Philippine Government retains
not only jurisdictional rights not granted, but also all such ceded
rights as the United States Military authorities for reasons of
their own decline to make use of (Military Bases Agreement).
Hence, in the exercise of its sovereignty, the State through the
City of Olongapo does have administrative jurisdiction over the
lot located within the US Naval Base.

Tanada v. Angara, 272 SCRA 8

Facts: As the representative of the Philippines, the respondent,
Rizalino Navarro, then Secretary of the Department of Trade
and Industry signed a multilateral agreement Final Act
Embodying the Results of the Uruguay Round of Multilateral
Negotiations in Marrakesh, Morocco on August 15, 1994. It was
subsequently ratified by the President of the Philippines and
was concurred by the Senate.

The said Final Act aims to liberalize and expand world
trade and strengthen the interrelationship between trade and
economic policies affecting growth and development of the
different countries in the world. Like the Philippines as one of
their signatories, it was assumed that signing the agreement
would improve the countrys access to foreign markets, increase
movement of personnel thereby increasing job opportunities and
other benefits. In short, the agreement would help the
Philippines improve its economy.

The petitioners filed a petition for certiorari and produced
seven (7) grounds regarding the issue. One of which is that the
provisions of the Agreement establishing the World Trade
Organization unduly limit, restrict and impair Philippine
sovereignty specifically the legislative power which, under Sec.
2, Article XII is vested in the Congress in the Philippines and
Article II Sec. 19 of the 1987 Constitution.

Issue: Does the Final Act, an agreement signed by the
Philippines unduly restrict and impair Philippine sovereignty
specifically the exercise of legislative power of the Congress?

Held: No. The agreement voluntarily signed by the Philippines
does not unduly restrict and impair Philippine sovereignty.

In the concept of auto limitation of sovereignty, a state,
in its consent, expressly or impliedly restricts it jurisdiction over
its territory and limit its exercise of sovereign power, however, it
should be noted that sovereign power, by waiving some of it,
does not equate into a abrogation of sovereign power but only
restricted

Traditionally, the jurisdiction and exercise of sovereign
power is absolute over the territory of a state but, in a voluntary
participation in an international arena, either bilateral or
multilateral, the exercise of sovereign power is subjected to
limitations and restrictions as a member of the international


body or family of nations. A state, through voluntary agreement, is
an automatic waiver of some sovereign power in the spirit of
international law and cooperation.

In the case at bar, the voluntary agreement undergone by
the Philippines as a signatory of the Final Act with a purpose of
economic prosperity and cooperation among nations implies a
voluntary waiver of some of the Philippine sovereignty in
exchange of greater benefits that would render economic
prosperity. Moreover, it does not violate any provisions of the 1987
Constitution since as stated in the Fundamental principle and
State Policies that, the Philippines will adopt a generally accepted
principle of international law as part of the law of the land.
Adopting international law of economic cooperation is part of the
law of the land.

The Constitution does not envision the Philippines to
exercise isolationist method of national development such that it
would rather choose a hermit-type of a country rather than pursuit
of prosperity through international cooperation. In growing
complexities of the society, the modern world, in attaining growth,
have ended in exercising isolationist type of national development
and started international collaboration for a common goal.

Thus, petition was dismissed.

Note:
Article XII. National Economy and Patrimony
The state has the power to regulate its
industries and natural resources and promote
national development.

Article II Section 19. Fundamental Principe and State
Policies.
Sec 19. The State shall develop a self-
reliant and independent national
economy effectively controlled by
Filipinos.

f). Doctrine of Parens Patriae

Govt. V. Monte de Piedad, 35 Phil 738

On June 3, 1863 a devastating earthquake occurred in the
Philippines. The Spanish Dominions then provided $400,000.00
as aid for the victims and it was received by the Philippine
Treasury. Out of the aid, $80,000.00 was left untouched; it was
then invested in the Monte de Piedad Bank which in turn invested
the amount in jewelries. But when the Philippine government later
tried to withdraw the said amount, the bank cannot provide for the
amount. The bank argued that the Philippine government is not an
affected party hence has no right to institute a complaint. Bank
argues that the government was not the intended beneficiary of
the said amount.
ISSUE: Whether or not the Philippine government is competent to
file a complaint against the respondent bank?
HELD: The Philippine government is competent to institute action
against Monte de Piedad, this is in accordance with the doctrine of
Parens Patriae. The government being the protector of the rights
of the people has the inherent supreme power to enforce such
laws that will promote the public interest. No other party has been
entrusted with such right hence as parents of the people the
government has the right to take back the money intended for the
people.

Cabanas V. Pilapil, 58 SCRA 94

In case of conflict on who shall hold the insurance proceeds
for the minor child: whether it should be the uncle by virtue of
the insurance contract entered into by the late father or the
mother, the courts would come in to act as parens patriae
to determine what is best for the interests of the child.

Florentino Pilapil insured himself and he indicated in his insurance
plan that his child will be his beneficiary. He also indicated that if
upon his death the child is still a minor; the proceeds of his
benefits shall be administered by his brother Francisco Pilapil.
The child was only ten years of age when Florentino died and
Francisco then took charge of Florentinos benefits for the child.
On the other hand, the mother of the child Melchora Cabaas
filed a complaint seeking the delivery of the sum of money to be
placed in favor of her and for her to be the childs trustee and
the childs benefits. Francisco asserted the terms of the
insurance policy and that as a private contract its terms and
obligations must be binding only to the parties and intended
beneficiaries.
ISSUE: Whether or not the state may interfere by virtue of
parens patriae to the terms of the insurance policy?
HELD: The Constitution provides for the strengthening of the
family as the basic social unit, and that whenever any member
thereof such as in the case at bar would be prejudiced and his
interest be affected then the judiciary if a litigation has been filed
should resolve according to the best interest of that person. The
uncle here should not be the trustee, it should be the mother as
she was the immediate relative of the minor child and it is
assumed that the mother shall show more care towards the
child than the uncle will. The application of parens patriae here
is in consonance with this countrys tradition of favoring conflicts
in favor of the family hence preference to the parent (mother) is
observed.

P R I N C I P L E S

1. The Philippines as a democratic and republican state.
Why was democratic added to republican? Read:
Tolentino vs. Commission on Elections, 420 SCRA
438(2004);

The basic principles underlying the 1935, 1973 and 1987
Constitutions.

NOTE: The word democratic was included because it
is no longer exclusive for Congress to enact, repeal or
amend laws or to propose amendments to the
Constitution but said power may now be directly
exercised by the people under Section 2, Art. XVII
and Section 32, Art. VI of the Constitution.

FACTS:
Petitioners assailed the manner by which the simultaneous
regular and special elections of 2001 were conducted by the
COMELEC.Petitioners contend that, if held simultaneously, a
special and a regular election must be distinguished in the
documentation as well as in the canvassing of their results.
Thirteen senators were proclaimed from the said election with
the 13th placer to serve that of the remaining term of Sen.
Guingona, who vacated a seat in the senate.

Petitioners sought for the nullification of the special election and,
consequently, the declaration of the 13
th
elected senator.

Issue:
1Whether or not Court had jurisdiction.
2Whether or not the petition was moot.
3Whether or not petioners had locus standi.
4Whether a Special Election for a Single, Three-Year Term
Senatorial Seat was Validly Held on 14 May 2001

RULING:
On the issue of jurisdiction, Court had jurisdiction because what
petitioners were questioning was the validity of the special
election on 14 May 2001 in which Honasan was elected and not
to determine Honasans right in the exercise of his office as
Senator proper under a quo warranto.

On the issue of mootness, it was held that courts will decide a
question otherwise moot if it is capable of repetition yet evading
review.

On the issue of locus standi, the court had relaxed the
requirement on standing and exercised our discretion to give
due course to voters suits involving the right of suffrage,


considering that the issue raised in this petition is likely to arise
again

On the VAlidity of the Election, the Court held that the May 14,
2001 Election was valid.

The Court held that COMELECs Failure to Give Notice of the
Time of the Special Election as required under RA 6645, as
amended, did Not Negate the Calling of such Election. Section 2
of R.A. No. 6645 itself provides that in case of vacancy in the
Senate, the special election to fill such vacancy shall be held
simultaneously with the next succeeding regular election. The
law charges the voters with knowledge of this statutory notice and
COMELECs failure to give the additional notice did not negate the
calling of such special election, much less invalidate it. Further,
there was No Proof that COMELECs Failure to Give Notice of the
Office to be Filled and the Manner of Determining the Winner in
the Special Election Misled Voters. IT could not be said that the
voters were not informed since there had been other accessible
information resources. Finally, the Court held that unless there
had been a patent showing of grave abuse of discretion, the Court
will not interfere with the affairs and conduct of the Comelec.

2. Essential features and manifestations of republicanism

"The Philippines is a republican a democratic state. Sovereignty
resides in the people and all government authority emanates from
them." The Philippines as a republican state, is anchored on the
principle that supreme power rests in the body of the people.
Republicanism more simply means a state that establishes a
government of the people, by the people and for the people. The
Constitution likewise characterizes our country not only as a
republican state, but also a democratic state. Many believe there
is not much difference. If there is, the difference lies in emphasis.
This was what the 1986 constitutional commission highlighted
during its deliberations on this important principle. In inserting the
word "democratic" apart from the word "republican,"
Commissioner Nolledo came up with a description of the insertion
as a "pardonable redundancy." But Commissioner Adolf Azcuna
(now justice of the Supreme Court) hit the nail on the head by
stressing that the word "democratic" is significant because it
emphasizes that our country is one that is participatory in nature.
We do not only elect representatives upon whom sovereign power
is delegated. The people themselves reserve their right to directly
participate in the affairs of state and governance such as voting in
an election, plebiscite, initiative, referendum, and setting up
people's organizations. All these have been enshrined in our
Constitution. The word "democratic" in our Constitution is to
institutionalize and to capture the spirit of "people power."
Elections and direct democratic participation Chief Justice
Renato Puno said in his dissent in Arturo M Tolentino versus
Comelec (January 21, 2004): "An outstanding feature of the 1987
Constitution is the expansion of the democratic space giving the
people greater power to exercise their sovereignty. "Thus, under
the 1987 Constitution, the people can directly exercise their
sovereign authority through the following modes, namely: (1)
elections; (2) plebiscite; (3) initiative; (4) recall; and (5)
referendum. Through elections, the people choose the
representatives to whom they will entrust the exercise of powers of
government.

3. Renunciation of war ; Incorporation Clause; Sec.2

Section 2. The Philippines renounces war as an instrument of
national policy, adopts the generally accepted principles of
international law as part of the law of the land and adheres to the
policy of peace, equality, justice, freedom, cooperation, and amity
with all nations.

It is the principle embodied in Section 2, Article II of the
Constitution which states that The Philippines adopts the
generally accepted principles of international law as part of
the law of the land.

a. Aggressive war vs. defensive war

A war of aggression, sometimes also war of conquest, is a
military conflict waged without the justification of self-defense,
usually for territorial gain and subjugation whereas defensive
war connotes self defense.

b. Incorporation vs. Transformation; hard law vs. soft law,
meaning of generally accepted principles of international law
Pharmaceutical and Health Care Assn. vs. Duque, GR
173034, 10/9/07

Facts: Named as respondents are the Health Secretary,
Undersecretaries, and Assistant Secretaries of the Department
of Health (DOH). For purposes of herein petition, the DOH is
deemed impleaded as a co-respondent since respondents
issued the questioned RIRR in their capacity as officials of said
executive agency.1Executive Order No. 51 (Milk Code) was
issued by President Corazon Aquino on October 28, 1986 by
virtue of the legislative powers granted to the president under
the Freedom Constitution. One of the preambular clauses of the
Milk Code states that the law seeks to give effect to Article 112
of the International Code of Marketing of Breastmilk Substitutes
(ICMBS), a code adopted by the World Health Assembly (WHA)
in 1981. From 1982 to 2006, the WHA adopted several
Resolutions to the effect that breastfeeding should be
supported, promoted and protected, hence, it should be ensured
that nutrition and health claims are not permitted for breastmilk
substitutes.In 1990, the Philippines ratified the International
Convention on the Rights of the Child. Article 24 of said
instrument provides that State Parties should take appropriate
measures to diminish infant and child mortality, and ensure that
all segments of society, specially parents and children, are
informed of the advantages of breastfeeding. On May 15, 2006,
the DOH issued herein assailed RIRR which was to take effect
on July 7, 2006.
Issue: Whether Administrative Order or the Revised
Implementing Rules and Regulations (RIRR) issued by the
Department of Health (DOH) is not constitutional;

Held: YES under Article 23, recommendations of the WHA do
not come into force for members,in the same way that
conventions or agreements under Article 19 and regulations
under Article 21 come into force. Article 23 of the WHO
Constitution reads:
Article 23. The Health Assembly shall have authority to make
recommendations to Members with respect to any matter within
the competence of the Organization
for an international rule to be considered as customary law, it
must be established that such rule is being followed by states
because they consider it obligatory to comply with such rules

Under the 1987 Constitution, international law can become part
of the sphere of domestic law either

By transformation or incorporation. The transformation method
requires that an international law be transformed into a domestic
law through a constitutional mechanism such as local
legislation. The incorporation method applies when, by mere
constitutional declaration, international law is deemed to have
the force of domestic law.

Consequently, legislation is necessary to transform the
provisions of the WHA Resolutions into domestic law. The
provisions of the WHA Resolutions cannot be considered as
part of the law of the land that can be implemented by executive
agencies without the need of a law enacted by the legislature.

c. Incorporation Clause and related matters

Kuroda v. Jalandoni, 83 Phil 171

Petitioner argues that respondent Military Commission has no
Jurisdiction to try petitioner for acts committed in violation of the
Hague Convention on Rules and Regulations covering Land
Warfare and the Geneva Convention because the Philippines is
not a signatory to the first and signed the second only in 1947. It
cannot be denied that the rules and regulation of the Hague and
Geneva conventions form, part of and are wholly based on the
generally accepted principals of international law. In facts these
rules and principles were accepted by the two belligerent nation
the United State and Japan who were signatories to the two
Convention, Such rule and principles therefore form part of the


law of our nation even if the Philippines was not a signatory to the
conventions embodying them for our Constitution has been
deliberately general and extensive in its scope and is not confined
to the recognition of rule and principle of international law as
continued inn treaties to which our government may have been or
shall be a signatory.

Furthermore when the crimes charged against petitioner were
allegedly committed the Philippines was under the sovereignty of
United States and thus we were equally bound together with the
United States and with Japan to the right and obligation contained
in the treaties between the belligerent countries. These rights and
obligation were not erased by our assumption of full sovereignty. If
at all our emergency as a free state entitles us to enforce the right
on our own of trying and punishing those who committed crimes
against crimes against our people. In this connection it is well to
remember what we have said in the case of Laurel vs. Misa (76
Phil., 372).

Facts: The petitioner, Lt. General Shigenori Kuroda of the
Japanese Imperial Army was the Commanding General of the
Japanese Army in the Philippines from 1943 to 1944. He was
charged under the crime prohibited by the Geneva and Hague
Convention for failure of discharging duties as a commander to
control the operations of the members of his command, permitting
them to commit brutal atrocities and other high crimes against
noncombatant civilians and prisoners of war, thereby violating
laws and customs of war. Thus, they were charged of violating the
rules and regulations of Land Warfare.

The Military Commission of the National War Crimes Office,
created by executive Order 68 of the President, was in charge in
the case of the petitioner.

The petitioners filed a petition of writ of prohibition on three
grounds. One of their contention is that the Military Commission
has no jurisdiction to try the case on the grounds that the violation
committed by the petitioners is under the Hague and Geneva
Conventions, the Philippines was recognized to be a signatory to
the latter but not to the former convention that provides rules and
regulations on Land Warfare.

Issue: Was the contention of the petitioners valid?

Held: No. The contention of the petitioners that the Military
Commission has no jurisdiction to the case because the
Philippines is not an adherent nor a signatory to the Hague
Convention that provides rules and regulations on Land Warfare is
invalid.

The fundamental principles and state policies provided under
the constitution adopts a generally accepted principle of
international law as part of the land the Philippines therefore
could adopt and incorporate a generally accepted principle of
international law and would enforce these laws as if it is a part of
its municipal law.

In the case at bar, it was held on two grounds. 1) The Hague
and Geneva Convention established by the United Nations are
vital and provides a generally accepted principle of international
law and jurisprudence, thus, pursuant to the Principles and State
Policies in the Philippine Constitution, these principles of
international law is adopted and incorporated to the municipal law
of the Philippines even if the Philippines was no a signatory nor a
adherent to the Hague Convention. This is so because it is
presumed that, the Philippine constitution provides a deliberate
and extensive scope such that it does not confined the state to
recognize and adopt only those treatise and principle, which the
Philippines may have been and shall be a signatory. 2) Is that the
crimes committed by the petitioner was committed in the time the
Philippines was under the sovereignty of the United States, thus,
the Philippines is bound together with the US and Japan, to the
rights and obligations contained in the treatise between the two
belligerent nations.

Thus, the petition was denied.



Note:
Under the provision of the Hague and Geneva
Convention all those person, military or civilian, who have been
guilty of planning, preparing or waging a war of aggression and
of the commission of the crimes and offenses consequential and
incidental thereto, in violation of the laws and customs of war of
humanity and civilization, are held accountable therefore.

Agustin v. Edu, 88 SCRA 195

The Geneva Convention on Road Signs and Signals, is
also considered part of the law of the Philippines since the
same is a generally accepted principle of international law in
accordance with the Incorporation clause of the Constitution.

Facts:
Pursuant to the 1968 Vienna Convention on Road Signs
and Signals, the Philippine government enacted and ratified
Presidential Decree 207 ordering all car owners to install a road
safety designs and devices. The president of the Philippines
then ordered a letter of instruction for the implementation of
road safety by requiring all car owners to buy and install Early
Warning Devices for road safety to lessen road accidents.

It was concluded in the convention mentioned that the
main cause of numerous road accidents are due to absence of
Early Warning Devices on most vehicles stranded on roads and
highways.
The petitioner, Leovillo Agustin, then filed a petition for
prohibition, prohibiting the government to implement (which was
then issued a TRO) the implementation of the issued Letter of
Instruction assailing that it is oppressive, unreasonable, and
unconstitutional and abuse of police power by the government.
The petitioner prayed for the declaration of its
unconstitutionality.

Issue: Pursuant to the Section 2 of the Fundamental Principle
and State Policy under the Constitution of the Philippines,
should the 1968 Vienna Convention not to be adopted by the
Philippines in grounds that it is unreasonable and oppressive on
the part of the car-owners?

Held: The question of adopting the 1968 Vienna Convention on
Road Signs and Signals as part of the law of the land is another
question as to its reasonableness and oppressiveness.

The provisions of the convention as the basis of the Letter
of Instruction in question maybe unreasonable to implement in
the Philippines because, considering the prize imposed on each
pair of EWD, it would be too burdensome to the public.
Moreover, the government has shown no clear effort to provide
other alternative and cheaper means to have warning devices
on every motor vehicle.

However, it maybe unreasonable and burdensome to the
public, it should be considered that as part of the Philippines
Constitution, the Philippines could adopt a generally accepted
principle of International law as part of the law of the land.

In the case at bar, the 1968 Vienna Convention provides a
generally accepted principle of international law. The convention
purpose is to strengthen and maintain public safety on roads
and transportation, thus a generally accepted international law
like the provisions of the mentioned convention, the Philippines
can adopt such law as part of the law of the land.

Petition was dismissed, TRO lifted.

ISAE vs. QUISUMBING, 333 SCRA 13

Facts: The petitioner, International School Alliance of Educators
is a legitimated labor union and collective bargaining
representative of all faculty members of the International
school, Inc. filed a special civil action for certiorari contesting
that there has been discrimination against local hires of the
School and foreign hires.

Pursuant to PD 732, the International school Inc. was
created for dependents of foreign diplomatic personnel and


other temporary residents. The School hires both foreign and local
hires as educators. However, foreign-hires enjoys benefits and
privileges not accorded to local-hires like housing, travel
allowances etc including a twenty-five percent higher salary rates
compared to the local hires- because of two grounds 1) dislocation
factors and 2) limited tenure. This reason brought the petition.

The petitioner claims that the point-of-hire classification
employed by the school is discriminatory to Filipinos and that the
grant of higher salaries to foreign-hires constitutes racial
discrimination.

Issue: Was the contention of the petitioners correct?

Held: Yes, the contention of the petitioners that the regulation of
the School pertaining to hiring personnel constitutes racial
discrimination.

Both in the Philippines constitution and its law like the Labor
Code and ART XIII Section 1 (Articles on Social Justice and
Human Rights and Article II Section 18 of the Fundamental
Principle and State Policies, and other adopted international laws
as incorporated in the municipal law of the Philippines by virtue of
Section 2 of Article II of the 1987 Constitution, like the Universal
Declaration of Human Rights, the International Covenant on
Economic, Social and Cultural Rights, the International
Convention on Elimination on all Forms of Racial Discrimination,
the Convention Against Discrimination in Education, the
Convention Concerning Discrimination in Respect of Employment
and Occupation, all provides principles that abhors discrimination
of any kind.

In the case at bar, the School cannot invoke the need to
entice foreign-hires to leave their domicile to rationalize the
distinction in salary rates without violating the principle of equal
work for equal pay. The local-hires work and perform the same
functions as those foreign hires, thus there is no reasonable
ground for the school to provide lesser compensation and salaries
as it would violate provisions on labor and against discrimination
both prescribed by national and international law.

Petition granted.

On UN Declaration of Human Rights

Mejoff v. Director of Prisons, 90 Phil. 70

The Philippines adopts the Universal Declaration of
Human Rights since it is a generally accepted principle of
international law. As such, it should be applied to illegal
aliens like the petitioner so that it would be a violation of the
said international law to detain him for an unreasonable
length of time since no vessel from his country is willing to
take him.

"The meaning of "reasonable time" depends upon the
circumstances, specially the difficulties of obtaining a
passport, the availability of transportation, the diplomatic
arrangements concerned and the efforts displayed to send
the deportee away. Considering that this Government desires
to expel the alien, and does not relish keeping him at the
people's expense, we must presume it is making efforts to
carry out the decree of exclusion by the highest officer of the
land. On top of this presumption assurances were made
during the oral argument that the Government is really trying
to expedite the expulsion of this petitioner. On the other
hand, the record fails to show how long he has been under
confinement since the last time he was apprehended. Neither
does he indicate neglected opportunities to send him abroad.
And unless it is shown that the deportee is being indefinitely
imprisoned under the pretense of awaiting a chance for
deportation 3 or unless the Government admits that it can not
deport him or unless the detainee is being held for too long a
period our courts will not interfere.

Facts: Mejoff is of Russian descent and was arrested as a
Japanese spy after the Philippine liberation. It was found out that
he illegally entered the Philippines in 1944. He was without
inspection and admission by the immigration officials at a
designated port of entry. He was then ordered to be deported to
Russia on the first available transportation to said country. But
Russian ships refused to take him due to their alleged lack of
authority to do so. He was then transferred to the Bilibid Prison
and was kept in detention as the Commissioner of Immigration
believes it is of best interest to detain the unwanted alien while
arrangements for his deportation are being made. Mejoff
contends that he was legally brought to the Philippines by the
then Japanese forces and he may not now be deported. He also
contends that the statutory period to deport him has long lapsed
and that we cannot detain him for an unreasonable period of
time pursuant to the Universal Declaration on Human rights.
ISSUE: Whether or not Mejoff shall remain in detention?
HELD: The government has the power and the authority to eject
from the Islands any and all unwanted aliens including members
of the Niponese Army of occupation . On the other hand, there
is no indication that the statutory period to deport him has
lapsed. He entered the country illegally in 1944 and was
arrested in 1948. Pursuant to Sec 37 of the Philippine
Immigration Act of 1940 an unwanted alien is subject to
deportation within 5 years from arrest. And he may be held for a
reasonable period of time (depending on the circumstances)
while arrangements are being held for his deportation. There is
no allegation however as to the length of time that he has been
detained. Hence, the same cannot be construed as
unreasonable.
Salonga v. Hermoso, 97 SCRA 121

Supra. Salonga filed a mandamus proceeding to compel
Hermoso of the Travel Processing Center to issue a
certificate of eligibility to travel to Salonga. This is not
however the first time that Salonga filed such a complaint and
this issue is considered moot and academic. The Soc-Gen, in
his reply, has already indicated that the certificate was indeed
issued and that there should be no cause of action. The
issuance of the certificate is in pursuant to the Universal
Declaration of Human Rights on the Right to Travel. The
Philippines, even though it is under martial law, shall in no
instance facilitate the erosion of human rights. The Travel
Processing Center should exercise the utmost care to avoid
the impression that certain citizens desirous of exercising
their constitutional right to travel could be subjected to
inconvenience or annoyance this is to avoid such similar
cases to face the Court which needlessly expire the Courts
effort and time.

Mijares v. Raada, 455 SCRA 397 (2005)

Facts: Invoking the Alien Tort Act, petitioners Mijares, et al.*, all
of whom suffered human rights violations during the Marcos era,
obtained a Final Judgment in their favor against the Estate of
the late Ferdinand Marcos amounting to roughly $1.9B in
compensatory and exemplary damages for tortuous violations of
international law in the US District Court of Hawaii. This Final
Judgment was affirmed by the US Court of Appeals.

As a consequence, Petitioners filed a Complaint with the RTC
Makati for the enforcement of the Final Judgment, paying P410
as docket and filing fees based on Rule 141, 7(b) where the
value of the subject matter is incapable of pecuniary estimation.
The Estate of Marcos however, filed a MTD alleging the non-
payment of the correct filing fees. RTC Makati dismissed the
Complaint stating that the subject matter was capable of
pecuniary estimation as it involved a judgment rendered by a
foreign court ordering the payment of a definite sum of money
allowing for the easy determination of the value of the foreign
judgment. As such, the proper filing fee was P472M, which
Petitioners had not paid.

Issue: Whether or not the amount paid by the Petitioners is the
proper filing fee.

Held:

Yes, but on a different basisamount merely corresponds to
the same amount required for other actions not involving


property. RTC Makati erred in concluding that the filing fee should
be computed on the basis of the total sum claimed or the stated
value of the property in litigation. The Petitioners Complaint was
lodged against the Estate of Marcos but it is clearly based on a
judgment, the Final Judgment of the US District Court. However,
the Petitioners err in stating that the Final Judgment is incapable
of pecuniary estimation because it is so capable. On this point,
Petitioners state that this might lead to an instance wherein a first
level court (MTC, MeTC, etc.) would have jurisdiction to enforce a
foreign judgment. Under the B.P.129, such courts are not vested
with such jurisdiction. 33 of B.P.129 refers to instances wherein
the cause of action or subject matter pertains to an assertion of
rights over property or a sum of money. But here, the subject
matter is the foreign judgment itself. 16 of B.P.129 reveals that
the complaint for enforcement of judgment even if capable of
pecuniary estimation would fall under the jurisdiction of the RTCs.
Thus, the Complaint to enforce the US District Court judgment is
one capable of pecuniary estimations but at the same time, it is
also an action based on judgment against an estate, thus placing
it beyond the ambit of 7(a) of Rule 141. What governs the proper
computation of the filing fees over Complaints for the enforcement
of foreign judgments is 7(b)(3), involving other actions not
involving property.

Shangri-la v. Developers Group of Companies, Inc., 486
SCRA 405 (2006).

Facts: On June 21, 1988, the Shangri-La International Hotel et. al.
("Shangri-La Group"), filed with the Bureau of Patents,
Trademarks and Technology Transfer ("BPTTT") a petition praying
for the cancellation of the registration of the "Shangri-La" mark
and "S" device/logo issued to the Developers Group of
Companies, Inc., on the ground that the same was illegally and
fraudulently obtained and appropriated for the latter's restaurant
business. The Shangri-La Group alleged that it is the legal and
beneficial owners of the subject mark and logo; that it has been
using the said mark and logo for its corporate affairs and business
since March 1962 and caused the same to be specially designed
for their international hotels in 1975, much earlier than the alleged
first use thereof by the Developers Group in 1982.
Likewise, the Shangri-La Group filed with the BPTTT its own
application for registration of the subject mark and logo. The
Developers Group filed an opposition to the application.
Almost three (3) years later, or on April 15, 1991, the Developers
Group instituted with the Regional Trial Court of Quezon City,
Branch 99, a complaint for infringement and damages with prayer
for injunction against the Shangri-La Group.
On January 8, 1992, the Shangri-La Group moved for the
suspension of the proceedings in the infringement case on
account of the pendency of the administrative proceedings before
the BPTTT. The trial court denied the motion to suspend. The
Shangri-La Group then filed a petition for certiorari before the
Court of Appeals. On February 15, 1993, the Court of Appeals
rendered its decision dismissing the petition for certiorari.
Issue: The core issue in this case is simply whether, despite the
institution of an Inter Partes case for cancellation of a mark with
the BPTTT (now the Bureau of Legal Affairs, Intellectual Property
Office) by one party, the adverse party can file a subsequent
action for infringement with the regular courts of justice in
connection with the same registered mark.
Ruling: Section 151.2 of Republic Act No. 8293, otherwise known
as the Intellectual Property Code, provides, as follows -
SECTION 151.2. Notwithstanding the foregoing provisions, the
court or the administrative agency vested with jurisdiction to hear
and adjudicate any action to enforce the rights to a registered
mark shall likewise exercise jurisdiction to determine whether the
registration of said mark may be cancelled in accordance with this
Act. The filing of a suit to enforce the registered mark with the
proper court or agency shall exclude any other court or agency
from assuming jurisdiction over a subsequently filed petition to
cancel the same mark. On the other hand, the earlier filing of
petition to cancel the mark with the Bureau of Legal Affairs shall
not constitute a prejudicial question that must be resolved before
an action to enforce the rights to same registered mark may be
decided.
Similarly, Rule 8, Section 7, of the Regulations on Inter Partes
Proceedings, provides to wit:
SECTION 7. Effect of filing of a suit before the Bureau or with
the proper court. - The filing of a suit to enforce the registered
mark with the proper court or Bureau shall exclude any other
court or agency from assuming jurisdiction over a subsequently
filed petition to cancel the same mark. On the other hand, the
earlier filing of petition to cancel the mark with the Bureau shall
not constitute a prejudicial question that must be resolved
before an action to enforce the rights to same registered mark
may be decided.
Hence, as applied in the case at bar, the earlier institution of an
Inter Partes Case by the Shangri-La Group for the cancellation
of the "Shangri-La" mark and "S" device/logo with the BPTTT
cannot effectively bar the subsequent filing of an infringement
case by registrant Developers Group. The law and the rules are
explicit.
The rationale is plain: Certificate of Registration No. 31904,
upon which the infringement case is based, remains valid and
subsisting for as long as it has not been cancelled by the
Bureau or by an infringement court. As such, Developers
Group's Certificate of Registration in the principal register
continues as "prima facie evidence of the validity of the
registration, the registrant's ownership of the mark or trade-
name, and of the registrant's exclusive right to use the same in
connection with the goods, business or services specified in the
certificate." Since the certificate still subsists, Developers Group
may thus file a corresponding infringement suit and recover
damages from any person who infringes upon the former's
rights.
Furthermore, the issue raised before the BPTTT is quite
different from that raised in the trial court, said the Court. The
issue raised before the BPTTT was whether the mark registered
by Developers Group is subject to cancellation, as the Shangri-
La Group claims prior ownership of the disputed mark. On the
other hand, the issue raised before the trial court was whether
the Shangri-La Group infringed upon the rights of Developers
Group within the contemplation of Section 22 of Republic Act
166.

Comment:
(1) It is interesting to note that the petition for cancellation was
filed three years ahead of the infringement suit. In the earlier
Shangri-La case involving the same parties on the matter of
injunction, the Supreme Court ruled, "[t]he prima facie validity of
its registration has been put in serious question by the
cancellation cases filed by Shangri-La with the BPTTT 3 years
ahead of the complaint." Then the Court apparently reversed
itself saying that the prima facie validity of the registration
remains in effect "for as long as it has not been cancelled." Has
the meaning of the term prima facie got lost somewhere?
(2) The Shangri-La case appears to be a dangerous precedent.
If the courts find Shangri-La guilty of trademark infringement,
can the Intellectual Property Office still cancel the registration of
Developers Group on the basis of evidence presented before it?
The Supreme Court was quick to avoid this situation by ordering
the then Bureau of Patents to suspend, but not to stop, the
cancellation proceedings.
23

(3) Whatever happened to the rule on primary jurisdiction and
the rule on agency expertise? This ruling appears to be
inconsistent with the ruling in Amigo Manufacturing v. Cluett
Peabody
24
where the Court said that administrative agencies'
(IPO in this case) findings of fact in matters falling under their
jurisdiction are generally accorded great respect, it not finality.
That's consistency and predictability for us.





Government of Hongkong Special
Administrative Region v. Olalia, 521 SCRA 470 (2007)

Juan Muoz was charged before a Hong Kong Court with
several counts of offenses in violation of Hong Kong laws. If
convicted, he faces a jail term of 7 to 14 years for each charge.
After Juan Muoz was arrested in the Philippines, the Hong Kong
Special Administrative Region filed with the RTC of Manila a
petition for the extradition of Juan Muoz. On December 20, 2001,
Judge X of RTC-Manila allowed Juan Muoz to post bail.
However, the government of Hong Kong alleged that the trial court
committed grave abuse of discretion amounting to lack or excess
of jurisdiction in admitting him to bail because there is nothing in
the Constitution or statutory law providing that a potential
extraditee a right to bail, the right being limited solely to criminal
proceedings. May Juan Muoz, a potential extradite, be granted
bail on the basis of clear and convincing evidence that he is not a
flight risk and will abide with all the orders and processes of the
extradition court?

SUGGESTED ANSWER:

Yes. In a unanimous decision the SC remanded to the Manila
RTC, to determine whether Juan Muoz is entitled to bail on the
basis of clear and convincing evidence. If Muoz is not entitled to
such, the trial court should order the cancellation of his bail bond
and his immediate detention; and thereafter, conduct the
extradition proceedings with dispatch.

If bail can be granted in deportation cases, we see no justification
why it should not also be allowed in extradition cases. Likewise,
considering that the Universal Declaration of Human Rights
applies to deportation cases, there is no reason why it cannot be
invoked in extradition cases. After all, both are administrative
proceeding where the innocence or guilt of the person detained is
not in issue, the Court said.

Citing the various international treaties giving recognition and
protection to human rights, the Court saw the need to reexamine
its ruling in Government of United States of America v. Judge
Purganan which limited the exercise of the right to bail to criminal
proceedings. (visit fellester.blogspot.com)

It said that while our extradition law does not provide for the grant
of bail to an extraditee, there is no provision prohibiting him or her
from filing a motion for bail, a right under the Constitution.

It further said that even if a potential extradite is a criminal, an
extradition proceeding is not by its nature criminal, for it is not
punishment for a crime, even though such punishment may follow
extradition. It added that extradition is not a trial to determine the
guilt or innocence of potential extraditee. Nor is it a full-blown civil
action, but one that is merely administrative in character. (GR No.
153675, Government of Hong Kong Special Administrative
Region v. Judge Olalia, Jr. and Muoz, April 19, 2007)

Note: In Government of United States of America v. Judge
Purganan, September 24, 2002, The SC ruled that Mark
Jimenez is not entitled to the right to bail and provisional
liberty while the extradition proceedings are pending except
upon a clear and convincing showing (1) that, once granted
bail, the applicant will not be a flight risk or a danger to the
community; and (2) that there exist special, humanitarian and
compelling circumstances.

c. IN CASE OF CONFLICT, WHICH SHOULD PREVAIL ? Treaty
or Constitution or law ? SEE ruling in Balikatan 02-1 in LIM et al.
Vs. Exec. Sec., GR 15445, 4/11/02

Facts: The petitioners, Arthur Lim et al, filed a petition for
certiorari and prohibition, restraining the respondents from
proceeding with the Balikatan 02-1 as provided by the Mutual
Defense Treaty entered between the two states. The petitioners
attacked the constitutionality of the said agreement.

The contention of the petitioner is that, the Visiting Forces
Agreement between the US and the Philippines does not
authorize American Soldiers to enter and engage in combat
operations in Philippine territory and that according to the
constitution, it prohibits foreign military bases, troops or facilities
unless a treaty permits the same. In view of this grounds, the
petitioner contended that the deployment of American forces in
the Philippine territory is unconstitutional since the Mutual
Defense Treaty through VFA only contemplates a joint military
activities in case or external attack. The Abu Sayaff Group, is
one of the reasons why the joint military exercise for counter
terrorism is conducted, as argued the ASG group is not
considered as an external element and it is only a group of
bandits present within the Philippine territory.

In return, the respondents answered that, the deployment
of foreign troops in the Philippine territory is within the
provisions of the VFA such that it falls under the umbrella of
sanctioned or allowable activities in the context of the
agreement- the combat related activities which includes socio-
economic assistance projects, mutual counter terrorism
advising, assisting and training and other.


Issue: In case of conflict between the constitution and
international agreement, what should prevail in the case at bar?
Held: Treatise should prevail over municipal law of the
contracting parties in the international agreement.

Under the principle of public international law pacta sunt
servada, which means that, every treaty in force is binding
upon the parties and should be performed by them in good
faith. Under this principle also invoke that a party cannot
use the provisions of their municipal law as an excuse to
justify their non-compliance on the treaty.

There is a clear conflict between the Philippine
Constitution and the treaty the Philippines agreed with the US
(MDT and the VFA). The constitution only provides that foreign
military presence in the Philippines can only be allowed if there
is an external threat, thus it prohibits foreign military facilities
and army to engage an aggressive/offensive war within the
Philippine territory, however and the VFA as compliment of the
MDT between the Philippines and the US, it gives legitimacy on
the deployment of US army and facilities in the Philippines as
provided in the treaty. In such conflict, pursuant to the principle
of public international law mentioned, it is the treaty that should
be favored over the municipal law.

This is so because, in the case at bar, 1) Balikatan 02-1
should be construed as a mutual anti-terrorism advising,
assisting and training exercise meant to preserve and maintain
public order and security against the threat of terrorism,
externally or internally like the ASG, and it is within the
allowable and permissible activities prescribed in the
agreement. 2) Long before the establishment of the constitution,
the treatise the Philippines entered, specifically the Mutual
Defense Treaty on 1951 has been a long source of obligation of
the Philippines in the international arena, thus under the
principle, the treaty should be honored and performed in good
faith.

Petition denied. International treatise prevails.

Section 3. Civilian Authority is, at all times, supreme over the
military. The armed forces of the Philippines is the protector of
the people and the state. Its goal is to secure the sovereignty of
the state and the integrity of the national territory.

a. The President as Commander in chief
b. READ Sec 18, Art. VII-

Section 18. The President shall be the Commander-in-Chief of
all armed forces of the Philippines and whenever it becomes
necessary, he may call out such armed forces to prevent or
suppress lawless violence, invasion or rebellion. In case of
invasion or rebellion, when the public safety requires it, he may,
for a period not exceeding sixty days, suspend the privilege of
the writ of habeas corpus or place the Philippines or any part
thereof under martial law. Within forty-eight hours from the
proclamation of martial law or the suspension of the privilege of
the writ of habeas corpus, the President shall submit a report in


person or in writing to the Congress. The Congress, voting jointly,
by a vote of at least a majority of all its Members in regular or
special session, may revoke such proclamation or suspension,
which revocation shall not be set aside by the President. Upon the
initiative of the President, the Congress may, in the same manner,
extend such proclamation or suspension for a period to be
determined by the Congress, if the invasion or rebellion shall
persist and public safety requires it.
The Congress, if not in session, shall, within twenty-four hours
following such proclamation or suspension, convene in
accordance with its rules without need of a call.
The Supreme Court may review, in an appropriate proceeding
filed by any citizen, the sufficiency of the factual basis of the
proclamation of martial law or the suspension of the privilege of
the writ of habeas corpus or the extension thereof, and must
promulgate its decision thereon within thirty days from its filing.
A state of martial law does not suspend the operation of the
Constitution, nor supplant the functioning of the civil courts or
legislative assemblies, nor authorize the conferment of jurisdiction
on military courts and agencies over civilians where civil courts are
able to function, nor automatically suspend the privilege of the writ
of habeas corpus.
The suspension of the privilege of the writ of habeas corpus shall
apply only to persons judicially charged for rebellion or offenses
inherent in, or directly connected with, invasion.
During the suspension of the privilege of the writ of habeas
corpus, any person thus arrested or detained shall be judicially
charged within three days, otherwise he shall be released.

c. Civilian supremacy-not diminished by calling the marines to
maintain peace and order or suppress violence (IBP vs.
ZAMORA, 338 SCRA 81

Facts: Because of the alarming increase of crimes and lawless
violence in Metro Manila such as robbery, kidnapping and
carnappings, President Estrada verbally ordered the Philippine
National Police (PNP) and the Philippine Marines to carry out a
Joint Implementing Police Visibility Patrols, called task Force
TULUNGAN to be headed by the former. The joint patrol forces of
the PNP and Marines will be deployed in the different hot spot
areas in Metro Manila for the purpose of crime prevention and
suppression. The rationale behind such orders from the president
are grounded on the power of the President as Commander in
Chief under Section 18 Article VII of the Constitution and the
Presidents desire to improve peace and order in Metro Manila by
implementing an effective crime prevention program such as
increase and augmented police visibility in the area.

Pursuant to the Presidents mandate, the PNP Chief
formulated a Letter of Instruction (LOI) providing a detailed
manner on how the Task Force Tulungan should be implemented.

The petitioner, the Integrated Bar of the Philippines (IBP)
filed a special civil action of certiorari and prohibition to annul the
issued LOI and to declare the deployment of the Philippine
Marines null and void and unconstitutional. They provided four
grounds for their contention.

One of their contentions for the unconstitutionality of the LOI
is that the administration is unwittingly making the military more
powerful than what it should really be under the constitution by
militarizing law enforcement in Metro Manila. The petitioner
asserts that by deployment of the military, the civilian law
enforcement is militarized such that violates Section 3 Art II of the
Constitution,

Issue: Does the Task Force Tulungan unconstitutional such that
the military became superior over civilians making it inconsistent
under section 3 Art II of the constitution?

Held: No, the issued LOI does not make the military superior over
civilians such that it will be inconsistent of the provision of the
constitution.
As provided by the constitution, the President, as exercise
of his power as the Commander in Chief of all armed forces in
the Philippines, may call such armed forces whenever
necessary to prevent and suppress lawless violence, invasion
and rebellion. The intent of the framers of the constitution is
that, it vested the president a sole discretionary power to call on
the military whenever possible and that such discretion is
founded on factual basis. It cannot be the subject of judicial
review, unless there is a grave abuse of power or it was
exercised outside the permissible limit provided in the
constitution such that it violates the civilian supremacy clause
provided by the constitution.

In the case at bar, it is clear and unquestionably that Metro
Manila is suffering from increase of lawless violence, thus,
pursuant to the power vested on the president, he may call the
armed forces to supplement the National Police in assisting
civilian law enforcement, thus, the calling made by the
president is grounded on factual basis and that its aim is for the
protection and maintenance of public peace and order.
Referring to the civilian supremacy clause, since the role
of the military was under the permissible use of military on
civilian law enforcement. It was clearly detailed in the LOI that
the Marines have limited role in the program such that it does
not breach the civilian supremacy since the program is under
the leadership and management of the PNP, which is not part of
the Military and is a civilian institution. The authority of the
program in solely vested on the PNP not on the AFP and the
deployment of the Marines are only for supplemental purposes.

The grounds presented for the contention of the petitioner
is also held as unfounded such that the petitioner fails to
present sufficient evidence to show that there is a grave abuse
of use of military power in the program and that military power
was implemented and used in manner that is outside the
permissible boundary provided by the constitution, such that it
violates Art II Section 3 or the civilian supremacy clause
provided by the constitution.

Lastly, even if the Court use rigid standards to identify if an
act by the military constitutes a breach of civilian supremacy,
the results does not conform with the contention of the
petitioners. One of these standards is to identify if there is a
regulatory power used by the military. Using this standard in the
case at bar, the military does not direct or control the program
since the authority of the implemented measure of civilian law
enforcement is vested in the PNP and the military is only
deployed for supplementary purpose as shown in the detailed
procedures in the LOI by the PNP.

Another standard is identifying whether the military use
Proscriptive power, in the case at bar, there is no evident to
show that the military has the power to condemn or prohibit
since all person arrested are brought in the nearest police
precinct for disposition.

Lastly, is whether the military used Coercive power in
the case at bar. The Court held that there is no coercive power
used by the military since the materials and equipment used in
the implementation of Task Force Tulungan by the military are
low impact and defensive in nature such that they only used
whistles, handcuffs and Probatons.

The calling of the military to maintain peace and order
or suppress lawless violence does not constitute a breach to the
civilian supremacy clause.
Thus, the petition was dismissed.

d. role of the military

Section 4. The Prime Duty of the government is to serve and
protect the people. The government may call upon the people
to defend the state, in the fulfillment thereof, all citizens may
be required, under conditions provided by law, to render
personal military or civil service.

a. Refusal to enlist (P. V. Lagman, 66 Phil 13 [1913])



The appellants argument that he does not want to join the
armed forces because he does not want to kill or be killed
and that he has no military inclination is not acceptable
because it is his obligation to join the armed forces in
connection with the defense of the State provision of the
Constitution.

Facts: The appellants, Tranquillino Lagman and Primitivo de
Sosa, both 20 years old, was charged by the Court of First
Instance, with a violation of Section 60 of the Commonwealth Act
no. 1 otherwise known as the National Defense Act for being
willfully and unlawfully refused to register in the military service
between the 1
st
and 7
th
April 1936.

Both appellants, does not deny the allegations, and admitted
that they have not registered to the military service because they
both are fatherless and have family to support. However, these
excuses were not taken into consideration since the national
Defense Law requires any men of legal age for compulsory
military service, to register before the Acceptance Board to render
their service to the military.

The appellants filed an appeal on the grounds that the
National Defense Law was unconstitutional as it defeats the
provisions of the Constitution specifically under Article II Section 2
(3).

Issue: Was the contention of the appellants correct?

Held: No. The contention of the appellants that the National
defense Law violates art II section 2(3) of the Constitution is not
correct.

Under the Article in question, the defense of the state is a
prime duty of government and in fulfillment of this duty all citizens
may be required by law to render personal military or civil service.
This provisions prescribes that, the government is mandated to
defend the state through its military, and to be able to defend and
maintain the national sovereignty and territorial integrity of the
state, the government requires its citizens to render their military
service, should there be no enough strength of the military to
comply their mandated duty.

In the case at bar, the National Defense Law does not defeat
the provision of the constitution, on the contrary, it is a law that
faithfully comply with the provision, such that, it compel the
citizens of the state to undergo compulsory military training, in
case of emergency such as war, should there be a shortage of
personnel in the army to defend and execute such mandate. The
law just reiterated that the compulsory military training service is a
consequence of duty to defend the territorial integrity and
sovereignty of the state and is reciprocal with its duty to defend
the life, liberty and property of the citizen.
Executive

Thus, the judgment appealed from is affirmed, the petition is
denied.

Section 5. the maintenance of peace and order, protection of life,
liberty and property, and promotion of the general welfare are
essential for the enjoyment by all the people of the blessings of
democracy.

Section 6. The separation of Church and State shall be inviolable.

a. Correlative provisions
1). Sec. 5, Art. III-non establishment/free exercise and
religious test
2). Sec. 28(3) Art. VI-tax exempt
3). Sec. 29 (2) Art. VI-non appropriation clause
4). Par C, 2 (5), Art. IX-religious party
5). Sec. 3(3) Art. XIV-optional religious instructions in public
schools
.
ART. III, Sec. 5. No law shall be made respecting an
establishment of religion, or prohibiting the free exercise thereof.
The free exercise and enjoyment of religious profession and
worship, without discrimination or preference, shall forever be
allowed. NO RELIGIOUS TEST SHALL BE REQUIRED FOR
THE EXERCISE OF CIVIL OR POLITICAL RIGHTS.

ART. VI, Sec. 28 (3). Charitable institutions, churches,
mosques, non-profit cemeteriesactually, directly and
exclusively used for religious, charitable, or educational
purposes shall be exempt from taxation.

ART. VI, Sec. 29 .(2). No public money or property shall be
appropriated, applied, paid, for the benefit, directly or indirectly,
for the use, benefit, or support of any sect, church,
denomination or religion, except when such priest, minister.. is
assigned to the armed forces, or to any penal institution, or
government orphanage or leprosarium.

ART. IX, C, 2(5). Religious denominations and sects shall not
be registeredas political parties. (NOTE: Religious
organizations are also prohibited in connection with sectoral
representatives under Art. VI)

ART. XIV, Sec. 3(3). At the option in writing by parents, religion
shall be allowed to be taught to their children in elementary and
high schools within the regular class hours by instructors
designated or approved by religious authorities to which said
children belong, without additional cost to the government.

b. READ:

Aglipay v. Ruiz, 64 Phil 201
(1) benevolent neutrality or accommodation, whether
mandatory or permissive, is the spirit, intent and
framework underlying the religion clauses in our
Constitution; and (2) in deciding respondents plea of
exemption based on the Free Exercise Clause (from
the law with which she is administratively charged), it is
the compelling state interest test, the strictest test

Facts: The petitioner (Gregorio Aglipay) is the Supreme Head
of the Philippine independent Church that seeks the issuance
from the court of writ of prohibition preventing the respondent
Director of Post (Ruiz) from issuing and selling a
commemorative stamp of the 33
rd
International Eucharistic
Congress as enacted and appropriated under RA 4052.

The petitioner contented that RA 4052 is unconstitutional
as it violated Article VI Section 29 (2) stating that no public
money shall be appropriated, applied or paid to any sect, church
or any system of religion including its priests preacher etc

The law in question appropriates P60, 000 for printing the
new commemoration stamps. (The stamp design is suggestive
of Roman Catholicism i.e chalice)

Issue: Does RA 4052 unconstitutional as it violates the
provision of the constitution specifically Article VI Section 29
(2)?

Held: No. RA 4052 does not violate the provision of the
constitution as alleged by the petitioner.

In the case at bar, the commemorative stamp are not
issued and sold to appropriate or pay the Roman Catholic
Church but its purpose is to advertise the Philippines and attract
tourist in the Philippines. The proceeds are not for any religious
institution but for the interest of the public.

Estrada vs. Escritor, 492 SCRA 1(2006)

Facts: The respondent, Soledad Escritor, a Court Interpreter of
Branch 253 Regional Trial Court of Las Pias, was charged by
complainant Alejandro Estrada, of disgraceful and immoral
conduct of a public officer for living a man not her husband
(Luciano Quilapio) and having borne a child within her live in
arrangement, which is a violation under Book V, Title I, Chapter
VI, Section 46 (b) (5) of the Revised Administrative Code.

Estrada requested Judge Caoibes, presiding judge of the
same court for an investigation of the respondent, believing that
Escritor is committing an immoral act that tarnishes the image of


the Court, thereby she should not be allowed to remain employed
therein as it might appear that the court condones her act.

In Escritors defense, she admitted that she had previous
marriage, but her husband was living with another woman, thus,
started living with Luciano Quilapo, a man not her husband, whom
she had a son. However, she invoke her religious freedom, being
a member of Jehovahs Witness, and that her religious affiliation
permitted and recognized her acts after she filed her Declaration
of Pledging Faithfulness on July 28 1991.

Such declaration was in conformity with her religion and
that it allows the members of the congregation, who were left by
there former spouse, to live with new partner without the benefit of
marriage and a judicial declaration of annulment of nullity of
marriage. In short, base on their congregation on Escritors case,
there is nothing immoral about the conjugal arrangement between
Escritor and Quilapio.

Escritor thus contended that, she should not be
administratively liable by invoking her religious belief and practices
under her congregation as pursuant to Section 6 Article II and
Section 5 Art III of the constitution.

Issue: Should the respondent not be administratively liable
pursuant to her religious belief as provided by the constitution
regarding separation of church and the state and free exercise of
religion clause under the constitution?

Held:Yes. The respondents should not be administratively liable
pursuant to the establishment and free exercise clause of the
constitution.

The establishment and free exercise clause has a purpose to
promote freedom of individual religious beliefs and practices, it
prohibits the government from inhibiting religious beliefs with
penalties as well as rewards for religious beliefs and practice. The
separation clause provided by the constitution also established a
wall of separation between state and religious activities.
From all of the theories of standards regarding the free
exercise of religion clause and separation of the church and the
state as applied in American jurisprudence such as the standard
separation (strict separation, strict neutrality and benevolent
neutrality or accommodation, the court held that the latter is
what the Philippines constitution follows. Thereby, the benevolent
neutrality or accommodation approach is the spirit, intent and
framework of the Philippine constitution as far as interpreting
religious clause such as the establishment and free exercise of
religion clause are concern.

Benevolent neutrality theory of standards recognizes the
importance church or religion in the public life and to the state as a
whole. Moreover, the theory also emphasize that the wall of
separation between the church and of the state is meant not
protect the state from the church as invoked by Jeffersonian view,
but in the contrary, it is meant to protect the church from the
pervasive and virtually limitless power of the state. With respect to
governmental actions, accommodations of religion maybe allowed,
not to promote the governments favored form, but to allow
individuals and groups to exercise their religion without hindrance
or burden.

This theory of standards provides protection or exemption of
individuals or groups of individuals from infringement of practicing
religious belief and practices from laws of general application
enforced by the state. Thus, the legislature may accommodate
religious belief and practices when enacting laws of general
application, and if the legislature did not foresee their enacted law
to infringe religious practices, these religions may turn to courts for
protection.

As stated before, the benevolent neutrality approach is
followed in interpreting religion clause under the Philippine
constitution that covers both mandatory and permissive
accommodation of religious beliefs and practices. In the case of
Philippine jurisprudence, the government may require (mandatory)
or allow (permissive) accommodation of religious beliefs and
practices in enforcing laws of general application. Required
(mandatory) accommodations prevents needless injury to the
religious consciences of those who can have influence in the
legislature and allowed (permissive) accommodation extends
this treatment to religious faiths that are less able to protect
themselves in the political arena.

However, even if the Philippine jurisprudence adopts
benevolent neutrality that either allows permissive and
mandatory accommodations in interpreting religion clauses, it
does not automatically mean that the Court can grant
exemptions every time a free exercise claims is invoked. The
SC reiterated that this theory only provides aid in interpreting
such clause but the compelling state interest should be given
primary protection.

In identifying the compelling state interest test, there
are three pertinent steps, 1) won the statute or government
action created a burden on the free exercise of religion.
This question is for the ascertainment of the claimants belief to
avoid the mere claim of religious beliefs to escape mandatory
regulation or in other words, the courts examine the sincerity of
the religious belief being claimed 2) won there is a sufficient
compelling state interest to justify this infringement of
religious liberty. This step requires the government to
establish that its purposes are legitimate for the state and that
they are compelling, such that the state should show how and to
what extent those objectives will undermined if exemptions are
granted. And 3) won the state in achieving its legitimate
purposes used the least intrusive means possible so that
the free exercise is not infringe any more than necessary to
achieve the legitimate goal of the state.

In the case at bar, the court investigates if the respondent
is entitled to exemptions, by subjecting the case to the
benevolent neutrality approach and compelling state interest.

Referring to the first step, there is no doubt in the
sincerity or centrality of the respondents religious belief and that
there is a clear infringement in such religious belief and practice,
such that a government policy and practice inhibits the
respondent in exercising her religious freedom.

On the second step, the government, through the
Solicitor General, invoke that it is the interest and objective of
the state to enforce its laws for proper administration of justice,
as provided in the Revised Administrative and Penal Code
pertaining to liabilities against immoral conduct of public officer
and crimes against chastity. And, it is further invoke by the
government that the state has a primary goal to protect the
sanctity of marriage and inviolable institution of family in the
society as provided under the provisions of the Civil and Family
Code. However, the SC also ruled that even if these goals are
recognized as one of the priority of the state to protect and
uphold, the exercise of religious freedom is a fundamental right
of every person that is regarded as the most inalienable and
sacred of all human rights. In the eyes of the SC, the exercise of
religious freedom is much more compelling and prioritized by
the state as oppose to the priorities contended by the
government. Thus, to satisfy the court not to grant exemptions
to the respondent from exercising her religious freedom, the
government must provide evidence that shows gravest abuse
and immediate endangerment of the paramount interest of the
state. The government thus should precisely show to what
extent these objectives will be undermined if exemptions are to
be granted. In fact, if exemption is to be denied for the
respondent, the government just breaks up an ideal union of two
individuals who manage to live together for 25 years, which
pictures a relationship of harmony and mutual support, thus, it
will only defeat the very substance of marriage and family.
Lastly, there is also an absence of evidence from the
government to show that the means it seeks to achieve its
legitimate state objective is the least intrusive means.

As a general rule stated by the Supreme Court of the
Philippines in cases of involving the exercise of religious
freedom, the court recognizes that state interests must be
upheld in order that freedoms such as religious freedom is
enjoyed. In the area of religious rights as provided in the
establishment and free exercise clause of the Constitution, man
can exercises such freedom as long as such exercise may not


undermine the very fabric of the state, otherwise, if there is an
evidence showing that such practice is performed in gravest
abuse and provides an immediate and compelling endangerment
of state interests, a man is therefore accountable for any liabilities.

Thus, in the case at bar, Escritor is not administratively liable
as charged.

STATE POLICIES
SECTION 7- The state shall pursue and Independent Foreign
Policy. In its relations with other states, the paramount
consideration shall be national sovereignty, territorial integrity,
national interest, and the right to self determination.

What is the consideration for foreign relations?

SECTION 8. The Philippines, consistent with the national
interest, adopts and pursues a policy of freedom from nuclear
weapons in its territory.

a. What is the meaning of the policy?

A policy is a principle or rule to guide decisions and achieve
rational outcomes. A policy is a statement of intent, and is
implemented as a procedure

or protocol. Policies are generally
adopted by the Board of or senior governance body within an
organization whereas procedures or protocols would be developed
and adopted by senior executive officers. Policies can assist in
both subjective and objective decision making. Policies to assist
in subjective decision making would usually assist senior
management with decisions that must consider the relative merits
of a number of factors before making decisions and as a result are
often hard to objectively test e.g. work-life balance policy. In
contrast policies to assist in objective decision making are usually
operational in nature and can be objectively tested e.g. password
policy.

Section 9. The state shall promote a Just and dynamic social
order that will ensure the prosperity and independence of the
nation and free the people from poverty through policies that
provide adequate social services, promote full employment, a
rising standard of living, and an improved quality of life for all.

Section 10. the state shall promote Social Justice in all
phases of national development.

a. Read: Article XIII

ARTICLE XIII
SOCIAL JUSTICE AND HUMAN RIGHTS

Section 1. The Congress shall give highest priority to the
enactment of measures that protect and enhance the right of all
the people to human dignity, reduce social, economic, and political
inequalities, and remove cultural inequities by equitably diffusing
wealth and political power for the common good.
To this end, the State shall regulate the acquisition, ownership,
use, and disposition of property and its increments.
Section 2. The promotion of social justice shall include the
commitment to create economic opportunities based on freedom
of initiative and self-reliance.

Labor
Agrarian and Natural Resources Reform
Urban Land Reform and Housing
Health
Women
Role and Rights of People's Organization
Human Rights

b. Social Justice: Definition & Concept-
Calalang v. Williams, 70 Phil 726;

Facts: Maximo Calalang, filed a petition for writ of prohibition
against the respondents (D. Williams as Chairman of the National
Traffic Commission; Vicente Fragante as Director of Public Works;
Sergio Bayan, as acting Secretary of Public Works and
Communications; Eulogio Rodriguez as Manila City Mayor and
Juan Dominguez as acting Chief of Police in Manila), prohibiting
the implementation of the Commonwealth Act 548.

The said Law, as alleged by the petitioner, authorizing the
concerned public officials to promulgate rules and regulations
and control of the use of and traffic on national roads and
streets, by prohibiting animal driven vehicles from passing and
picking up passengers from the specified roads and streets in
Manila in a given period of time, one year from the date of the
opening of the Colgante Bridge, which is detrimental for the
owners of the animal drawn vehicles and also of the riding
public as well.

From the implementation and enforcement of the Act in
question, the petitioner alleged that it infringes the constitutional
precept regarding the promotion of social justice, which is a vital
goal of the state to insure the well being and economic security
of all the people.

Issue: Does the Act in question violates the promotion of social
justice as provided under Section 10 Article II of the
constitution?

Held: No. Commonwealth Act 548 does not infringe the
constitutional precept regarding the promotion of Social Justice
as provided by the constitution.

Section 9 Article II provides The State shall promote
social justice in all phases of national development. The court,
upon interpretation of the term social justice was construed as
not merely a sympathy towards any specific and given group of
people. Social justice in neither communism, despotism,
atomism nor anarchy, rather, it is the humanization of laws and
equalization of social and economic forces by the State so that
justice in its rational and objectively secular conception may at
least at least e approximated. It should be construed as the
promotion of the welfare of all the people belonging in the socio-
economic strata in the society. This is achieved by adopting an
effective and efficient measure to insure economic stability and
progress, and social equilibrium in the interrelations between
members of the society. The protection and promotion of law
should be extended among all the units of the society,
regardless of class, race, religion, sex or socio-economic class.
Thus, such concept is grounded on the principle of salus populi
est suprema les (the interest of the people is the supreme law)-
greatest good to the greatest number.

In the case at bar, the prohibition of the Act of passing
and picking up of animal driven vehicles along roads and streets
in the City of Manila is not to infringe and or create burden
among owners of animal driven vehicles, rather, to promote safe
transit upon and avoid obstructions on national roads, in the
interest and convenience of the public. It rather aims to
decongest traffic, which is a main concern of the public in terms
of transpirations in rural areas.

Thus, the law in question is guided towards public welfare.

Alalayan v. NPC, 24 SCRA 172;

Facts: The petitioner, Santiago P. Alalayan filed an appeal from
the decision of the CFI Manila, denying the writ for preliminary
injunction against the respondent National Power Corporation.
Alalayan, is one of the 137 franchise holders who have existing
and valid contracts with the respondent, to receive electric
power and energy from it.

The petitioner assails that Section 3 of RA 3043 which
limits the net profit of franchise holders who receive at least
50% of electric energy and power from NPC and empowers the
respondent to renew all existing contracts with franchise holders
for the supply of electric energy and power, deprives franchise
holders, like the petitioner, the liberty to contract without due
process of law.

Issue: Does the law in question infringes the petitioners right of
contract protection of property, such that it regulates the


petitioners net profit and deprives their liberty to contract without
due process of law?

Held: No. The government regulates the petitioners liberty to
contract and rights of property through police power in accordance
to law.

It is true that one of the goals of the state is to protect the
rights, liberty and property of the people. However, the
government also has the right or power to regulate freedoms and
property rights of individuals for the protection of public safety,
health and welfare and the promotions of public convenience and
general prosperity through the fundamental power of police power.
Persons and property maybe subjected to all kinds of restraints
and burdens, in order to secure the general comfort health and
prosperity of the state. This is one of the few means of the
government to promote social justice as guaranteed by the
constitution.

In the case at bar, the government, through the law in
question limits the franchise holders net profit to 12 percent and
that the National Power Corporation, a government owned
company, has the authority to renew its existing contracts with
franchise holders. In doing so, the government regulates the
liberty and property enjoyed by business enterprises, in order to
satisfy the needs of public welfare, to protect those who suffer
from poverty and cannot afford their basic needs and to promote a
welfare state, which is one of the primary goals of the state.

Thereby, all these ends are government thrust for the
promotion of social justice. The law in question does not intend to
infringe but to the liberty to contract and rights of property among
electric energy and power franchise holders, and seeks to protect
the welfare of the consumers or the general public, against electric
companies.

Tuazon v. Land Tenure, 31 SCRA 413

FACTS: R.A. 2616 authorized expropriation of the Tatalon Estate
in Quezon City owned by petitioner and 2 others. Lands were to
be divided to lots to be sold. They prayed that it be declared
unconstitutional because violative of equal protection clause since
statute applies only to Tatalon estate.
HELD: No person shall be denied equal protection. A judicial
being is included within its terms. Those adversely affected may
under such circumstances invoke the equal protection clause only
if they can show that the governmental act assailed was prompted
by the spirit of hostility, or at the very least discrimination that finds
no support in reason. Petitioner failed to prove denial of equal
protection. Occupants believe in gf that veterans subdivision is the
real owner. Only when the place vastly improved with building of
roads, infrastructure did petitioner claimed for the first time that
they are the owners.

c. Social justice, tempered-

Astudillo v. Board, 73 SCRA 15 [1976]

Facts: The respondent, Peregrina Astudillo, filed an appeal from
the resolution made by the CFI Quezon City, denying her petition
to nullify the awarding of piece of land located in East Avenue
Subdivison of the Peoples Homesite and Housing Corporation to
Ramon Mitra and to re-award the land in question to her.

The petitioner claims that she has the possession of the land
in question where she constructed her residential house and
admits that she also squatting in the same area from 1957 up to
the present. Ramon Mitra, however, is the titled and valid owner of
the land in question since Feb 18, 1965.

Thus, in line with the appeal of the petitioner, she prayed for
the nullification of the awarding of the land in question to Mitra and
re award the land to her.

Issue: Should the PHHC award the land to Astudillo instead of
awarding it to the titled owner, Mitra, pursuant to the states
promotion of social justice?

Held: No. The state cannot award the land in question to the
petitioner just to satisfy its goal to promote social justice.

While it may be true that the states goal is to promote
social justice and to maintain adequate social services in the
field of housing, however, the States consideration to the poor
and destitute does not tolerate usurpations of property whether
public and private. "In carrying out its social readjustment*
policies, the government cannot just always favor usurpers,
squatters and intruders who does not give considerations and
high regards to the nature and character of their occupancy,
because in doing otherwise, the government cannot attain its
goal of proper administration since it will just perpetuate conflict.

In the case at bar, it is admitted that the petitioner is a
squatter but the titled and valid owner of the land in question is
Mitra, therefore, having no possessory right and not being a
bona fide resident of the land in question, the petitioner cannot
just force the government to grant the land to her and cancel the
awarding of the same to Mitra. The awarding of land to Mitra
therefore wont be prejudicial to Astudillos rights because in the
eyes of the law, she is not principally or subsidiarily bound in the
contract of sale between Mitra and the PHHC.

In the promotion of social justice to protect the public
safety, prosperity and welfare, all actions by the government
should always be in accordance to law, in cancellation of the
awarding of lot to Mitra and granting the land to Astudillo would
otherwise be contrary to this goal, it wont met just solution.
Thus, the petition is dismissed, judgment from CFI QC affirmed.

Salonga v. Farrales, 105 SCRA 359, 369[1981]

Facts: The defendant, Julita B. Farrales is the titled owner of a
residential lot in Sta. Rita, Olongapo City. Within the owned
parcel of land by the defendant, the plaintiff, spouses Salonga,
are the lessees of the 156 sq. meters of land where the latter
erected their house and is paying rentals to the defendant.

Sometime before 1968, the plaintiff failed to pay rental and
that as a result, the defendant filed an ejectment case against
for non-payment of rentals against the plaintiff. Thus, the
defendant forced the plaintiff. The plaintiff then offered that they
will just buy their occupied parcel of land instead of vacating the
land and the house of strong materials, however, despite of the
insistence of the plaintiff, the titled owner defendant, refused to
accept the offer, thus there is no contract of sale or sell in the
aforesaid land was realized.

The plaintiff then, after a strict refusal from the defendant-
owner to sell her land, filed for petition for relief. The case was
heard and elevated until the CA, praying for ordering to the
defendant to sell her parcel of land where the house of the
plaintiff was erected and that the plaintiff invoke their right to be
subjected under section 6 (9) Article of the constitution, referring
to the application of social justice, which they contended that it
delimits and regulated property rights and private gains.

Issue: Was the contention of the plaintiff correct such that by
invoking for the promotion of social justice, as provided under
Section 6 (9) Article II of the Constitution, they could gain their
contention for a relief and force the defendant to sell her land?

Held: No. The concept of social justice cannot be invoked by
the plaintiff, just gain relief and force the defendant to sell her
land where the plaintiffs house was erected.

Social justice is said to be for the promotion of economic
development and proper equilibrium between the relationships
of all units of the society. It extends among all of the people in
the state regardless of SEC. However, social justice cannot be
invoked to trample on the rights of property owners who under
our Constitution and laws are also entitled for protection. Social
justice is not intended to take away rights from a person and
give them to another who is not entitled thereto.

In the case at bar, the plaintiff cannot force the defendant
to sell her title by invoking equity and justice, rather, the


plaintiffs may remove the improvements should the lessor refuse
to reimburse, buy the lessee do not have the right to by the land.
The right of property of the defendant over her owned land cannot
be simply override by invoking social justice, since the right of
property is also protected by the State. Moreover, by granting a
relief to the plaintiff because of social justice, it will only defeat the
purpose of this concept of salus populi est suprema lex, since the
ends are only for the benefit of the few (plaintiff).

Thus, Judgment affirmed in favor of the defendant.

d. Compare the Social Justice provisions under the 1935, 1973,
1987 Constitutions

Social justice was construed as not merely sympathy
towards any specific and given group of people. Social justice in
neither communism, despotism, atomism nor anarchy, rather, it is
the humanization of laws and equalization of social and economic
forces by the State so that justice in its rational and objectively
secular conception may at least at least be approximated.
(Calalang Case)

Comparison of Social justice from the 1935, 1973 and 1985
Constitution:

In 1935 Constitution:
Social justice is justice to the common-tao: its relatives,
farmers, children spouse and others. Those who have less in life
should have more in law.

In 1973 Constitution:
The interpretation of social justice is that it the law should
favor those who are underprivileged. The economic factor then
sets in such that, the state have the authority to regulate acquire
the ownership of the property for its equitable diffusion in the
society.

Thus, social justice involves proper administration and
regulation of property for the greater number. A concept of
economic inequities

In the 1987 Constitution:
Covers all phases of national development with emphasis
on socio-economic, cultural and political equity.

The meaning of self-reliant and independent economy (section 19,
Art II)
The constitution does not envision a hermit-type of state nor
isolationist policy of economic development. It pursues the
protection of its local market in the same time, opens the door for
foreign markets provided that the Philippines will benefit from fair
competition (Tanada vs. Angara Case)

Section 11. The state values the dignity of every human
person and guarantees full respect for human rights.

READ: Article XIII, Sec. 17-19

ARTICLE XIII
HUMAN RIGHTS
Section 17. (1) There is hereby created an independent office
called the Commission on Human Rights.
(2) The Commission shall be composed of a Chairman and four
Members who must be natural-born citizens of the Philippines and
a majority of whom shall be members of the Bar. The term of
office and other qualifications and disabilities of the Members of
the Commission shall be provided by law.
(3) Until this Commission is constituted, the existing Presidential
Committee on Human Rights shall continue to exercise its present
functions and powers.
(4) The approved annual appropriations of the Commission shall
be automatically and regularly released.
Section 18. The Commission on Human Rights shall have the
following powers and functions:
(1) Investigate, on its own or on complaint by any party, all
forms of human rights violations involving civil and political
rights;
(2) Adopt its operational guidelines and rules of procedure, and
cite for contempt for violations thereof in accordance with the
Rules of Court;
(3) Provide appropriate legal measures for the protection of
human rights of all persons within the Philippines, as well as
Filipinos residing abroad, and provide for preventive measures
and legal aid services to the under-privileged whose human
rights have been violated or need protection;
(4) Exercise visitorial powers over jails, prisons, or detention
facilities;
(5) Establish a continuing program of research, education, and
information to enhance respect for the primacy of human rights;
(6) Recommend to Congress effective measures to promote
human rights and to provide for compensation to victims of
violations of human rights, or their families;
(7) Monitor the Philippine Government's compliance with
international treaty obligations on human rights;
(8) Grant immunity from prosecution to any person whose
testimony or whose possession of documents or other evidence
is necessary or convenient to determine the truth in any
investigation conducted by it or under its authority;
(9) Request the assistance of any department, bureau, office, or
agency in the performance of its functions;
(10) Appoint its officers and employees in accordance with law;
and
(11) Perform such other duties and functions as may be
provided by law.
Section 19. The Congress may provide for other cases of
violations of human rights that should fall within the authority of
the Commission, taking into account its recommendations.


Section 12. The state recognizes the sanctity of family life
and shall protect and strengthen the family as a basic social
institution. it shall equally protect the life of the mother and the
life of the unborn from conception. The natural and primary
right and duty of parents in the rearing of the youth for civic
efficiency and the development of moral character shall receive
the support of the government.

a. READ: Article XV

ARTICLE XV
THE FAMILY

Section 1. The State recognizes the Filipino family as the
foundation of the nation. Accordingly, it shall strengthen its
solidarity and actively promote its total development.
Section 2. Marriage, as an inviolable social institution, is the
foundation of the family and shall be protected by the State.
Section 3. The State shall defend:
(1) The right of spouses to found a family in
accordance with their religious convictions and the
demands of responsible parenthood;
(2) The right of children to assistance, including proper
care and nutrition, and special protection from all forms
of neglect, abuse, cruelty, exploitation and other
conditions prejudicial to their development;
(3) The right of the family to a family living wage and
income; and
(4) The right of families or family associations to
participate in the planning and implementation of
policies and programs that affect them.


Section 4. The family has the duty to care for its elderly members
but the State may also do so through just programs of social
security.

b. The family
Cabanas v. Pilapil, supra

c. The unborn-limitation to abortion to avoid application of Roe v.
Wade, 4410 US 113 [1973]

Section 12, Art. II prohibits all forms of abortion except
therapeutic abortion or when the life of the mother is in danger.
(Note: In the United States, abortion is allowed but only up to the
2
nd
trimester of the pregnancy [ROE vs. WADE]).

d. Rearing of the youth-as primary and natural right and duty of
parents
Meyer v. Nebraska, 260 US 260 [1922]

May the State prohibit the teaching of a particular language in any
school?

No as held in MEYER VS. NEBRASKA, 260 US 260
(1922) because the child is not a mere creature of the State and
the parents have the natural right and duty of rearing their children
for civic efficiency.

Pierce v. Society of Sisters, 268 US 510 [1925]

May the State require parents to enroll their small children only to
public schools valid?

As held in PIERCE VS. SOCIETY OF SISTERS, 268 US
510 (1925), a law requiring small kids to be enrolled in public
schools only is unconstitutional since it interferes with the right of
parents in rearing their children. They have the right to choose
which school is best suited for the development of their children
without interference from the State. THIS IS SO BECAUSE THE
CHILDREN ARE NOT MERE CREATURES OF THE STATE.

Ginsberg v. New York, 390 US 629 [1968]

Is a law prohibiting the sale of girlie(bold) magazines to minors
violates the right of parents in rearing their children for civic
efficiency?

No, as held in the case of GINSBERG VS. NEW YORK,
390 US 629 (1969), a law prohibiting the sale of girlie magazines
[bold?) is constitutional and does not violate the above provision.
This is so because parents could buy said magazines for their
children if they believe the same is already suitable to the
understanding of their child. This is in accordance with this
provision which states that the parents have the natural and
primary right in rearing their child for civic efficiency

Wisconsin v. Yoder, 106 US 205 [1972]

Citation. 22 Ill.406 U.S. 205, 92 S. Ct. 1526, 32 L. Ed. 2d 15
(1972)

Facts: The Respondents refused to send their children to school
after they completed the eighth grade, in conformance with their
religious practices. The Petitioner, the State of Wisconsin
(Petitioner), brought an action seeking to enforce its compulsory
education law. The Respondents were convicted in violation of the
law. The Supreme Court of Wisconsin reversed, sustaining the
Respondents argument that their actions fell under the Free
Exercise Clause of the First Amendment of the United States
Constitution (Constitution). Wisconsin appealed, and the Supreme
Court of the United States (Supreme Court) granted certiorari.

Issue. This case considers whether members of a religious
community can be compelled to follow a compulsory education
scheme, which could be detrimental to their own religious
teachings.

Brief Fact Summary. The Respondents, Yoder and other
members of a Wisconsin Amish community (Respondents) took
issue with the States compulsory education law, maintaining that
keeping children in school until the age of sixteen was against
their religious principals, in violation of the Free Exercise
Clause.

Synopsis of Rule of Law. When a true religious interest exists,
a state cannot enforce a law, which abrogates that interest,
provided the public interest in enforcing the law is not otherwise
burdened.

Angeles v. Judge Sison, 112 SCRA 26 [1982]

Facts:
The plaintiff, Jose Angeles is a faculty under the FEU
Institute of Technology. Sometime in 1975, the respondent
Eduardo Picar and Wilfredo Patawaran, both students of the
said university, mauled the petitioner outside the campus out of
contempt and grudge.

Angeles, requested an investigation and administrative
complaint through the Dean of the said Institute, Gilberto
Mercado, subjecting the students to disciplinary actions
provided under the University Code of Conduct Manual.

The respondents, then filed a petition praying for a
preliminary injunction before Judge Rafael Sison of CFI Manila,
contending that the administrative complaint filed and heard
under the investigative body instituted by the University has no
authority to do so since, the alleged incidence was committed
within the territorial jurisdiction of the university and was beyond
school hours.

The respondent judge then concurred with the
respondents contentions, stating that the incidence should be
left to the police and judicial authorities, and therefore issued a
preliminary injunction, halting the continuance of the
investigative body.

The plaintiff then filed a petition for a review of certiorari
from the decision of the respondent judge.

Issue:
Does the University has the authority to conduct an
administrative complaint from incidence in the case at bar?

Held: Yes.
The university has the authority to conduct an investigation
in the case at bar even if the alleged actions of the respondents
was committed outside the school hours and its territorial
jurisdiction.

The state, as provided under section 12 article II of the
constitution has the responsibility for rearing the youth for civic
efficiency and their development of moral character. And one of
the agencies of the government to implement such responsibility
is the educational institution, regardless of its proprietary nature.

The SC concurred with the statement of the respondent
judge that as a general rule, the authority of the school is co-
extensive with its territorial jurisdiction and that any incidences
committed outside of the school premises should be left for the
police and judicial authorities. However, such rule is not
absolute.

There are two exemptions to the rule, such that the school
has the authority to review or subject in accordance to its school
regulations and policies, the students and acts of its students
committed outside the school premises 1)in cases of violations
or rules and regulations in connection of a school-sponsored
activities conducted off-campus and 2)in cases where the
misconduct of the student involves his status as a student or
affects the good name or reputation of the school.

In the case at bar, the alleged mauling of the two
respondent students to the plaintiff directly affects the reputation
of the university. And that, as an educational institution, the
school has the right to subject the students to disciplinary action
in accordance to its policies as part of the states responsibility
to discipline and rear and develop the character and civic
efficiency the youth.



Thus, the investigative body conducted by the university
should continue against the respondent student as a student of
the university as well as the criminal complaint in courts of justice
as a citizen of the state.

Section 13. the state recognizes the Vital role of the youth in
nation building, and shall promote and protect their physical,
moral, spiritual, intellectual well being. it shall inculcate in the
youth patriotism and nationalism, and encourage in public and
civic affairs.

CASE:PEOPLE VS RITTER, 194 SCRA 690

RULING: Ritter was prosecuted for rape with homicide and not
pedophilia, assuming this is a crime by itself. Pedophilia is clearly
a behavior offensive to public morals and violative of the declared
policy of the state to promote and protect the physical, moral,
spiritual and social well-being of our youth. (Article II, Section 13,
1987 Constitution) (Harvey v. Defensor Santiago, 162 SCRA 840,
848 [1989]). Pedophiles, especially thrill seeking aliens have no
place in our country.

In this case, there is reasonable ground to believe that the
appellant committed acts injurious not only to Rosario Baluyot but
also to the public good and domestic tranquility of the people. The
state has expressly committed itself to defend the right of children
to assistance and special protection from all forms of neglect,
abuse, cruelty, exploitation and other conditions prejudicial to their
development. (Art. XV, Section 3 [2] (Harvey v. Santiago, supra).
The appellant has abused Filipino children, enticing them with
money. The appellant should be expelled from the country.

..And finally, the Court deplores the lack of criminal laws
which will adequately protect street children from exploitation by
pedophiles, pimps, and, perhaps, their own parents or guardians
who profit from the sale of young bodies. The provisions on
statutory rape and other related offenses were never intended for
the relatively recent influx of pedophiles taking advantage of
rampant poverty among the forgotten segments of our society.
Newspaper and magazine articles, media exposes, college
dissertations, and other studies deal at length with this serious
social problem but pedophiles like the appellant will continue to
enter the Philippines and foreign publications catering to them will
continue to advertise the availability of Filipino street children
unless the Government acts and acts soon. We have to acquit the
appellant because the Bill of Rights commands us to do so. We,
however, express the Court's concern about the problem of street
children and the evils committed against them. Something must be
done about it.


Section 14.the state recognizes the role of women in nation-
building, and shall ensure the fundamental equality before the law
of women and men.

See: Sec. 14 of Article XIII

SOCIAL JUSTICE AND HUMAN RIGHTS: WOMAN

Section 14. The State shall protect working women by providing
safe and healthful working conditions, taking into account their
maternal functions, and such facilities and opportunities that will
enhance their welfare and enable them to realize their full potential
in the service of the nation.

Section 15. the state shall protect and promote the right to health
of the people and instill health consciousness among them.

See: Article XIII, Secs. 11-13 (social justice provisions)


SOCIAL JUSTICE AND HUMAN RIGHTS: HEALTH

Section 11. The State shall adopt an integrated and
comprehensive approach to health development which shall
endeavor to make essential goods, health and other social
services available to all the people at affordable cost. There
shall be priority for the needs of the under-privileged, sick,
elderly, disabled, women, and children. The State shall
endeavor to provide free medical care to paupers.

Section 12. The State shall establish and maintain an effective
food and drug regulatory system and undertake appropriate
health, manpower development, and research, responsive to
the country's health needs and problems.

Section 13. The State shall establish a special agency for
disabled person for their rehabilitation, self-development, and
self-reliance, and their integration into the mainstream of
society.

Construction of high voltage electric transmission lines may
be enjoined - HERNANDEZ et al., v. NPC, GR. 145328,
3/23/06

CASE: HERNANDEZ vs NPC

FACTS: In 1996, the National Power Corporation (NPC)
began building towers to support high tension cables for a
power transmission project. The transmission lines ran
through the Dasmarinas Village, where the Petitioners lived.

The Petitioners, fearing the health effects of the towers,
discovered studies linking high rates of cancer and other
illnesses with exposure to electromagnetic fields. They
discussed their concerns with NPC over a series of meetings.
The two parties tried to come to a compromise, but
negotiations stalled.

The petitioners filed a Complaint for Damages, seeking a
temporary restraining order and preliminary injunction against
NPC. The Petitioners alleged that exposure to
electromagnetic radiation from the transmission lines posed a
risk of health hazards to local residents, and sought damages
and the relocation of the transmission lines.

The Trial Court issued a temporary restraining order
preventing NPC from energizing and transmitting high voltage
current through the lines. NPC challenged this order on the
basis that the Presidential Decree No. 1818 prevented courts
from issuing temporary restraining orders or injunctions. In the
interim, the Trial Court issued a writ of a preliminary injunction
against NPC, saying it was necessary because the power
lines posed possible health risks to the Petitioners. It also
ruled that Presidential Decree No. 1818 did not apply to the
case because of these health risks.
In 2000, the Court of Appeals reversed the Trial Courts order,
finding that Presidential Decree No. 1818 did apply to the
case. The Petitioners filed a further appeal to the Supreme
Court, claiming that Presidential Decree No. 1818 should not
be construed to apply to cases of extreme urgency as in the
present case when no less than the rights of the petitioners to
health and safety hangs on the balance. The Petitioners also
sought a preliminary injunction on the ground that the NCP
Project impinged on their right to health as enshrined in
Article II, Section 15 of the Constitution.

HELD: The Court found that Presidential Decree no. 1818 did
not apply to the case at bar, and reinstated the preliminary
injunction.

It considered first that there had been a long string of cases
establishing that the Decree applied only to cases involving
facts or the exercise of discretion, rather than controversies
involving questions of law. Because the Petitioners were
arguing that the power transmission projct impinged on their
constitutional right to health, and further, that the government
had failed to consult them as an affected community prior to
the project, questions of law were at issue in this case. As
such, Presidential Decree No. 1818 was inapplicable here.

Having decided this, the Court considered there was good
reason to award a writ of preliminary injunction, because
there was ample evidence that the power transmission project
would endanger the health and lives of the Petitioners. The
Court cited studies linking the presence of power lines to


diseases such as cancer, as well as Congressional debates and
the NPCs own guidelines on power line safety. The Court also
considered that the area the project was located in was subject
to earthquake faults and extreme weather, and that the NPC
had taken shortcuts with its consultation requirements, as well
as with its feasibility studies.

The Court acknowledged that the question of whether the power
lines were safe was one to be decided in the merits
proceedings. However, taken together the above factors
suggested that there was enough probability of imperiling the
Petitioners health and life that it would be prudent to preserve
the status quo. As such, the Court considered that the trial court
had been justified in issuing the injunction.

Section 16. The state shall protect and advance the right of the
people to a balanced and healthful ecology in accord with the
rhythm and harmony of nature.

An enforceable right- Oposa v. Factoran, 7/30/93 which can be
enforced by Mandamus(MMDA v. Concerned Residents of
Manila Bay, 643 SCRA 90 [2011]

CASE: OPOSA VS FACTORAN

FACTS: This case is unique in that it is a class suit brought by 44
children, through their parents, claiming that they bring the case in
the name of their generation as well as those generations yet
unborn. Aiming to stop deforestation, it was filed against the
Secretary of the Department of Environment and Natural
Resources, seeking to have him cancel all the timber license
agreements (TLAs) in the country and to cease and desist from
accepting and approving more timber license agreements. The
children invoked their right to a balanced and healthful ecology
and to protection by the State in its capacity as parens patriae.
The petitioners claimed that the DENR Secretary's refusal to
cancel the TLAs and to stop issuing them was "contrary to the
highest law of humankind-- the natural law-- and violative of
plaintiffs' right to self-preservation and perpetuation." The case
was dismissed in the lower court, invoking the law on non-
impairment of contracts, so it was brought to the Supreme
Court on certiorari.

ISSUE: Did the children have the legal standing to file the case?

HELD: Yes. The Supreme Court in granting the petition ruled that
the children had the legal standing to file the case based on the
concept of intergenerational responsibility. Their right to a
healthy environment carried with it an obligation to preserve that
environment for the succeeding generations. In this, the Court
recognized legal standing to sue on behalf of future generations.
Also, the Court said, the law on non-impairment of contracts must
give way to the exercise of the police power of the state in the
interest of public welfare.

CASE: MMDA vs CONCERNED CITIZENS OF MANILA BAY

FACTS: On January 29, 1999, respondents Concerned Residents
of ManilaBay filed a complaint before the
RTC in Imus, Cavite against several government agencies, among
them the petitioners, for the cleanup, rehabilitation, and protection
of the Manila Bay, and to submit to the RTC a concerted concrete
plan of action for the purpose. The complaint alleged that the
water quality of the Manila Bay had fallen way below the allowable
standards set by law, which was confirmed by DENRs
Water Quality Management Chief, Renato T. Cruz that water
samples collected from different beaches around the Manila Bay
showed that the amount of fecal coliform contentranged from
50,000 to 80,000 most probable number (MPN)/ml which is
beyond the standard 200 MPN/100ml or the SB level under DENR
Administrative Order No.
3490.The reckless, wholesale, accumulated and ongoing acts of o
mission or commission [of the defendants] resulting in the clear
and present danger to public health and in the depletion and
contamination of the marine life of Manila Bay, the RTC held
petitioners liable and ordered to clean up and rehabilitate Manila
Bay and to restore its water quality to class B waters fit for
swimming, skin-diving, and other forms of contact recreation.

Herein petitioners appealed before the Court of Appeals
contending that the pertinent provisions of the Environment
Code (PD 1152) relate only to the cleaning of specific pollution
incidents and do not cover cleaning in general. They also
asserted that
he cleaning of the Manila Bay is not a ministerial act which can
be compelled by mandamus. The CA sustained RTCs decision
stressing that petitioners were not required to do tasks outside
of their basic functions under existing laws, hence, this appeal.

ISSUE: WON Petitioners can be compelled by mandamus to
clean up and rehabilitate Manila Bay?

HELD: YES. Supreme Court held that the cleaning up and
rehabilitating Manila Bay is a ministerial in nature and can be
compelled by mandamus. Sec. 3(c) of R.A. No. 7924 (the law
creating MMDA) states that the MMDA is mandated to put up an
adequate and appropriate sanitary landfill and solid waste and
liquid disposal as well as other alternative garbage disposal
systems. SC also noted that MMDAs duty in the area of solid
waste disposal is set forth not only in the Environment Code
(PD 1152) and RA 9003, but also in its charter, therefore, it is
ministerial in nature and can be compelled by mandamus

Section 17.The state shall give priority to education, science
and technology, arts, culture, and sports to foster patriotism and
nationalism, accelerate social progress, and promote total
human liberation and development.

read also Article XIV: EDUCATION, SCIENCE AND
TECHNOLOGY, ARTS, CULTURE AND SPORTS.

Section 18. The state affirms labor as primary social economic
force. it shall protect the rights of workers and promote their
welfare.

See: Sec. 3 Article XIII

SOCIAL JUSTICE AND HUMAN RIGHTS: LABOR

Section 3. The State shall afford full protection to labor, local and
overseas, organized and unorganized, and promote full
employment and equality of employment opportunities for all.

It shall guarantee the rights of all workers to self-organization,
collective bargaining and negotiations, and peaceful concerted
activities, including the right to strike in accordance with law. They
shall be entitled to security of tenure, humane conditions of work,
and a living wage. They shall also participate in policy and
decision-making processes affecting their rights and benefits as
may be provided by law.

The State shall promote the principle of shared responsibility
between workers and employers and the preferential use of
voluntary modes in settling disputes, including conciliation, and
shall enforce their mutual compliance therewith to foster industrial
peace.

The State shall regulate the relations between workers and
employers, recognizing the right of labor to its just share in the
fruits of production and the right of enterprises to reasonable
returns to investments, and to expansion and growth.

Section 19.The state shall develop a self-reliant and
independent national economy effectively controlled by
Filipinos.

READ: Article XII-NATIONAL ECONOMY AND PATRIMONY

b. Meaning of self reliant and independent: not in isolation-
Tanada v. Angara, 5/2/97

CASE: TANADA vs ANGARA

FACTS: Petitioners Senators Taada, et al. questioned the
constitutionality of the concurrence by the Philippine Senate of
the Presidents ratification of the international Agreement
establishing the World Trade Organization (WTO). They argued


that the WTO Agreement violates the mandate of the 1987
Constitution to develop a self-reliant and independent national
economy effectively controlled by Filipinos . . . (to) give preference
to qualified Filipinos (and to) promote the preferential use of
Filipino labor, domestic materials and locally produced goods.
Further, they contended that the national treatment and parity
provisions of the WTO Agreement place nationals and products
of member countries on the same footing as Filipinos and local
products, in contravention of the Filipino First policy of our
Constitution, and render meaningless the phrase effectively
controlled by Filipinos.

ISSUE: Does the 1987 Constitution prohibit our country from
participating in worldwide trade liberalization and economic
globalization and from integrating into a global economy that is
liberalized, deregulated and privatized?

HELD:[The Court DISMISSED the petition. It sustained the
concurrence of the Philippine Senate of the Presidents ratification
of the Agreement establishing the WTO.]

NO, the 1987 Constitution DOES NOT prohibit our country from
participating in worldwide trade liberalization and economic
globalization and from integrating into a global economy that is
liberalized, deregulated and privatized.

There are enough balancing provisions in the Constitution to allow
the Senate to ratify the Philippine concurrence in the WTO
Agreement.

[W]hile the Constitution indeed mandates a bias in favor of Filipino
goods, services, labor and enterprises, at the same time, it
recognizes the need for business exchange with the rest of the
world on the bases of equality and reciprocity and limits protection
of Filipino enterprises only against foreign competition and trade
practices that are unfair. In other words, the Constitution did not
intend to pursue an isolationist policy. It did not shut out foreign
investments, goods and services in the development of the
Philippine economy. While the Constitution does not encourage
the unlimited entry of foreign goods, services and investments into
the country, it does not prohibit them either.In fact, it allows an
exchange on the basis of equality and reciprocity, frowning only on
foreign competition that is unfair.

xxx xxx xxx

[T]he constitutional policy of a self-reliant and independent
national economy does not necessarily rule out the entry of
foreign investments, goods and services. It contemplates neither
economic seclusion nor mendicancy in the international
community. As explained by Constitutional Commissioner
Bernardo Villegas, sponsor of this constitutional policy:
Economic self-reliance is a primary objective of a developing
country that is keenly aware of overdependence on external
assistance for even its most basic needs. It does not mean
autarky or economic seclusion; rather, it means avoiding
mendicancy in the international community. Independence refers
to the freedom from undue foreign control of the national
economy, especially in such strategic industries as in the
development of natural resources and public utilities.

The WTO reliance on most favored nation, national treatment,
and trade without discrimination cannot be struck down as
unconstitutional as in fact they are rules of equality and reciprocity
that apply to all WTO members. Aside from envisioning a trade
policy based on equality and reciprocity, the fundamental law
encourages industries that are competitive in both domestic and
foreign markets, thereby demonstrating a clear policy against a
sheltered domestic trade environment, but one in favor of the
gradual development of robust industries that can compete with
the best in the foreign markets. Indeed, Filipino managers and
Filipino enterprises have shown capability and tenacity to compete
internationally. And given a free trade environment, Filipino
entrepreneurs and managers in Hongkong have demonstrated the
Filipino capacity to grow and to prosper against the best offered
under a policy of laissez faire.

xxx xxx xxx

It is true, as alleged by petitioners, that broad constitutional
principles require the State to develop an independent national
economy effectively controlled by Filipinos; and to protect and/or
prefer Filipino labor, products, domestic materials and locally
produced goods. But it is equally true that such principles
while serving as judicial and legislative guides are not in
themselves sources of causes of action. Moreover, there are
other equally fundamental constitutional principles relied upon
by the Senate which mandate the pursuit of a trade policy that
serves the general welfare and utilizes all forms and
arrangements of exchange on the basis of equality and
reciprocity and the promotion of industries which are
competitive in both domestic and foreign markets, thereby
justifying its acceptance of said treaty. So too, the alleged
impairment of sovereignty in the exercise of legislative and
judicial powers is balanced by the adoption of the generally
accepted principles of international law as part of the law of the
land and the adherence of the Constitution to the policy of
cooperation and amity with all nations.

That the Senate, after deliberation and voting, voluntarily and
overwhelmingly gave its consent to the WTO Agreement
thereby making it a part of the law of the land is a legitimate
exercise of its sovereign duty and power. We find no patent
and gross arbitrariness or despotism by reason of passion or
personal hostility in such exercise. It is not impossible to
surmise that this Court, or at least some of its members, may
even agree with petitioners that it is more advantageous to the
national interest to strike down Senate Resolution No. 97. But
that is not a legal reason to attribute grave abuse of discretion to
the Senate and to nullify its decision. To do so would constitute
grave abuse in the exercise of our own judicial power and duty.
Ineludibly, what the Senate did was a valid exercise of its
authority. As to whether such exercise was wise, beneficial or
viable is outside the realm of judicial inquiry and review. That is
a matter between the elected policy makers and the people. As
to whether the nation should join the worldwide march toward
trade liberalization and economic globalization is a matter that
our people should determine in electing their policy
makers. After all, the WTO Agreement allows withdrawal of
membership, should this be the political desire of a member.


Section 20.The state recognizes the indispensable role of the
private sector, encourages private enterprise, and provides
incentives to needed investments.

LET ALONE POLICY

CASE: MARINE RADIO vs REYES 11/6/90 (on role of private
sector)

FACTS: The petitioners are self-described "Filipino
enterpreneurs deeply involved in the business of marine radio
communications in the country.
1
They are also operators of
"shore-to-ship and ship-to-shore public marine coastal radio
stations,
2
and are holders of certificates of public convenience
duly issued by the National Telecommunications Commission.
Among other things, they handle correspondence between
vessel passengers or crew and the public.
Sometime in July, 1988, the Department of Transportation and
Communications unveiled an P880-million maritime coastal
communications system project, designed to "ensure safety of
lives at sea (SOLAS) through the establishment of efficient
communication facilities between coast stations and ship
stations and the improvement of safety in navigational routes at
sea."
4
It was set out to provide, among other things, ship-to-
shore and shore-to-ship public corresponding, free of charge.
On August 1, 1988, Atty. F. Reyes Cabigao, in his capacity as
counsel for the petitioner, Marine Radio Communications
Association of the Philippines, Inc., addressed an appeal to then
Secretary Rainerio Reyes, in the tenor as follows:

xxx xxx xxx
But you undoubtedly would understand their fears. It was their
feeling that entry of the government into their line of business
would certainly spell for them financial ruin as it would put into
serious doubt the viability of the entire marine radio
communications industry. They say that, as it is today, the


industry is not viable enough. What more, they ask, if the
government steps in and eventually dips its strong fingers into the
pie?


The petitioners hold that the Department can not compete in the
business of public correspondence, and rely on the provisions of

Section 20, of Article II, of the Constitution, which states:
Sec. 20. The State recognizes the indispensable role of the private
sector, encourages private enterprise, and provides incentives to
needed investments.

The Solicitor General, on the other hand, submits that in spite of
the above provision, the Government "cannot abandon its
ministerial functions of rendering public services to the citizenry
which private capital would not ordinarily undertake, or which by
its very nature is better equipped to administer for the public
welfare than by any private individual or entity.

HELD: The petitioners can not legitimately rely on the provisions
of Section 20, of Article II, of the Constitution, to defeat the act
complained of. The mandate "recogni[zing] the indispensable role
of the private sector" is no more than an acknowledgment of the
importance of private initiative in building the nation. However, it is
not a call for official abdication of duty to citizenry.

The novel provisions of the Charter prescribing private sector
participation, especially in the field of economic activity,
13
come,
indeed, no more as responses to State monopoly of economic
forces which has unfairly kept individual initiative from the
economic processes and has held back competitiveness in the
market. The Constitution does not bar, however, the Government
from undertaking its own initiatives, especially in the domain of
public service, and neither does it repudiate its primacy as chief
economic caretaker of the nation.

The principle of laissez faire has long been denied validity in this
jurisdiction. In 1969, the Court promulgatedAgricultural Credit and
Cooperative Financing Administration v. Confederation of Unions
in Government Corporations and offices,
14
where it was held:

... The areas which used to be left to private enterprise and
initiative and which the government was called upon to enter
optionally and only because it was better equipped to administer
for the public welfare than in any private individual or group of
individuals," continue to lose their well-defined boundaries and to
be absorbed within activities that the government must undertake
in its sovereign capacity if it is to meet the increasing social
challenges of the times. Here as almost everywhere else the
tendency is undoubtedly towards a greater socialization of
economic forces. Here of course this development was
envisioned, indeed adopted as a national policy, by the
Constitution itself in its declaration of principle concerning the
promotion of social justice.
15

The requirements of social justice and the necessity for a
redistribution of the national wealth and economic opportunity find
in fact a greater emphasis in the 1987 Constitution,
notwithstanding the novel concepts inscribed there.
16
And two
decades after this Court wrote it, ACCFAs message remains the
same and its lesson holds true as ever.


Section 21. The state shall promote comprehensive rural
development and agrarian reform.

READ: Sec. 4-10 of Article XIII

AGRARIAN AND NATURAL RESOURCES REFORM

Section 4. The State shall, by law, undertake an agrarian reform
program founded on the right of farmers and regular farmworkers
who are landless, to own directly or collectively the lands they till
or, in the case of other farmworkers, to receive a just share of the
fruits thereof. To this end, the State shall encourage and
undertake the just distribution of all agricultural lands, subject to
such priorities and reasonable retention limits as the Congress
may prescribe, taking into account ecological, developmental, or
equity considerations, and subject to the payment of just
compensation. In determining retention limits, the State shall
respect the right of small landowners. The State shall further
provide incentives for voluntary land-sharing.

Section 5. The State shall recognize the right of farmers,
farmworkers, and landowners, as well as cooperatives, and
other independent farmers' organizations to participate in the
planning, organization, and management of the program, and
shall provide support to agriculture through appropriate
technology and research, and adequate financial, production,
marketing, and other support services.

Section 6. The State shall apply the principles of agrarian reform
or stewardship, whenever applicable in accordance with law, in
the disposition or utilization of other natural resources, including
lands of the public domain under lease or concession suitable to
agriculture, subject to prior rights, homestead rights of small
settlers, and the rights of indigenous communities to their
ancestral lands.

The State may resettle landless farmers and farm workers in its
own agricultural estates which shall be distributed to them in the
manner provided by law.

Section 7. The State shall protect the rights of subsistence
fishermen, especially of local communities, to the preferential
use of the communal marine and fishing resources, both inland
and offshore. It shall provide support to such fishermen through
appropriate technology and research, adequate financial,
production, and marketing assistance, and other services. The
State shall also protect, develop, and conserve such resources.
The protection shall extend to offshore fishing grounds of
subsistence fishermen against foreign intrusion. Fishworkers
shall receive a just share from their labor in the utilization of
marine and fishing resources.

Section 8. The State shall provide incentives to landowners to
invest the proceeds of the agrarian reform program to promote
industrialization, employment creation, and privatization of
public sector enterprises. Financial instruments used as
payment for their lands shall be honored as equity in enterprises
of their choice.

URBAN LAND REFORM AND HOUSING

Section 9. The State shall, by law, and for the common good,
undertake, in cooperation with the private sector, a continuing
program of urban land reform and housing which will make
available at affordable cost, decent housing and basic services
to under-privileged and homeless citizens in urban centers and
resettlement areas. It shall also promote adequate employment
opportunities to such citizens. In the implementation of such
program the State shall respect the rights of small property
owners.

Section 10. Urban or rural poor dwellers shall not be evicted nor
their dwelling demolished, except in accordance with law and in
a just and humane manner.

No resettlement of urban or rural dwellers shall be undertaken
without adequate consultation with them and the communities
where they are to be relocated.

PD 27, RA 3844 and 6389, CARP (RA 6657, as extended by
CARPer, RA 9700)

PERTINENT LAWS :


PD 27 : DECREEING THE EMANCIPATION OF TENANTS
FROM THE BONDAGE OF THE SOIL, TRANSFERRING TO
THEM THE OWNERSHIP OF THE LAND THEY TILL AND
PROVIDING THE INSTRUMENTS AND MECHANISM
THEREFOR

RA 3844: THE AGRICULTURAL LAND REFORM CODE

The Agricultural Land Reform Code (RA 3844) was a
major Philippine land reform law enacted in 1963 under


President Diosdado Macapagal. It abolished tenancy and
established a leasehold system in which farmers paid fixed rentals
to landlords, rather than a percentage of harvest. It also
established the Land Bank of the Philippines to help with land
reform, particularly the purchase of agricultural estates for division
and resale to small landholders, and the purchase of land by the
agricultural lessee.

While the law was a significant advance over previous legislation,
though the bill was weakened by numerous amendments imposed
by Congress, which was dominated by landlords. It was also
weakened by the failure of Congress to allocate necessary funds
for effective implementation of the law. The act has been further
amended several times subsequent to becoming law by later
legislation

RA 6389: AN ACT AMENDING REPUBLIC ACT NUMBERED
THIRTY-EIGHT HUNDRED AND FORTY-FOUR, AS AMENDED,
OTHERWISE KNOWN AS THE AGRICULTURAL LAND
REFORM CODE, AND FOR OTHER PURPOSES

RA 6657: CARP

This Act institutes the Comprehensive Agrarian Reform Program
(CARP). CARP takes as its declared aim "the establishment of
owner cultivatorship of economic-size farms as the basis of
Philippine agriculture" (Section 2). It prohibits the retention by
landowners of more than five hectares of land and, for each child
of the landowner over 15 years old, an additional three hectares,
providing he or she is personally cultivating that land (Section 6).
That limit of three hectares is also the upper limit for the
redistribution of land to "qualified beneficiaries." These may be
agricultural lessees and share tenants, regular, seasonal, or other
farm workers, actual tillers or occupants of public lands, or some
others directly working on the land (Section 22). Where splitting up
of the land is economically inappropriate, then it may be taken
over by collectives or cooperatives of these categories of
agricultural workers (Sections 25 and 29). The land reforms
envisaged under CARP are scheduled to take place in three
phases over a 10-year period. Land in the public domain held or
leased by multinational corporations is scheduled for expropriation
within three years of the passing of the Act (Section 8); land
otherwise controlled to be redistributed during the first phase is
generally scheduled for redistribution in order of the size of the
landholdings: that is, the larger the holding, the earlier it is
scheduled for redistribution. Land acquisition is to be carried out
by means of the Department of Agrarian Reform (DAR) and the
Land Bank of the Philippines (LBP). The level of compensation
payable to the expropriated landowner is to be determined by the
DAR, the LBP, and the landowner, according to the cost of
acquisition and development of the land, current market values,
the income it generates in the light of the sworn valuation of the
owner, and any tax returns pertaining to it. If the landowner
disagrees with the level of compensation offered, there is a right of
appeal to the courts. Payment of compensation, carried out
through the medium of the LBP, is to be partly in cash and partly
in government bonds, or in credits for taxes or other government
funded services such as education and hospitalization. For land in
excess of 50 hectares, the proportion payable in cash is 25% and
for land below 24 hectares in area, 35%. These amounts may be
increased by 5% in cases where the landowner voluntarily puts
the land forward for transfer under the Program.

RA 9700: CARPER

CARPer incorporates compulsory acquisition, despite the well-
organized attempt from landed legislators or their allies to force
that particular mode off the table. Without compulsory acquisition,
a land reform program is essentially inutile. CARPer restores this
mode of acquisition to its principal roleand allocates P150 billion
over the next five years to fund it.

Mandating it in the law is one thing; implementing it is completely
another, especially in view of the pro-landlord provisions included
in the new law. But all legislation is an exercise in compromise;
citizens only hope that the compromises made, the trade-offs, are
reasonable.

The reforms in RA 9700 include provisions on the sourcing of
the funds, which will allow the Department of Agrarian Reform to
target the acquisition and distribution of the remaining 1 million
hectares or so of agricultural lands covered by CARP at a much
faster pace; the creation of a joint Congressional Oversight
Committee on Agrarian Reform, or COCAR, to closely monitor
the implementation of the new law; the strengthening of the ban
on land-use conversion by landowners eager to avoid CARP, by
extending the scope of the ban to allow no exceptions, by
levying heavier penalties for illegal conversion of agricultural
land into non-agricultural use and by mandating the automatic
coverage of converted land if the conversion is unimplemented
or its terms violatedthus legislating the lesson from the
Sumilao farmers issue.

The law features many more reforms, including: the end of the
mode of voluntary land transfer, which was abused in the past
to favor landowners; the implementation of the principle of
actual and physical possession, when it comes to the award of
land, thus avoiding non-distributive schemes like Stock-
Distribution Options; and the dramatic shortening of the period
for installing farmer-beneficiaries, through such measures as the
inclusion of the standing crop in computing the just-
compensation estimatethus legislating the lesson from the
Hacienda Malaga case.



Section 22. The state recognizes and promotes the rights of
indigenous cultural communities within the framework of
national unity and development.

See:
Article X (LOCAL GOVERNMENT),
XII (NATIONAL ECONOMY AND PATRIMONY) , XIV
(EDUCATION, SCIENCE AND TECHNOLOGY, ARTS AND
CULTURE) and
XVI (GENERAL PROVISIONS)

Read: IPRA LAW, RA 8371 :CRUZ & EUROPA vs. DENR,
347 SCRA 128, GR 135385, 12/6/00

IPRA LAW: Indigenous People's Rights Act of 1997 (IPRA) (RA
8371) is a legislation that recognize and promote all the rights of
Indigenous Cultural Communities/Indigenous Peoples of
the Philippines.

CASE: CRUZ vs DENR

FACTS: Petitioners Isagani Cruz and Cesar Europa brought this
suit for prohibition and mandamus as citizens and taxpayers,
assailing the constitutionality of certain provisions of RA 8371,
otherwise known as the Indigenous Peoples Rights Act of 1997
(IPRA) and its Implementing Rules and Regulations. The
Solicitor General is of the view that the IPRA is partly
unconstitutional on the ground that it grants ownership over
natural resources to indigenous peoples and prays that the
petition be granted in part. The Commission on Human Rights
asserts that IPRA is an expression of the principle of parens
patriae and that the State has the responsibility to protect and
guarantee the rights of those who are at a serious disadvantage
like indigenous people. It prays that the petition be dismissed.

HELD: After due deliberation, 7 voted to dismiss the petition,
while 7 other members of the Court voted to grant the petition.
As the votes were equally divided and the necessary majority
was not obtained, the case was redeliberated upon. However,
after redeliberation, the voting remained the same. Accordingly,
pursuant to Rule 56, Section 7 of the Rules of Civil Procedure,
the petition is DISMISSED.

Section 23.The State shall encourage non-governmental,
community-based, or sectoral organizations that promote the
welfare of the nation.

See: Sec. 15 & 16 of Article XIII



NATIONAL ECONOMY AND PATRIMONY: ROLE AND RIGHTS
OF PEOPLE'S ORGANIZATIONS

Section 15. The State shall respect the role of independent
people's organizations to enable the people to pursue and protect,
within the democratic framework, their legitimate and collective
interests and aspirations through peaceful and lawful means.

People's organizations are bona fide associations of citizens with
demonstrated capacity to promote the public interest and with
identifiable leadership, membership, and structure.

Section 16. The right of the people and their organizations to
effective and reasonable participation at all levels of social,
political, and economic decision-making shall not be abridged. The
State shall, by law, facilitate the establishment of adequate
consultation mechanisms.


Section 24.The state recognizes the vital role of communication
and information in nation-building.

See Article XVI, Secs. 10 and 11

GENERAL PROVISIONS:

Section 10. The State shall provide the policy environment for the
full development of Filipino capability and the emergence of
communication structures suitable to the needs and aspirations of
the nation and the balanced flow of information into, out of, and
across the country, in accordance with a policy that respects the
freedom of speech and of the press.

Section 11. (1) The ownership and management of mass media
shall be limited to citizens of the Philippines, or to corporations,
cooperatives or associations, wholly-owned and managed by such
citizens.

The Congress shall regulate or prohibit monopolies in commercial
mass media when the public interest so requires. No combinations
in restraint of trade or unfair competition therein shall be allowed.

(2) The advertising industry is impressed with public interest, and
shall be regulated by law for the protection of consumers and the
promotion of the general welfare.
Only Filipino citizens or corporations or associations at least
seventy per centum of the capital of which is owned by such
citizens shall be allowed to engage in the advertising industry.

The participation of foreign investors in the governing body of
entities in such industry shall be limited to their proportionate
share in the capital thereof, and all the executive and managing
officers of such entities must be citizens of the Philippines.

Section 25.The state shall ensure the autonomy of local
governments.

Read: Article X: LOCAL GOVERNMENT
1991 Local Government Code; its features on autonomy

Section 26.The state shall guarantee equal access to
opportunities for public service, and prohibit political dynasties as
may be defined by law.

Not self-executing; nuisance candidates
PAMATONG vs. COMELEC, G.R. No. 161872, April 13, 2004

CASE: PAMATONG vs COMELEC

FACTS: Petitioner Pamatong filed his Certificate of Candidacy
(COC) for President. Respondent COMELEC declared petitioner
and 35 others as nuisance candidates who could not wage a
nationwide campaign and/or are not nominated by a political party
or are not supported by a registered political party with a national
constituency.

Pamatong filed a Petition For Writ of Certiorari with the Supreme
Court claiming that the COMELEC violated his right to "equal
access to opportunities for public service" under Section 26,
Article II of the 1987 Constitution, by limiting the number of
qualified candidates only to those who can afford to wage a
nationwide campaign and/or are nominated by political parties.
The COMELEC supposedly erred in disqualifying him since he
is the most qualified among all the presidential candidates, i.e.,
he possesses all the constitutional and legal qualifications for
the office of the president, he is capable of waging a national
campaign since he has numerous national organizations under
his leadership, he also has the capacity to wage an international
campaign since he has practiced law in other countries, and he
has a platform of government.

ISSUE:Is there a constitutional right to run for or hold public
office?

RULING: No. What is recognized in Section 26, Article II of the
Constitution is merely a privilege subject to limitations imposed
by law. It neither bestows such a right nor elevates the privilege
to the level of an enforceable right. There is nothing in the plain
language of the provision which suggests such a thrust or
justifies an interpretation of the sort.

The "equal access" provision is a subsumed part of Article II of
the Constitution, entitled "Declaration of Principles and State
Policies." The provisions under the Article are generally
considered not self-executing, and there is no plausible reason
for according a different treatment to the "equal access"
provision. Like the rest of the policies enumerated in Article II,
the provision does not contain any judicially enforceable
constitutional right but merely specifies a guideline for legislative
or executive action. The disregard of the provision does not give
rise to any cause of action before the courts.

Obviously, the provision is not intended to compel the State to
enact positive measures that would accommodate as many
people as possible into public office. Moreover, the provision as
written leaves much to be desired if it is to be regarded as the
source of positive rights. It is difficult to interpret the clause as
operative in the absence of legislation since its effective means
and reach are not properly defined. Broadly written, the myriad
of claims that can be subsumed under this rubric appear to be
entirely open-ended. Words and phrases such as "equal
access," "opportunities," and "public service" are susceptible to
countless interpretations owing to their inherent impreciseness.
Certainly, it was not the intention of the framers to inflict on the
people an operative but amorphous foundation from which
innately unenforceable rights may be sourced.

The privilege of equal access to opportunities to public office
may be subjected to limitations. Some valid limitations
specifically on the privilege to seek elective office are found in
the provisions of the Omnibus Election Code on "Nuisance
Candidates. As long as the limitations apply to everybody
equally without discrimination, however, the equal access
clause is not violated. Equality is not sacrificed as long as the
burdens engendered by the limitations are meant to be borne by
any one who is minded to file a certificate of candidacy. In the
case at bar, there is no showing that any person is exempt from
the limitations or the burdens which they create.

The rationale behind the prohibition against nuisance
candidates and the disqualification of candidates who have not
evinced a bona fide intention to run for office is easy to divine.
The State has a compelling interest to ensure that its electoral
exercises are rational, objective, and orderly. Towards this end,
the State takes into account the practical considerations in
conducting elections. Inevitably, the greater the number of
candidates, the greater the opportunities for logistical confusion,
not to mention the increased allocation of time and resources in
preparation for the election. The organization of an election with
bona fide candidates standing is onerous enough. To add into
the mix candidates with no serious intentions or capabilities to
run a viable campaign would actually impair the electoral
process. This is not to mention the candidacies which are
palpably ridiculous so as to constitute a one-note joke. The poll
body would be bogged by irrelevant minutiae covering every
step of the electoral process, most probably posed at the


instance of these nuisance candidates. It would be a senseless
sacrifice on the part of the State.

The question of whether a candidate is a nuisance candidate or
not is both legal and factual. The basis of the factual determination
is not before this Court. Thus, the remand of this case for the
reception of further evidence is in order. The SC remanded to the
COMELEC for the reception of further evidence, to determine the
question on whether petitioner Elly Velez Lao Pamatong is a
nuisance candidate as contemplated in Section 69 of the Omnibus
Election Code.

Obiter Dictum: One of Pamatong's contentions was that he was an
international lawyer and is thus more qualified compared to the
likes of Erap, who was only a high school dropout. Under the
Constitution (Article VII, Section 2), the only requirements are the
following: (1) natural-born citizen of the Philippines; (2) registered
voter; (3) able to read and write; (4) at least forty years of age on
the day of the election; and (5) resident of the Philippines for at
least ten years immediately preceding such election.

Note: the bill of political dynasty remains a bill in Congress

Section 27.The state shall maintain honesty and integrity in the
public service and take positive and effective measures against
graft and corruption.

Read: Article XI on Accountability of Public Officers

Section 1. Public office is a public trust. Public officers and
employees must, at all times, be accountable to the people,
serve them with utmost responsibility, integrity, loyalty, and
efficiency; act with patriotism and justice, and lead modest lives.

Section 28. Subject to reasonable conditions prescribed by
law, the State adopts and implements a policy of full public
disclosure of all its transactions involving public interest.

(EO 464) SENATE vs. ERMITA, GR No. G.R. No. 169777, April
20, 2006,

Read: Sec. 7 Art. III- Right to information and access to official
records

SENATE vs ERMITA

FACTS: This is a petition for certiorari and prohibition proffer that
the President has abused power by issuing E.O. 464 Ensuring
Observance of the Principles of Separation of Powers, Adherence
to the Rule on Executive Privilege and Respect for the Rights of
Public Officials Appearing in Legislative Inquiries in Aid of
Legislation Under the Constitution, and for Other Purposes.
Petitioners pray for its declaration as null and void for being
unconstitutional.
In the exercise of its legislative power, the Senate of the
Philippines, through its various Senate Committees, conducts
inquiries or investigations in aid of legislation which call for, inter
alia, the attendance of officials and employees of the executive
department, bureaus, and offices including those employed in
Government Owned and Controlled Corporations, the Armed
Forces of the Philippines (AFP), and the Philippine National Police
(PNP).

The Committee of the Senate issued invitations to various officials
of the Executive Department for them to appear as resource
speakers in a public hearing on the railway project, others on the
issues of massive election fraud in the Philippine elections, wire
tapping, and the role of military in the so-called Gloriagate
Scandal.
Said officials were not able to attend due to lack of consent from
the President as provided by E.O. 464, Section 3 which requires
all the public officials enumerated in Section 2(b) to secure the
consent of the President prior to appearing before either house of
Congress.

ISSUE: Is Section 3 of E.O. 464, which requires all the public
officials, enumerated in Section 2(b) to secure the consent of the
President prior to appearing before either house of Congress,
valid and constitutional?

RULING:No. The enumeration in Section 2 (b) of E.O. 464 is
broad and is covered by the executive privilege. The doctrine of
executive privilege is premised on the fact that certain
information must, as a matter of necessity, be kept confidential
in pursuit of the public interest. The privilege being, by definition,
an exemption from the obligation to disclose information, in this
case to Congress, the necessity must be of such high degree as
to outweigh the public interest in enforcing that obligation in a
particular case.
Congress undoubtedly has a right to information from the
executive branch whenever it is sought in aid of legislation. If
the executive branch withholds such information on the ground
that it is privileged, it must so assert it and state the reason
therefor and why it must be respected.
The infirm provisions of E.O. 464, however, allow the executive
branch to evade congressional requests for information without
need of clearly asserting a right to do so and/or proffering its
reasons therefor. By the mere expedient of invoking said
provisions, the power of Congress to conduct inquiries in aid of
legislation is frustrated.

BILL OF RIGHTS:

Section 7. The right of the people to information on matters of
public concern shall be recognized. Access to official records,
and to documents and papers pertaining to official acts,
transactions, or decisions, as well as to government research
data used as basis for policy development, shall be afforded the
citizen, subject to such limitations as may be provided by law.

The right to information guarantees the right of the people to
demand information, while Section 28 recognizes the duty of
officialdom to give information even if nobody demands. THE
PROVINCE OF NORTH COTABATO vs. GRP PEACE PANEL
ON ANCESTRAL DOMAIN, G.R. No. 183591, October 14,
2008.

PROVINCE OF NC vs GRP

FACTS: On August 5, 2008, the Government of the Republic of
the Philippines (GRP) and the MILF, through the Chairpersons
of their respective peace negotiating panels, were scheduled to
sign a Memorandum of Agreement on the Ancestral Domain
(MOA-AD) Aspect of the GRP-MILF Tripoli Agreement on
Peace of 2001 in Kuala Lumpur, Malaysia.

The signing of the MOA-AD between the GRP and the MILF
was not to materialize, however, for upon motion of petitioners,
specifically those who filed their cases before the scheduled
signing of the MOA-AD, this Court issued a Temporary
Restraining Order enjoining the GRP from signing the same.

The MOA-AD was preceded by a long process of negotiation
and the concluding of several prior agreements between the two
parties beginning in 1996, when the GRP-MILF peace
negotiations began. On July 18, 1997, the GRP and MILF
Peace Panels signed the Agreement on General Cessation of
Hostilities. The following year, they signed the General
Framework of Agreement of Intent on August 27, 1998.

On July 23, 2008, the Province of North Cotabato and Vice-
Governor Emmanuel Piol filed a petition, docketed as G.R. No.
183591, for Mandamus and Prohibition with Prayer for the
Issuance of Writ of Preliminary Injunction and Temporary
Restraining Order. Invoking the right to information on matters
of public concern, petitioners seek to compel respondents to
disclose and furnish them the complete and official copies of the
MOA-AD including its attachments, and to prohibit the slated
signing of the MOA-AD, pending the disclosure of the contents
of the MOA-AD and the holding of a public consultation thereon.
Supplementarily, petitioners pray that the MOA-AD be declared
unconstitutional.

ISSUES:

1. Whether the petitions have become moot and academic



(i) insofar as the mandamus aspect is concerned, in view of the
disclosure of official copies of the final draft of the Memorandum of
Agreement (MOA); and

(ii) insofar as the prohibition aspect involving the Local
Government Units is concerned, if it is considered that
consultation has become fait accompli with the finalization of the
draft;

2. Whether the constitutionality and the legality of the MOA is ripe
for adjudication;

3. Whether respondent Government of the Republic of the
Philippines Peace Panel committed grave abuse of discretion
amounting to lack or excess of jurisdiction when it negotiated and
initiated the MOA vis--vis ISSUES Nos. 4 and 5;

4. Whether there is a violation of the people's right to information
on matters of public concern (1987 Constitution, Article III, Sec. 7)
under a state policy of full disclosure of all its transactions
involving public interest (1987 Constitution, Article II, Sec. 28)
including public consultation under Republic Act No. 7160 (LOCAL
GOVERNMENT CODE OF 1991)[;]

If it is in the affirmative, whether prohibition under Rule 65 of the
1997 Rules of Civil Procedure is an appropriate remedy;

5. Whether by signing the MOA, the Government of the Republic
of the Philippines would be BINDING itself

a) to create and recognize the Bangsamoro Juridical Entity (BJE)
as a separate state, or a juridical, territorial or political subdivision
not recognized by law;

b) to revise or amend the Constitution and existing laws to
conform to the MOA;

c) to concede to or recognize the claim of the Moro Islamic
Liberation Front for ancestral domain in violation of Republic Act
No. 8371 (THE INDIGENOUS PEOPLES RIGHTS ACT OF 1997),
particularly Section 3(g) & Chapter VII (DELINEATION,
RECOGNITION OF ANCESTRAL DOMAINS)

If in the affirmative, whether the Executive Branch has the
authority to so bind the Government of the Republic of the
Philippines;

6. Whether the inclusion/exclusion of the Province of North
Cotabato, Cities of Zamboanga, Iligan and Isabela, and the
Municipality of Linamon, Lanao del Norte in/from the areas
covered by the projected Bangsamoro Homeland is a justiciable
question; and

7. Whether desistance from signing the MOA derogates any prior
valid commitments of the Government of the Republic of the
Philippines.

HELD:
The main body of the MOA-AD is divided into four strands,
namely, Concepts and Principles, Territory, Resources, and
Governance.

The power of judicial review is limited to actual cases or
controversies. Courts decline to issue advisory opinions or to
resolve hypothetical or feigned problems, or mere academic
questions. The limitation of the power of judicial review to actual
cases and controversies defines the role assigned to the judiciary
in a tripartite allocation of power, to assure that the courts will not
intrude into areas committed to the other branches of
government.

As the petitions involve constitutional issues which are of
paramount public interest or of transcendental importance, the
Court grants the petitioners, petitioners-in-intervention and
intervening respondents the requisite locus standi in keeping with
the liberal stance adopted in David v. Macapagal-Arroyo.

Contrary to the assertion of respondents that the non-signing of
the MOA-AD and the eventual dissolution of the GRP Peace
Panel mooted the present petitions, the Court finds that the
present petitions provide an exception to the "moot and
academic" principle in view of (a) the grave violation of the
Constitution involved; (b) the exceptional character of the
situation and paramount public interest; (c) the need to
formulate controlling principles to guide the bench, the bar, and
the public; and (d) the fact that the case is capable of repetition
yet evading review.

The MOA-AD is a significant part of a series of agreements
necessary to carry out the GRP-MILF Tripoli Agreement on
Peace signed by the government and the MILF back in June
2001. Hence, the present MOA-AD can be renegotiated or
another one drawn up that could contain similar or significantly
dissimilar provisions compared to the original.

That the subject of the information sought in the present cases
is a matter of public concern faces no serious challenge. In fact,
respondents admit that the MOA-AD is indeed of public
concern. In previous cases, the Court found that the regularity of
real estate transactions entered in the Register of Deeds, the
need for adequate notice to the public of the various laws, the
civil service eligibility of a public employee, the proper
management of GSIS funds allegedly used to grant loans to
public officials, the recovery of the Marcoses' alleged ill-gotten
wealth, and the identity of party-list nominees, among others,
are matters of public concern. Undoubtedly, the MOA-AD
subject of the present cases is of public concern, involving as it
does the sovereignty and territorial integrity of the State, which
directly affects the lives of the public at large.

In sum, the Presidential Adviser on the Peace Process
committed grave abuse of discretion when he failed to carry out
the pertinent consultation process, as mandated by E.O. No. 3,
Republic Act No. 7160, and Republic Act No. 8371. The furtive
process by which the MOA-AD was designed and crafted runs
contrary to and in excess of the legal authority, and amounts to
a whimsical, capricious, oppressive, arbitrary and despotic
exercise thereof. It illustrates a gross evasion of positive duty
and a virtual refusal to perform the duty enjoined.

The MOA-AD cannot be reconciled with the present Constitution
and laws. Not only its specific provisions but the very concept
underlying them, namely, the associative relationship
envisioned between the GRP and the BJE, are unconstitutional,
for the concept presupposes that the associated entity is a state
and implies that the same is on its way to independence.

The Memorandum of Agreement on the Ancestral Domain
Aspect of the GRP-MILF Tripoli Agreement on Peace of 2001 is
declared contrary to law and the Constitution.

ARTICLE VI.
LEGISLATIVE DEPARTMENT

Separation of Powers
A. To avoid concentration of powers/authority
B. Sample cases on separation of powers:

Planas v. Gil, 67 Phil 62

FACTS: On 17 Nov 1978, Carmen Planas (then a municipal
board member of Manila) statement criticizing the acts of certain
government officials including Pres. Quezon was published in la
vanguardia. The following morning, she received a letter from
Vargas (Secretary to the President) by order of the president
containing the following: In the above statement, you appear to
make the following charges: (1) That the President of the
Philippines has violated the Constitution in that he has taken
part in politics, expressing his preference for the candidates of
the Nacionalista Party; (2) That the whole government
machinery has been put in action to prevent the election to the
National Assembly of the candidates of the people; (3) That the
candidates of the NP and of the administration have won the
election through frauds and violations of the civil service rules;
(4) That the administration does not permit the people to freely
elect the candidates of their choice. Planas was then ordered to
appear before the CSC for investigation and to prove her
allegations against the administration. She appeared before the


CSC but she questioned the jurisdiction of the CSC over the
matter. She said that as an elective official, is accountable for her
political acts to her constituency alone, unless such acts constitute
offenses punishable under our penal laws, and not to executive
officials belonging to a party opposed to that to which petitioner is
affiliated. Further, her statement of November 17th made by her
as a private citizen and in the exercise of her right to discuss freely
political questions cannot properly be the subject of an
administrative investigation had with a view to her suspension or
removal, and is only cognizable by our courts of justice in case the
contents of said statement infringe any provision of our Penal
Code and that her removal due to the same would be an arbitrary
act by the CSC. The CSC however took cognizance of the case
hence the appeal to the SC. The Solicitor General replied for the
CSC arguing that under the separation of powers marked by the
Constitution, the court has no jurisdictions to review the orders of
the Chief Executive which are of purely administrative character.

ISSUE: Whether or not the SC has jurisdiction to review orders
issued by the President.

HELD: The acts of the Chief Executive performed within the limits
of his jurisdiction are his official acts and courts will neither direct
nor restrain executive action in such cases. The rule is non-
interference. But from this legal premise, it does not necessarily
follow that the SC is precluded from making an inquiry into the
validity or constitutionality of his acts when these are properly
challenged in an appropriate legal proceeding. The classical
separation of governmental powers viewed in the light of political
philosophy is a relative theory of government. There is more
truism and actuality in interdependence than in independence and
separation of powers. In the present case, the President is not a
party to the proceeding. He is neither compelled nor restrained to
act in a particular way. The CSC is the party respondent and the
theory is advanced by the Sol-Gen that because an investigation
undertaken by him is directed by authority of the President of the
Philippines, the SC has no jurisdiction over the present
proceedings instituted by Planas. The argument is farfetched. A
mere plea that a subordinate officer of the government is acting
under orders from the Chief Executive may be an important
averment, but is neither decisive nor conclusive upon this court.
Like the dignity of his high office, the relative immunity of the Chief
Executive from judicial interference is not in the nature of a
sovereign passport for all the subordinate official and employees
of the executive Department to the extent that at the mere
invocation of the authority that it purports the jurisdiction of this
court to inquire into the validity or legality of an executive order is
necessarily abated or suspended. Nevertheless, SC ruled that
CSC can take cognizance of the case. Planas was not denied the
right to voice out her opinion but since she made allegations
against the administration it is but right for her to prove those
allegations. The CSC has the right to elicit the truth.

Santiago/Tatad v. Guingona, 11/18/98

FACTS: During the first regular session of the eleventh Congress,
Senator Fernan was declared the duly elected President of the
Senate by a vote of 20 to 2. Senator Tatad manifested that, with
the agreement of Senator Santiago, allegedly the only other
member of the minority, he was assuming the position of minority
leader. He explained that those who had voted for Senator Fernan
comprised the majority, while only those who had voted for him,
the losing nominee, belonged to the minority. Senator Flavier
manifested that the senators belonging to the Lakas-NUCD-UMDP
Party numbering 7 and, thus, also a minority had chosen Senator
Guingona as the minority leader. Thereafter, the majority leader
informed the body that he was in receipt of a letter signed by the 7
Lakas-NUCD-UMDP senators, stating that they had elected
Senator Guingona as the minority leader. By virtue thereof, the
Senate President formally recognized Senator Guingona as the
minority leader of the Senate. Senators Santiago and Tatad filed a
petition for quo warranto, alleging that Senator Guingona had
been usurping, unlawfully holding and exercising the position of
Senate minority leader, a position that, according to them,
rightfully belonged to Senator Tatad.

ISSUES:
(1) Whether or not the Court has jurisdiction over the petition
(2) Whether or not there is an actual violation of the Constitution

HELD: Regarding the first issue, jurisdiction over the subject
matter of a case is determined by the allegations of the
complaint or petition, regardless of whether the petitioner is
entitled to the relief asserted. In light of the allegations of the
petitioners, it is clear that the Court has jurisdiction over the
petition. It is well within the power and jurisdiction of the Court to
inquire whether indeed the Senate or its officials committed a
violation of the Constitution or gravely abused their discretion
in the exercise of their functions and prerogatives.

However, the interpretation proposed by petitioners finds no
clear support from the Constitution, the laws, the Rules of the
Senate or even from practices of the Upper House. The term
majority, when referring to a certain number out of a total
or aggregate, it simply means the number greater than half or
more than half of any total. In effect, while the
Constitution mandates that the President of the Senate must be
elected by a number constituting more than one half of all the
members thereof, it does not provide that the members who will
not vote for him shall ipso facto constitute the minority, who
could thereby elect the minority leader. No law or regulation
states that the defeated candidate shall automatically become
the minority leader.

While the Constitution is explicit in the manner of electing a
SenatePresident and a House Speaker, it is, however, dead
silent on the manner of selecting the other officers in both
chambers of Congress. All that the Charter says under Art. VI,
Sec. 16(1) is that each House shall choose such other officers
as it may deem necessary. The method of choosing who will be
such other officers is merely a derivative of the exercise of the
prerogative conferred by the said constitutional provision.
Therefore, such method must be prescribed by the Senate itself,
not by the Court.

Garcia v. Macaraig, 39 SCRA 106

FACTS: Judge Macaraig took his oath as Judge of the CFI of
Laguna and San Pablo City on June 29, 1970. The court, being
one of the 112 newly created CFI branches, had to be
organized from scratch. From July 1, 1970 to February 28,
1971, Macaraig was not able to assume the duties and
functions of a judge due to the fact that his Court Room can not
be properly established due to problems as to location and as to
appropriations to make his Court up and running. When
Macaraig realized that it would be sometime before he could
actually preside over his court, he applied for an extended leave
(during the 16 years he had worked in the Department of
Justice, respondent had, due to pressure of duties, never gone
on extended leave, resulting in his forfeiting all the leave
benefits he had earned beyond the maximum ten months
allowed by the law). The Secretary of Justice, however,
prevailed upon respondent to forego his leave and instead to
assist him, without being extended a formal detail, whenever
respondent was not busy attending to the needs of his court.
Paz Garcia on the other hand filed a complaint alleging that
Macaraig is incompetent, dishonest and has acted in violation of
his oath as a judge. Garcia said that Macaraig has not
submitted the progress of his Courts as required by law. And
that Macaraig has received salaries as a judge while he is fully
aware that he has not been performing the duties of a judge.

ISSUE: Whether or not Macaraig has acted with incompetence
and dishonesty as Judge.

HELD: Macaraigs inability to perform his judicial duties under
the circumstances mentioned above does not constitute
incompetence. Respondent was, like every lawyer who gets his
first appointment to the bench, eager to assume his judicial
duties and rid himself of the stigma of being a judge without a
sala, but forces and circumstances beyond his control
prevented him from discharging his judicial duties. On the other
hand, none of these is to be taken as meaning that the Court
looks with favor at the practice of long standing, to be sure, of
judges being detailed in the DOJ to assist the Secretary even if
it were only in connection with his work of exercising
administrative authority over the courts. The line between what
a judge may do and what he may not do in collaborating or


working with other offices or officers under the other great
departments of the government must always be kept clear and
jealously observed, lest the principle of separation of powers on
which our government rests by mandate of the people thru the
Constitution be gradually eroded by practices purportedly
motivated by good intentions in the interest of the public service.
The fundamental advantages and the necessity of the
independence of said three departments from each other, limited
only by the specific constitutional precepts on check and balance
between and among them, have long been acknowledged as
more paramount than the serving of any temporary or passing
governmental conveniences or exigencies. It is thus of grave
importance to the judiciary under our present constitutional
scheme of government that no judge of even the lowest court in
this Republic should place himself in a position where his
actuations on matters submitted to him for action or resolution
would be subject to review and prior approval and, worst still,
reversal, before they can have legal effect, by any authority other
than the Court of Appeals or the Supreme Court, as the case may
be. Needless to say, the Court feels very strongly that it is best
that this practice is discontinued.


CASE: Dolalas v. Ombudsman, 12/24/96

FACTS: Petitioners, Judge Ana Maria I. Dolalas, Evelyn K. Obido
and Wilberto B. Carriedo - Presiding Judge, Clerk of Court and
Clerk II, respectively of the Municipal Circuit Trial Court of
Kabasalan, Zamboanga del Sur, were charged administratively
by private respondent Benjamin Villarante, Jr. for miscarriage of
justice, dishonesty, gross neglect of duty, unnecessary delay in
the administration of justice and for failure to prosecute Criminal
Case No. 5881 for an unreasonable length of time before public
respondent Office of the Ombudsman-Mindanao.

ISSUE: Whether or not the Office of the Ombudsman may take
cognizance of the complaint against petitioner for purposes of
investigation and possible prosecution in accordance with its
mandate under Section 13 (1) and (2) of Article XI of the 1987
Constitution

for alleged violation of the Anti-Graft and Corrupt
Practices Act.

HELD: SC has jurisdiction and not the Ombudsman.

The complaint against petitioner-judge before the Office of the
Ombudsman is basically administrative in nature. In essence,
petitioner-judge is being charged with having violated Rule 1.02,
Canon 1

and Rule 3.05, Canon 3of the Code of Judicial Conduct.
It must be borne in mind that the resolution of the administrative
charge of unduly delaying the disposition of the said criminal case
involves the determination of whether, in resolving the alarms and
scandals case, petitioner-judge acted in accordance with the
guidelines provided in the Rules of Court and in the Administrative
Circulars in pursuance of the ideals embodied in the Code of
Judicial Conduct. Such is clearly an administrative
matter. Unquestionably, this Court is mandated under Section 6,
Article VIII of the 1987 Constitution to assume administrative
supervision over all courts and the personnel thereof.

This Court, in the case of Sanz Maceda v. Vasquez, 221 SCRA
464, held that:

Article VIII, Section 6 of the 1987 Constitution exclusively vests in
the Supreme Court administrative supervision over all courts and
court personnel, from the Presiding Justice of the Court of Appeals
down to the lowest municipal trial court clerk. By virtue of this
power, it is only the Supreme Court that can oversee the judges
and court personnels compliance with all laws, and take the
proper administrative action against them if they commit any
violation thereof. No other branch of government may intrude into
this power, without running afoul of the doctrine of separation of
powers.
Public respondent Ombudsman cannot justify its investigation of
petitioner on the powers granted to it by the Constitution, for such
a justification not only runs counter to the specific mandate of the
Constitution granting supervisory powers to the Supreme Court
over all courts and their personnel, but likewise undermines the
independence of the judiciary.
Power of Investigation vs. Executive Privilege- SENATE OF
THE PHIL. et al., vs. ERMITA, GR No. 169777, April 20, 2006-
constitutionality of EO 464 (SEE SECTION 28 CASE DIGEST)

CASE: ABAKADA GURO PARTY LIST vs. HON. CESAR V.
PURISIMA- on joint congressional oversight committee to
approve executive implementing rules

FACTS: Petitioners question the Attrition Act of 2005 and
contend that by establishing a system of rewards and incentives
when they exceed their revenue targets, the law (1) transforms
the officials and employees of the BIR and BOC into
mercenaries and bounty hunters; (2) violates the constitutional
guarantee of equal protection as it limits the scope of the law to
the BIR and BOC; (3) unduly delegates to the President the
power to fix revenue targets without sufficient standards; and (4)
violates the doctrine of separation of powers by creating a
Congressional Oversight Committee to approve the laws
implementing rules.

ISSUE: Is R.A. No. 9335 constitutional?

HELD:YES. R.A. No. 9335 is constitutional, except for Section
12 of the law which creates a Joint Congressional Oversight
Committee to review the laws IRR. That RA No. 9335 will turn
BIR and BOC employees and officials into bounty hunters and
mercenaries is purely speculative as the law establishes
safeguards by imposing liabilities on officers and employees
who are guilty of negligence, abuses, malfeasance, etc. Neither
is the equal protection clause violated since the law recognizes
a valid classification as only the BIR and BOC have the
common distinct primary function of revenue generation. There
are sufficient policy and standards to guide the President in
fixing revenue targets as the revenue targets are based on the
original estimated revenue collection expected of the BIR and
the BOC.However, the creation of a Joint Congressional
Oversight Committee for the purpose of reviewing the IRR
formulated by agencies of the executive branch (DOF, DBM,
NEDA, etc.) is unconstitutional since it violates the doctrine of
separation of powers since Congress arrogated judicial power
upon itself.

C. Blending of powers
instances: appointment, amnesty, appropriation,
ratification of treaties

D. Principle of Checks and Balances
legislative power- spending power, confirmatory
power, treaty ratification
executive power- veto, appointment, clemency
judicial power judicial review


ARTICLE VI
Legislative Department

Section 1. The legislative power shall be vested in the Congress
of the Philippines, which shall consist of a Senate and a House
of Representatives, except to the extent reserved to the people
by the provision on initiative and referendum.


Powers of Congress
Legislative - authority to make laws power of appropriation,
taxation and expropriation

Non legislative- power to canvass the presidential election,
declare the existence of war, give concurrence to treaties and
amnesties, propose constitutional amendments, impeach,
derivative and delegated power, implied powers

1) Legislative power-defined ( authority to make laws and to
alter and repeal them)

a) KINDS:

a. original- possessed by the people in their sovereign
capacity


b. derivative- delegated by the sovereign people to legislative
bodies and is subordinate to the original power of the
people
c. constituent- power to amend or revise the Constitution
d. ordinary- power to pass ordinary laws

2) Basic Concepts of legislative power

a) no passage of irrepealable laws
including imposition of a higher majority for its repeal KIDA v.
SENATE, 659 SCRA 270

KIDA vs. SENATE

FACTS: RA 9054, act to expanding and strengthen organic act for
ARMM was passed. In Section 1 of Article XVII of RA 9054, it
required a super majority voting requirement and gives RA 9054
the character of irrepealable laws by requiring more than what the
Constitution demands. Valid?

RULING: No. RA 9054 required 2/3 voting requirement which is
higher than what the Constitution required. It restrains plenary
power of Congress to amend, revise and repeal laws it had
passed. The Constitution provides only majority of each house
shall constitute a quorum to do business.

b) separation of powers- legislation belongs to Congress,
execution to the executive, settlement of legal
controversies to the judiciary.
c) non-delegability- Congress may not delegate to the court
or any other tribunal, powers which are strictly and
exclusively legislative (Congress may not delegate its
law-making power)

3) Nature-inherent

4) Limits to legislative power
a) Substantive- limitations on the content of laws, the
exercise of power itself and the allowable subject of
legislation.
b) Procedural- limitations on the manner of passing law and
the form they should take

5) Substantive limitations
i) Express limitations- Constitutional limitations

a. Bill of Rights
b. Art VI, Sec 24-26, 28-30

Section 24. All appropriation, revenue or tariff bills, bills
authorizing increase of the public debt, bills of local application,
and private bills, shall originate exclusively in the House of
Representatives, but the Senate may propose or concur with
amendments.

Section 25.
1. The Congress may not increase the appropriations
recommended by the President for the operation of the
Government as specified in the budget. The form, content,
and manner of preparation of the budget shall be prescribed
by law.
2. No provision or enactment shall be embraced in the general
appropriations bill unless it relates specifically to some
particular appropriation therein. Any such provision or
enactment shall be limited in its operation to the
appropriation to which it relates.
3. The procedure in approving appropriations for the Congress
shall strictly follow the procedure for approving
appropriations for other departments and agencies.
4. A special appropriations bill shall specify the purpose for
which it is intended, and shall be supported by funds actually
available as certified by the National Treasurer, or to be
raised by a corresponding revenue proposal therein.
5. No law shall be passed authorizing any transfer of
appropriations; however, the President, the President of the
Senate, the Speaker of the House of Representatives, the
Chief Justice of the Supreme Court, and the heads of
Constitutional Commissions may, by law, be authorized to
augment any item in the general appropriations law for their
respective offices from savings in other items of their
respective appropriations.
6. Discretionary funds appropriated for particular officials
shall be disbursed only for public purposes to be
supported by appropriate vouchers and subject to such
guidelines as may be prescribed by law.
7. If, by the end of any fiscal year, the Congress shall have
failed to pass the general appropriations bill for the
ensuing fiscal year, the general appropriations law for the
preceding fiscal year shall be deemed re-enacted and
shall remain in force and effect until the general
appropriations bill is passed by the Congress.

Section 26.
1. Every bill passed by the Congress shall embrace only
one subject which shall be expressed in the title
thereof.
2. No bill passed by either House shall become a law
unless it has passed three readings on separate days,
and printed copies thereof in its final form have been
distributed to its Members three days before its
passage, except when the President certifies to the
necessity of its immediate enactment to meet a public
calamity or emergency. Upon the last reading of a bill,
no amendment thereto shall be allowed, and the vote
thereon shall be taken immediately thereafter, and the
yeas and nays entered in the Journal.

Section 28.
1. The rule of taxation shall be uniform and equitable. The
Congress shall evolve a progressive system of
taxation.
2. The Congress may, by law, authorize the President to
fix within specified limits, and subject to such
limitations and restrictions as it may impose, tariff
rates, import and export quotas, tonnage and wharfage
dues, and other duties or imposts within the framework
of the national development program of the
Government.
3. Charitable institutions, churches and personages or
convents appurtenant thereto, mosques, non-profit
cemeteries, and all lands, buildings, and
improvements, actually, directly, and exclusively used
for religious, charitable, or educational purposes shall
be exempt from taxation.
4. No law granting any tax exemption shall be passed
without the concurrence of a majority of all the
Members of the Congress.

Section 29.
1. No money shall be paid out of the Treasury except in
pursuance of an appropriation made by law.
2. No public money or property shall be appropriated,
applied, paid, or employed, directly or indirectly, for the
use, benefit, or support of any sect, church,
denomination, sectarian institution, or system of
religion, or of any priest, preacher, minister, other
religious teacher, or dignitary as such, except when
such priest, preacher, minister, or dignitary is assigned
to the armed forces, or to any penal institution, or
government orphanage or leprosarium.
3. All money collected on any tax levied for a special
purpose shall be treated as a special fund and paid out
for such purpose only. If the purpose for which a
special fund was created has been fulfilled or
abandoned, the balance, if any, shall be transferred to
the general funds of the Government.

Section 30. No law shall be passed increasing the appellate
jurisdiction of the Supreme Court as provided in this Constitution
without its advice and concurrence.


ii) Implied limitation
(1) Principle of Non-delegation (delegata
potestas non delegari potest)
(a) corollary to separation of powers
(b) applies to the three(3) powers of
government


(c) And ALSO to grant of legislative franchise
Jaworksi vs. PAGCOR and SAGE, GR
No. 144463, 1/4/04

J aworski vs. PAGCOR

Facts: The PAGCOR was granted, subject to the terms and
conditions established in theDecree, for a period of 25 years,
renewable for another 25 years, the rights, privileges and authority
to operate and maintain gambling casinos, clubs, and other
recreation. On 31March 1998, PAGCORs board of directors
approved an instrument denominated as "Grant of Authority and
Agreement for the Operation of Sports Betting and Internet
Gaming", which granted SAGE the authority to operate and
maintain Sports Betting station in PAGCORs casino locations, and
Internet Gaming facilities to service local and international bettors,
provided that to the satisfaction of PAGCOR, appropriate
safeguards and procedures are established to ensure the integrity
and fairness of the games. Senator Jaworski, in his capacity as
member of the Senate and Chairman of the Senate Committee on
Games, Amusement and Sports, files the petition for certiorari and
prohibition, praying that the grant of authority by PAGCOR in favor
of SAGE be nullified. He contends that PAGCOR is not authorized
under its legislative franchise, P.D. 1869, to operate gambling on
the internet for the simple reason that the said decree could not
have possibly contemplated internet gambling since at the time
of its enactment the internet was yet inexistent and gambling
activities were confined exclusively to real-space. Further, he
argues that the internet, being an international network of
computers, necessarily transcends the territorial jurisdiction of the
Philippines, and the grant to SAGE of authority to operate internet
gambling contravenes the limitation in PAGCORs franchise.

Issue: Whether PAGCORs legislative franchise include the right to
vest another entity, SAGE, with the authority to operate Internet
gambling.

Held: A legislative franchise is a special privilege granted by the
state to corporations. It is a privilege of public concern which
cannot be exercised at will and pleasure, but should be reserved
for public control and administration, either by the government
directly, or by public agents, under such conditions and
regulations as the government may impose on them in the interest
of the public. It is Congress that prescribes the conditions on
which the grant of the franchise may be made. Thus the manner of
granting the franchise, to whom it may be granted, the mode of
conducting the business, the charter and the quality of the service
to be rendered and the duty of the grantee to the public in
exercising the franchise are almost always defined in clear and
unequivocal language. Herein, PAGCOR has acted beyond the
limits of its authority when it passed on or shared its franchise to
SAGE. While PAGCOR is allowed under its charter to enter into
operators and/or management contracts, it is not allowed under
the same charter to relinquish or share its franchise, much less
grant a veritable franchise to another entity such as SAGE.
PAGCOR cannot delegate its power in view of the legal principle
of delegata potestas delegare non potest, inasmuch as there is
nothing in the charter to show that it has been expressly
authorized to do so.


iii) Permissive Delegation of Legislative Power

(1) Delegations provided under the Constitutions
(a) Tariff powers of the President -Sec. 28 (2)
Art. VI
The Congress may, by law, authorize the President to fix within
specified limits, and subject to such limitations and restrictions as
it may impose, tariff rates, import and export quotas, tonnage and
wharfage dues, and other duties or imposts within the framework
of the national development program of the Government.

(b) Emergency powers to the Pres.-Sec. 23 (2)
Art. VI
In times of war or other national emergency, the Congress may,
by law, authorize the President, for a limited period and subject to
such restrictions as it may prescribe, to exercise powers
necessary and proper to carry out a declared national policy.
Unless sooner withdrawn by resolution of the Congress, such
powers shall cease upon the next adjournment thereof.

(2) Delegations to Local Governments - Sec. 5
Art. X
Each local government unit shall have the power to create its
own sources of revenues and to levy taxes, fees and charges
subject to such guidelines and limitations as the Congress may
provide, consistent with the basic policy of local autonomy. Such
taxes, fees, and charges shall accrue exclusively to the local
governments.

(3) Delegation of Rule-making power to
Administrative bodies;
(a) May rules and regulations issued by
Adm. Bodies have the force and effect of
a law?- Ans. Provided the requirement of
completeness and sufficiency of
standards are satisfied, the regulations
passed by the administrative body
pursuant to the delegation made by the
statute are just binding as if the
regulation had been written in the original
statute itself-BERNAS.

Delegation to the People (Sec. 32 Art. VI)

The Congress shall, as early as possible, provide for a system
of initiative and referendum, and the exceptions therefrom,
whereby the people can directly propose and enact laws or
approve or reject any act or law or part thereof passed by the
Congress or local legislative body after the registration of a
petition therefor signed by at least ten per centum of the total
number of registered voters, of which every legislative district
must be represented by at least three per centum of the
registered voters thereof.

Requisites for a valid delegation by a law
1. Completeness test- the law must set forth the policy

2. Sufficient Standard test- the law must set forth the
limits within which the delegate
must conform
Cases:
delegation to the executive department- Pelaez v.
Auditor General, 15 SCRA 569; US v. Ang Tang
Ho, 43 Phil 1 [1922]; P. v. Rosenthal, 68 Phil 328
[1939]; delegated power to remove BIR and BOC
officials under the Attrition Law. ABAKADA GURO
PARTY LIST vs. PURISIMA, G.R. No. 166715,
August 14, 2008

Emmanuel Pelaez vs. The Auditor General

FACTS:From September 4, 1964 to October 29, 1964 the
President of the Philippines issued executive orders to create
thirty-three municipalities pursuant to Section 69 of the Revised
Administrative Code. Public funds thereby stood to be
disbursed in the implementation of said executive orders.
Suing as a private citizen and taxpayer, Vice President
Emmanuel Pelaez filed a petition for prohibition with preliminary
injunction against the Auditor General. It seeks to restrain from
the respondent or any person acting in his behalf, from passing
in audit any expenditure of public funds in implementation of the
executive orders aforementioned.

ISSUE:
Whether the executive orders are null and void, upon the
ground that the President does not have the authority to create
municipalities as this power has been vested in the legislative
department.

RULING:
Section 10(1) of Article VII of the fundamental law ordains:
The President shall have control of all the executive
departments, bureaus or offices, exercise general supervision
over all local governments as may be provided by law, and take
care that the laws be faithfully executed.


The power of control under this provision implies the right of the
President to interfere in the exercise of such discretion as may be
vested by law in the officers of the executive departments,
bureaus, or offices of the national government, as well as to act in
lieu of such officers. This power is denied by the Constitution to
the Executive, insofar as local governments are concerned. Such
control does not include the authority to either abolish an
executive department or bureau, or to create a new one. Section
68 of the Revised Administrative Code does not merely fail to
comply with the constitutional mandate above quoted, it also gives
the President more power than what was vested in him by the
Constitution.
The Executive Orders in question are hereby declared null and
void ab initio and the respondent permanently restrained from
passing in audit any expenditure of public funds in implementation
of said Executive Orders or any disbursement by the
municipalities referred to.

US vs Ang Tang Ho

Facts: The Philippine Legislature passed Act No. 2868, which
authorized the Governor-General, for any cause, or conditions
resulting in an extraordinary rise in the price of palay, rice or corn,
to issue and promulgate, with the consent of the Council of State,
temporary rules and emergency measures for carring out the
purposes of the Act. Governor-General issued Executive Order
No. 53, fixing the price at which rice should be sold at Php0.63 per
ganta. A complaint was filed against Ang Tang Ho for the violation
of EO No. 53, for having sold to one Pedro Trinidad 1 ganta of rice
for Php0.80. TC: found Ang Tang Ho guilty and sentenced to 5
months imprisonment and to pay Php500.
ISSUE: WoN Act No. 2868 delegates legilative power to the
Governor-General, in authorizing the him to fix the price at which
price should be sold

RULING: YES. Act No. 2868 was an undue delegation of legisltive
power to the Governor-General. The Legislature cannot delegate
the power to enact any law. By the Organic Law, legislative power
is vested solely in the Legislature, and such power to make laws
cannot be delegated to the Governor-General or anyone else.

CASE: PEOPLE vs. ROSENTHAL

FACTS: RA 2581 empowers the insular treasurer to issue and
cancel certificate or permits for the sale of speculative securities.
Petitioner contends that NO STANDARD is fixed in the act which
can guide said official in determining the cases in which a
certificate or permit ought to be issued, making his opinion the
sole criterion in the matter of its issuance, with the result that,
legislative powers, being unduly delegated to the insular treasurer.
True?

RULING: No. Act furnishes sufficient standard for the treasurer to
follow in reaching a decision regarding the issuance or
cancellation of certificate permit. Authority of the insular treasurer
is conditioned upon a finding that such cancellation is in public
interest. Public interest is a sufficient standard. No undue
delegation of legislative power.

CASE: ABAKADA VS PURISIMA

FACTS: (Attrition Act of 2005).RA 9335 was enacted to optimize
the revenue-generation capability and collection of the Bureau of
Internal Revenue (BIR) and the Bureau of Customs (BOC). The
law intends to encourage BIR and BOC officials and employees to
exceed their revenue targets by providing a system of rewards
and sanctions through the creation of a Rewards and Incentives
Fund (Fund) and a Revenue Performance Evaluation Board
(Board). It covers all officials and employees of the BIR and the
BOC with at least six months of service, regardless of employment
status2.

Petitioners, invoking their right as taxpayers filed this petition
challenging the constitutionality of RA 9335, a tax reform
legislation. They contend that, by establishing a system of rewards
and incentives, the law "transform[s] the officials and employees of
the BIR and the BOC into mercenaries and bounty hunters" as
they will do their best only in consideration of such
rewards. Petitioners also assail the creation of a congressional
oversight committee on the ground that it violates the doctrine of
separation of powers,
for it permits legislative participation in the implementation and
enforcement of the law.
ISSUE:WON the joint congressional committee is valid
and constitutional

HELD:
No. It is unconstitutional. In the case of Macalintal, in the
discussion of J. Puno,
the power of oversight embraces all activities undertaken by
Congress to enhance its understanding of and influence over
the implementation of legislation it has enacted. Clearly,
oversight concerns post-enactment
measures undertaken by Congress: (a) to monitor bureaucratic
compliance with program objectives, (b) to determine whether
agencies are properly administered, (c) to eliminate executive
waste and dishonesty, (d) to prevent executive usurpation of
legislative authority,and (d) to assess executive conformity with
the congressional perception of public interest.
The power of oversight has been held to be intrinsic in the grant
of legislative power itself and integral to the checks and
balances inherent in a democratic system of government with
this backdrop, it is clear that congressional oversight is not
unconstitutional per se meaning, it neither necessarily
constitutes an encroachment on the executive power to
implement laws nor undermines the constitutional separation of
powers. Rather, it is integral to the checks and balances
inherent in a democratic system of government. It may in fact
even enhance the separation of powers as it prevents the over-
accumulation of power in the executive branch.
However, to forestall the danger of congressional encroachment
"beyond the legislative sphere," the Constitution imposes two
basic and related constraints on Congress
. It may not vest itself, any of its committees or its members with
either executive or judicial power.
And, when it exercises its legislative power, it must follow
the "single, finely wrought and exhaustively considered,
procedures" specified under the Constitution including the
procedure for enactment of laws and presentment.


delegation to administrative bodies/agencies-rule
making power -Tablarin v. Gutierrez, 152 SCRA
730 [1987] ; Eastern Shipping v. POEA, 166 SCRA
533 [1988]; Araneta v. Gatmaitan, 101 Phil 328
[1957]; P. v. Maceren, 76 SCRA 450 [1977](not in
the Scra)0; CEBU OXYGEN V. Sec. DRILON, 8/2/89


CASE: TABLARIN vs. GUTTIEREZ

FACTS: RA 2382 delegates to the Board of Medical Education
Authority to require the taking and passing of National Medical
Admission test as a condition for securing Certificates of
Eligibility for admission. Petitioners did not take NMAT.
Petitioner argues that that RA 2382 offend against the
Constitutional principle which forbids undue delegation of
legislative power, by failing to establish the necessary standard
to be followed by the delegate, the Board of Medical Authority.
True?

RULING: No. Standards may either be implied or express, from
the policy and purpose of the act considered as a whole. The
necessary standard in this case is the standardization and
regulation of medical education. It is considered as necessary
compliance with requirements of the non-delegation principle.
NMAT is a necessary requirement for regulation of medical
education.

CASE: EASTERN vs. POEA

HELD: Legislative discretion as to the substantive contents of
the law cannot be delegated. What can be delegated is the
discretion to determine how the law may be enforced, not what
the law shall be. The ascertainment of the latter subject is a
prerogative of the legislature. This prerogative cannot be
abdicated or surrendered by the legislature to the delegate.



CASE: ARANETA vs. GATMAITAN
FACTS: Congress promulgated RA 4003 (Banning of fishing and
imposing the use of fish net). EO No. 22, 66, 80 was issued was
issued by the President to prohibit the use of trawls in San Miguel
Bay and to carry out Act No. 4003. A group of otter trawl operators
took the matter to the court to restrain the Secretary of Agriculture
from enforcing said EO. Does the issuance of EOs by the
President constitute an undue delegation of power? (Please refer
to Sir Liceraldes discussion about this case because the full text
is punit-punit, cant find sa net. Facts incomplete.)

RULING: No. The true distinction of the power to legislate and the
conferring of authority or discretion as to the execution of the law
consists in that the former necessarily involves a discretion as to
what the law shall be, while in the latter, the authority or discretion
as to its execution, has to be exercised under and in pursuance of
law. President complied with the latter.

CASE: CEBU VS DRILON

HELD: it is a fundamental rule that implementing rules cannot add
or detract from the provisions of law it is designed to implement.
The implementing rules cannot provide for such a prohibition not
contemplated by the law. Administrative regulations adopted
under legislative authority by a particular department must be in
harmony with the provisions of the law, and should be for the sole
purpose of carrying into effect its general provisions. The law itself
cannot be expanded by such regulations. An administrative
agency cannot amend an act of Congress.

delegation to the courts- P. v. DACUYCUY, 5/5/8


CASE: P VS DACUYCUY
Facts:
Private respondents were charged with violation of RA 4670
(Magna Carta for Public School Teachers. They also charged
constitutionality of Sec.32 (be punished by a fine of not less
than P100 nor more than P1000, or by imprisonment, in the
discretion of the court.) of said R.A on grounds that it a.) imposes
a cruel and unusual punishment, b.) constitutes an undue
delegation of legislative power. Judge Dacuycuy ruled that the
said section is a matter of statutory construction and not an undue
of delegation of legislative power.

Issue:
W/N Sec. 6 constitutes undue delegation of legislative power and
is valid.

Held:
NOT VALID! The duration of penalty for the period of
imprisonment was left for the courts to determine as if the judicial
department was a legislative dept. The exercise of judicial power
not an attempt to use legislative power or to prescribe and create
a law but is an instance of the admin. of justice and the app. of
existing laws to the facts of particular cases. Said section violates
the rules on separation of powers and non-delegability of
legislative powers


delegation to local governments- LOCAL
GOVERNMENT CODE; Rubi v. Prov. Board, 39 Phil
660; P. v. Vera, 65 Phil 5

CASE: RUBI VS PROV.

Facts: This is an application for habeas corpus in favor of Rubi
and other Manguianes of the Province of Mindoro.

The provincial board of Mindoro adopted resolution No. 25 which
states that provincial governor of any province in which non-
Christian inhabitants (uncivilized tribes) are found is authorized,
when such a course is deemed necessary in the interest of law
and order, to direct such inhabitants to take up their habitation on
sites on unoccupied public lands to be selected by him and
approved by the provincial board. It is resolved that under section
2077 of the Administrative Code, 800 hectares of public land in the
sitio of Tigbao on Naujan Lake be selected as a site for the
permanent settlement of Mangyanes in Mindoro. Further,
Mangyans may only solicit homesteads on this reservation
providing that said homestead applications are previously
recommended by the provincial governor.

Thereafter, the provincial governor of Mindoro issued executive
order No. 2, which says that the provincial governor has
selected a site in the sitio of Tigbao on Naujan Lake for the
permanent settlement of Mangyanes in Mindoro. In that case,
pursuant to Section 2145 of the Revised Administrative Code,
all the Mangyans in the townships of Naujan and Pola and the
Mangyans east of the Baco River including those in the districts
of Dulangan and Rubi's place in Calapan, were ordered to take
up their habitation on the site of Tigbao, Naujan Lake. Also, that
any Mangyan who shall refuse to comply with this order shall
upon conviction be imprisoned not exceed in sixty days, in
accordance with section 2759 of the revised Administrative
Code.

Said resolution of the provincial board of Mindoro were claimed
as necessary measures for the protection of the Mangyanes of
Mindoro as well as the protection of public forests in which they
roam, and to introduce civilized customs among them.

It appeared that Rubi and those living in his rancheria have not
fixed their dwelling within the reservation of Tigbao and are
liable to be punished.

It is alleged that the Manguianes are being illegally deprived of
their liberty by the provincial officials of that province. Rubi and
his companions are said to be held on the reservation
established at Tigbao, Mindoro, against their will, and one
Dabalos is said to be held under the custody of the provincial
sheriff in the prison at Calapan for having run away form the
reservation.

Issue:

Whether or Not Section 2145 of the Administrative Code of
1917 is constitutional.

Held: The Court held that section 2145 of the Administrative
Code does not deprive a person of his liberty without due
process of law and does not deny to him the equal protection of
the laws, and that confinement in reservations in accordance
with said section does not constitute slavery and involuntary
servitude. The Court is further of the opinion that section 2145
of the Administrative Code is a legitimate exertion of the police
power, somewhat analogous to the Indian policy of the United
States. Section 2145 of the Administrative Code of 1917 is
constitutional.

The preamble of the resolution of the provincial board of
Mindoro which set apart the Tigbao reservation, it will be read,
assigned as reasons fort the action, the following: (1) The failure
of former attempts for the advancement of the non-Christian
people of the province; and (2) the only successfully method for
educating the Manguianes was to oblige them to live in a
permanent settlement. The Solicitor-General adds the following;
(3) The protection of the Manguianes; (4) the protection of the
public forests in which they roam; (5) the necessity of
introducing civilized customs among the Manguianes.

Considered purely as an exercise of the police power, the courts
cannot fairly say that the Legislature has exceeded its rightful
authority. It is, indeed, an unusual exercise of that power. But a
great malady requires an equally drastic remedy. One cannot
hold that the liberty of the citizen is unduly interfered without
when the degree of civilization of the Manguianes is considered.
They are restrained for their own good and the general good of
the Philippines. Nor can one say that due process of law has not
been followed.

CASE: PEOPLE VS VERA
FACTS: Cu Unjieng was convicted by the trial court in Manila.
He filed for reconsideration which was elevated to the SC and
the SC remanded the appeal to the lower court for a new trial.
While awaiting new trial, he appealed for probation alleging that
the he is innocent of the crime he was convicted of. Judge
Tuason of the Manila CFI directed the appeal to the Insular


Probation Office. The IPO denied the application. However, Judge
Vera upon another request by Cu Unjieng allowed the petition to
be set for hearing. The City Prosecutor countered alleging that
Vera has no power to place Cu Unjieng under probation because
it is in violation of Sec. 11 Act No. 4221 which provides that the act
of Legislature granting provincial boards the power to provide a
system of probation to convicted person. Nowhere in the law is
stated that the law is applicable to a city like Manila because it is
only indicated therein that only provinces are covered. And even if
Manila is covered by the law it is unconstitutional because Sec 1
Art 3 of the Constitution provides equal protection of laws. The
said law provides absolute discretion to provincial boards and this
also constitutes undue delegation of power. Further, the said
probation law may be an encroachment of the power of the
executive to provide pardon because providing probation, in effect,
is granting freedom, as in pardon.

ISSUE: Whether or not equal protection is violated when the
Probation Law provides that only in those provinces in which the
respective provincial boards have provided for the salary of a
probation officer may the probation system be applied.

HELD: The act of granting probation is not the same as pardon. In
fact it is limited and is in a way an imposition of penalty. There is
undue delegation of power because there is no set standard
provided by Congress on how provincial boards must act in
carrying out a system of probation. The provincial boards are
given absolute discretion which is violative of the constitution and
the doctrine of the non delegability of power. Further, it is a
violation of equity so protected by the constitution. The challenged
section of Act No. 4221 in section 11 which reads as follows: This
Act shall apply only in those provinces in which the respective
provincial boards have provided for the salary of a probation
officer at rates not lower than those now provided for provincial
fiscals. Said probation officer shall be appointed by the Secretary
of Justice and shall be subject to the direction of the Probation
Office. This only means that only provinces that can provide
appropriation for a probation officer may have a system of
probation within their locality. This would mean to say that
convicts in provinces where no probation officer is instituted
may not avail of their right to probation. The SC declared the
old probation law as unconstitutional.

SEC. 2.
The Senate shall be composed of twenty-four Senators who shall
be elected at large by the qualified voters of the Philippines, as
may be provided by law.

SEC. 3.
No person shall be a Senator unless he is a natural-born citizen of
the Philippines, and, on the day of the election, is at least thirty-
five years of age, able to read and write, a registered voter, and a
resident of the Philippines for not less than two years immediately
preceding the day of the election.

SEC. 4.
The term of office of the Senators shall be six years and shall
commence, unless otherwise provided by law, at noon on the
thirtieth day of June next following their election.
No Senator shall serve for more than two consecutive terms.
Voluntary renunciation of the office for any length of time shall not
be considered as an interruption in the continuity of his service for
the full term for which he was elected.

Composition, qualification and election of senators

SENATE:
a. Composition- 24 senators (elected at large by qualified
Filipino voters)
b. Qualification- natural born citizen, at least 35 years of age on
the day of election, able to read and write, resident for not
less than 2 years immediately preceding the day of election.
c. Term- 6 years, commencing at noon on the 30
TH
day of June
next following their election.

HOUSE OF REP: a. Composition- not more than 250 unless
otherwise provided by law
b. Qualifications- natural-born citizen, at least 25 years old
on the, able to read and write, except the party-list rep-a
registered voter in the district in which he shall be elected,
resident for a period not less than 1 year preceding the election
c. Term- 3 years commencing at noon of the 30
th
day of
June next following their election

May Congress add drug testing as a qualification?
PIMENTEL, JR. vs. COMELEC, G.R. No. 161658,
November 3, 2008

CASE: PIMENTEL VS COMELEC
Facts: Congress passed RA 9165, Comprehensive Dangerous
Drugs Act of 2002, and makes it mandatory for candidates for
public office, students of secondary and tertiary schools, officers
and employees of public and private offices, and persons
charged before the prosecutors office with certain offenses,
among other personalities, to undergo a drug test. Hence,
Senator Pimentel, who is a senatorial candidate for the 2004
synchronized elections, challenged Section 36(g) of the said
law.

Issue: is the mandatory drug testing of candidates for public
office an unconstitutional imposition of additional qualification on
candidates for Senator?


Held: Yes. Section 36 (g) of RA 9165, requiring all candidates
for public office whether appointed or elected both in the
national or local government undergo a mandatory drug test is
UNCONSITUTIONAL. Under Sec.3, Art. VI of the Constitution,
an aspiring candidate for Senator needs only to meet 5
qualifications: (1) citizenship, (2) voter registration, (3) literacy,
(4) age, and (5) residency. The Congress cannot validly amend
or otherwise modify these qualification standards, as it cannot
disregard, evade, or weaken the force of a constitutional
mandate, or alter or enlarge the Constitution. It is basic that if a
law or an administrative rule violates any norm of the
Constitution, that issuance is null and void and has no effect. In
the discharge of their defined functions, the three departments
of government have no choice but to yield obedience to the
commands of the Constitution. Whatever limits it imposes must
be observed.


May a senator may serve more than two terms? Yes.
Provided: the terms are NOT consecutive
SEC. 5.
(1) The House of Representatives shall be composed of not
more than two hundred and fifty members, unless otherwise
fixed by law, who shall be elected from legislative districts
apportioned among the provinces, cities, and the Metropolitan
Manila area in accordance with the number of their respective
inhabitants, and on the basis of a uniform and progressive ratio,
and those who, as provided by law, shall be elected through a
party-list system of registered national, regional, and sectoral
parties or organizations.

Membership in the house may exceed 250 if Congress decides
to create legislative districts. Tobias v. Abalos, 239 SCRA 106
(1994)

CASE: TOBIAS vs. ABALOS

FACTS: RA No. 7675 converts the municipality of Mandaluyong
into a highly-urbanned city (Before, San Juan and Mandaluyong
belonged to one legislative district). Petitioner argues that Act
No. 7675 divides San Juan and Mandaluyong into separate
congressional districts resulting to an increase in the
composition of the House of Representatives stated in Article VI
of the Constitution which is 250 only. Correct?

RULING: No. Article VI shows the present limit which is not
more than 250 members unless otherwise provided by law.
The inescapable import of the latter clause is that the present
composition of congress itself so mandates through a legislative
enactment. The increase in congressional representation
mandated in RA 7675 is NOT unconstitutional.




Only Congress can create provinces and cities because the
creation of provinces and cities necessarily includes the creation
of legislative districts, a power only Congress can exercise under
Section 5, Article VI of the Constitution and Section 3 of the
Ordinance appended to the Constitution. [Sema vs. Commission
on Elections, 558 SCRA 700(2008)]


CASE: SEMA vs. COMELEC

FACTS: RA 9054 was passed. This law grants to the Regional
Assembly or the ARMM the power to create provinces and cities.
ARMM created the province of Shariff Kabunsuan. Valid?

HELD: No. ARMM cannot create a province without a legislative
district because Constitution mandates that every province shall
have a legislative district. Only Congress can create provinces and
cities because the creation of provinces and cities necessarily
includes the creation of legislative districts, a power only Congress
can exercise under Section 5, Article VI of the Constitution and
Section 3 of the Ordinance appended to the Constitution.


(2) The party-list representatives shall constitute twenty per
centum of the total number of representatives including those
under the party list. For three consecutive terms after the
ratification of this Constitution, one-half of the seats allocated to
party-list representatives shall be filled, as provided by law, by
selection or election from the labor, peasant, urban poor,
indigenous cultural communities, women, youth, and such other
sectors as may be provided by law, except the religious sector.

RA 7941- party list-law
AN ACT PROVIDING FOR THE ELECTION OF PARTY-LIST
REPRESENTATIVES THROUGH THE PARTY-LIST SYSTEM,
AND APPROPRIATING FUNDS THEREFOR
The party-list system is a mechanism of proportional
representation in the election of representatives to the House of
Representatives from national, regional and sectoral parties or
organizations or coalitions thereof registered with the Commission
on Elections (COMELEC).

Qualifications of Party-List Nominees. No person shall be
nominated as party-list representative unless he is a natural-born
citizen of the Philippines, a registered voter, a resident of the
Philippines for a period of not less than one (1)year immediately
preceding the day of the election, able to read and write, a bona
fide member of the party or organization which he seeks to
represent for at least ninety (90) days preceding the day of the
election, and is at least twenty-five (25) years of age on the day of
the election.
In case of a nominee of the youth sector, he must at least be
twenty-five (25) but not more than thirty (30) years of age on the
day of the election. Any youth sectoral representative who attains
the age of thirty (30) during his term shall be allowed to continue in
office until the expiration of his term.

VETERANS FEDERATION PARTY et al., vs. COMELEC, et al.,
342 SCRA 244 [10-6-00]- the provision provides for the ceiling;-
RA 7941, providing for requirement of 2% of the total votes cast to
qualify for one representative is valid.

CASE: VETERANS vs. COMELEC
FACTS: RA 7941 provides for the 2% threshold requirement and
the 3-seat limit. Is it constitutional?

RULING: Yes. In imposing the 2% threshold, the Congress
wanted to ensure that only those parties, organizations and
coalitions, having a sufficient number of constituents deserving of
representation, are actually presented in Congress and to ensure
meaningful legal representation.


Who may participate in the party-list election? Ang Bagong
Bayani et al., vs. COMELEC, et al., GR 147589, June 26, 2001




CASE: BAGONG BAYANI VS COMELEC

Facts
Petitioners challenged the Comelecs Omnibus Resolution No.
3785
,
which approved the participation of 154 organizations and
parties, including those herein impleaded, in the 2001 party-list
elections. Petitioners sought the disqualification of private
respondents, arguing mainly that the party-list system was
intended to benefit the marginalized and underrepresented; not
the mainstream political parties, the non-marginalized or
overrepresented. Unsatisfied with the pace by which Comelec
acted on their petition, petitioners elevated the issue to the
Supreme Court.

Issue:
Whether or not political parties may participate in the party
list elections.

Ruling:
Political parties even the major ones -- may participate in
the party-list elections subject to the requirements laid down in
the Constitution and RA 7941, which is the statutory law
pertinent to the Party List System.

Under the Constitution and RA 7941, private respondents
cannot be disqualified from the party-list elections, merely on
the ground that they are political parties. Section 5, Article VI of
the Constitution provides that members of the House of
Representatives may "be elected through a party-list system of
registered national, regional, and sectoral parties or
organizations . It is however, incumbent upon the Comelec to
determine proportional representation of the marginalized and
underrepresented, the criteria for participation, in relation to
the cause of the party list applicants so as to avoid desecration
of the noble purpose of the party-list system.


BANAT vs. COMELEC, 592 SCRA 294- Clarifying and
summarizing the application of 2% vote requirement and the 3-
seat cap. under RA 7941.

CASE: BANAT vs. COMELEC

RULING: To summarize,
1. 20% of total membership of House of Representatives is
maximum number of seats.
2. Garnering 2% total votes cast guarantees a party list
organization one seat. The guaranteed seat shall be distributed
in a first round of seat allocation to parties receiving at least 2%.
3. Additional seats, that is, the remaining seats after allocation
of the guaranteed seats, shall be distributed to the party list
organization including those that received less than 2% total
votes. The continued operation of the 2% threshold as it applies
to the allocation of the additional seat is now unconstitutional
because this threshold mathematically and physically prevents
the filling up of the available party list seat.
4. 3-seat cap is constitutional, to prevent any party from
dominating the party list system.

BELLO v. COMELEC, Dec. 7, 2010- jurisdiction over the
qualifications of party list nominee.
New parameters on party list (ATONG PAGLAUM, INC. v.
COMELEC, GR No. 203766, 4/3/13, consolidated cases)

CASE: ATONG VS COMELEC

RULING: what are now the new parameters to be followed by
comelec?

They are as follows:

1. three different groups may participate in the party-list system:
(1) national parties or organizations, (2) regional parties or
organizations, and (3) sectoral parties or organizations.

2. national parties or organizations and regional parties or
organizations do not need to organize along sectoral lines and
do not need to represent any marginalized and
underrepresented sector.



3. political parties can participate in party-list elections provided
they register under the party-list system and do not field
candidates in legislative district elections. a political party, whether
major or not, that fields candidates in legislative district elections
can participate in partylist elections only through its sectoral wing
that can separately register under the party-list system. the
sectoral wing is by itself an independent sectoral party, and is
linked to a political party through a coalition.
4. sectoral parties or organizations may either be marginalized
and underrepresented or lacking in well-defined political
constituencies. it is enough that their principal advocacy pertains
to the special interest and concerns of their sector. the sectors that
are marginalized and underrepresented include labor, peasant,
fisherfolk, urban poor, indigenous cultural communities,
handicapped, veterans, and overseas
62 rule 64 in relation to rule 65, 1997 rules of civil procedure.
workers. the sectors that lack well-defined political constituencies
include professionals, the elderly, women, and the youth.

5. a majority of the members of sectoral parties or organizations
that represent the marginalized and underrepresented must
belong to the marginalized and underrepresented sector they
represent. similarly, a majority of the members of sectoral parties
or organizations that lack well-defined political constituencies
must belong to the sector they represent. the nominees of sectoral
parties or organizations that represent the marginalized and
underrepresented, or that represent those who lack well-defined
political constituencies, either must belong to their respective
sectors, or must have a track record of advocacy for their
respective sectors. the nominees of national and regional parties
or organizations must be bona-fide members of such
parties or organizations.

6. national, regional, and sectoral parties or organizations shall not
be disqualified if some of their nominees are disqualified, provided
that
they have at least one nominee who remains qualified.

(3) Each legislative district shall comprise, as far as practicable,
contiguous, compact, and adjacent territory. Each city with a
population of at least two hundred fifty thousand, or each province,
shall have at least one representative.

What is Gerrymandering?
ALDABA et al., vs. COMELEC, 611 SCRA 137 [2010]

ALDABA vs. COMELEC
HELD: The constitutional check against Gerrymandering, which
means the creation out of separate points of territory in order to
favor a candidate, is found in Section 5 (3), Article VI of the
Constitution.

You might also like